Você está na página 1de 144

PG

SERIES
PEDIATRIC
BUSTER
with High-Yield Facts

PG
SERIES

PEDIATRIC
BUSTER
with High-Yield Facts
(Based on latest revised edition of OP Ghai: Textbook of Pediatrics, 5th edition
and Nelson: Textbook of Pediatrics, 17th edition)

Compiled by

Dharmendra Sharma

Rinju Sharma

MBBS, MD

MBBS, MD

Consultant Cardiologist
Life Line Heart Centre and Hospital
Kamla Nagar, Bypass Road
Agra

Consultant Gynaecologist
Life Line Heart Centre and Hospital
Kamla Nagar, Bypass Road
Agra

JAYPEE BROTHERS
MEDICAL PUBLISHERS (P) LTD
New Delhi

Published by
Jitendar P Vij
Jaypee Brothers Medical Publishers (P) Ltd
EMCA House, 23/23B Ansari Road, Daryaganj
New Delhi 110 002, India
Phones: +91-11-23272143, +91-11-23272703, +91-11-23282021, +91-11-23245672
Fax: +91-11-23276490, +91-11-23245683 e-mail: jpmedpub@del2.vsnl.net.in
Visit our website: www.jaypeebrothers.com

Branches
202 Batavia Chambers, 8 Kumara Krupa Road, Kumara Park East,
Bangalore 560 001, Phones: +91-80-22285971, +91-80-22382956, +91-80-30614073
Tele Fax: +91-80-22281761 e-mail: jaypeebc@bgl.vsnl.net.in
282 IIIrd Floor, Khaleel Shirazi Estate, Fountain Plaza
Pantheon Road, Chennai 600 008, Phones: +91-44-28262665, +91-44-28269897
Fax: +91-44-28262331 e-mail: jpmedpub@md3.vsnl.net.in
4-2-1067/1-3, Ist Floor, Balaji Building, Ramkote
Cross Road, Hyderabad 500 095, Phones: +91-40-55610020, +91-40-24758498
Fax: +91-40-24758499 e-mail: jpmedpub@rediffmail.com
1A Indian Mirror Street, Wellington Square
Kolkata 700 013, Phone: +91-33-22451926 Fax: +91-33-22456075
e-mail: jpbcal@cal.vsnl.net.in
106 Amit Industrial Estate, 61 Dr SS Rao Road, Near MGM Hospital
Parel, Mumbai 400 012, Phones: +91-22-24124863, +91-22-24104532, +91-22-30926896
Fax: +91-22-24160828 e-mail: jpmedpub@bom7.vsnl.net.in

PG SeriesPediatric Buster
2005, Dharmendra Sharma, Rinju Sharma
All rights reserved. No part of this publication should be reproduced, stored in a retrieval system,
or transmitted in any form or by any means: electronic, mechanical, photocopying, recording,
or otherwise, without the prior written permission of the editors and the publisher.
This book has been published in good faith that the material provided by editors is
original. Every effort is made to ensure accuracy of material, but the publisher, printer
and editors will not be held responsible for any inadvertent error(s). In case of any
dispute, all legal matters to be settled under Delhi jurisdiction only.

First Edition: 2005


ISBN

81-8061-430-1

Typeset at JPBMP typesetting unit


Printed at Gopsons Papers Ltd., Sector 60, Noida

To
Our Parents
and
Our Patients

Tips to the Readers


First of all Best of Luck for starting the preparation
Always remember to have a thorough command on the subject
Keep the text and MCQs hand in hand; divide your time with 60% text and 40% question in first 6 months
of study and then shift the proforma
Always try to consult standard textbooks and recent advances; sometimes newspaper gives you the answer
before the text appears in books, viz Viagra
Dont cram the answers; try to understand the basics of topic and see other topics in its surrounding
Discussion will help you a lot in understanding the facts
Answers are right to our extent but for the possibility of computer and typographical error we apologise
in advance
Any correction with references, new questions and suggestions are welcomed by our readers. They will
be duly acknowledged in our further editions.
For reference see
OPG5th Textbook of Pediatrics (fifth edition)
N17th

Textbook of Pediatrics (seventeenth edition)

Preface
Its never too late to start. Adding to the PG series a new subject pediatrics, again
conceptualized and arranged chapterwise with references based on latest revised 5th edition
of OP Ghai and 17th edition of Nelson. Little text has been put in proximity only for revision
not for the sake to skip the textbooks. Always remember this is our way to make you revise
the subject and a testimony about how much you have grabbed after going through text.
Questions are all recent ones. So, start at the very earliest as every minute counts and
be systematic in your approach as it is what that matters.
Special thanks to Mr. JP Vij, CMD, Jaypee Brothers Medical Publishers (P) Ltd for his
personal interest. We wish to express our thanks to all our associates and colleagues.
Suggestion and criticism are always welcome and sorry for any typographical error.
Best of luck,

Dharmendra Sharma
Rinju Sharma

Contents
1. Growth and Development .................................................................................................................... 1
2. Disorders of Growth and Development ........................................................................................... 6
3. Adolescent Health and Development ................................................................................................ 9
4. Nutrition and Nutritional Disorders .............................................................................................. 11
5. Newborn Infants ................................................................................................................................... 21
6. Immunity and Infectious Diseases .................................................................................................. 39
7. Gastrointestinal System ..................................................................................................................... 50
8. Hematological Diseases ...................................................................................................................... 55
9. Diseases of Ear, Nose, Throat and Respiratory System ........................................................... 61
10. Cardiovascular System ....................................................................................................................... 66
11. Kidney and Urinary Tract ................................................................................................................. 75
12. Endocrine and Metabolic Disorders ............................................................................................... 81
13. Central Nervous System ..................................................................................................................... 86
14. Malignancies in Childhood ................................................................................................................ 97
15. Collagen Vascular Disorders and Genetics ................................................................................. 103
16. Inborn Errors of Metabolism .......................................................................................................... 105
17. Miscellaneous ...................................................................................................................................... 108
18. Self-assessment ................................................................................................................................... 115

Growth and Development

Human fetus secretes thyroxine from 12 wk of gestation.


Growth is fast in 1st half of gestation.
Order of growth is cephalocaudal and distal to proximal.
Brain enlarges rapidly during later months of foetal life and early months of postnatal life.
Brain reaches 90% of adult head size by 2 yr.
The growth spurt of puberty is attributed to neurohormonal stimulation of hypophysis by
hypothalamus.
Foetal or maternal growth hormone is not essential for growth in utero
Insulin stimulates foetal growth
Toddler means 1-3 yr of child.
Weight of child doubles in
4-5 months.
Triple in 1 yr.
4 time in 2 yr.
5 time in 3 yr.
Height at Birth - 45-50 cm
3 mon - 60 cm
1 yr
- 75 cm
4 yr
- 100 cm (double)

1. Consider the following statements with reference to the normal child development:
(OPG5th/1)(UPSC/04)
(1) The sequence and time of attainment of milestones may vary among individual children, (2) Development
always proceeds in a cephalocaudal sequence,
(3) Presence of primitive reflexes is essential for attaining
voluntary actions, (4) Development depends on the myelination and integrity of central nervous system.
Which of these statements is/are true interpretation of the laws of normal child development?
(a) 1 only
(b) 1 and 3
(c) 2 and 4
(d) 1, 2, 3 and 4
2. A newborn baby has a head circumference of 35 cm at birth. His optimal head circumference will be 43 cm at:
(OPG5th/4) (UPSC/99)
(a) 4 months of age
(b) 6 months of age
(c) 8 months of age
(d) 12 months of age
3. Increase in height in first year is by:
(OPG5th/4)
(DNB 2001)
(a) 40%
(b) 50%
(c) 60%
(d) 75%
4. Birth weight of a child doubles at five months of age while the birth length doubles at the age of:
(a) 1 year
(b) 2 years
(c) 3 years
(d) 4 years
(OPG5th/4) (UPSC 98)
5. Increase is length in the first year of life is:
(OPG5th/4) (AIIMS 81)
(a) 12 cm
(b) 15 cm
(c) 25 cm
(d) 35 cm

1 D

2 D

3 B

4 D

5 C

2 / Pediatric Buster
6. The optimum body weight of a baby by the end of one year is:
(a) Addition of three kg to birth weight
(b) Double the birth weight
(d) Four times the birth weight
7. Height of children 2-10 years increased by:
(a) 2 cm/year
(b) 3 cm/year
(c) 6 cm/year
(d) 10 cm/year
8. The infant triples weight at the age of:
(a) 5 months
(b) 11 months
(c) 18 months
(d) 2 years

(OPG5th/4)(UPSC/03)
(c) Three times the birth weight
(OPG5th/4) (AI 97)
(OPG5th/4) (AIIMS-96)

Weechs formula for estimating weight and height for age of normal children
Weight
Kilograms
Pounds
At birth
3.25
7
3 to 12 months
age in months + 9
Age in months + 11
2
1 to 6 years
(age in years 2) + 8
(age in years 5) + 17
7 to 12 years
(age in years 7) + 5
(age in years 7) - 5
2
Height
Centimetres
Inches
At birth
50
20
At 1 year
75
30
2 to 12 years
(age in years 6) + 77
(age in years 2) + 30
9. Two carpal bones are radiologically seen in the wrist X-ray of most of the children by the end of:
(a) 1 year
(b) 2 years
(c) 3 years
(d) 4 years
(OPG5th/6) (UPSC 96)
An average full-term newborn has the following 5 radiologically demonstrable ossification centers:
Distal end of femur
Proximal end of tibia
Talus
Calcaneus
Cuboid
By the age of 6 months, ossification centers for two carpal bones, i.e. capitate and hamate, appear
that number of centers at wrist is equal to age in years plus one. Thus, a child of 2 years should have
3 centers.
10. First permanent tooth to erupt is:
(a) 1st premolar
(b) 1st molar

(OPG5th/6) (AI 88)


(c) 1st incisor

Eruption sequence of temporary dentition


Central incisors
6-7 months
Lateral incisors
8-9 months
Canine
16-18 months
First molar
12-14 months
Second molar
20-24 months
Eruption sequence of permanent dentition
Central incisors
7-8 years
Lateral incisors
8-9 years
Canine
11-12 years
First premolar
10-11 years
Second premolar
10-12 years
First molar
6-7 years
Second molar
12-13 years
6 D

7 C

8 B

9 A

10 A

(d) 2nd premolar

Growth and Development / 3


11. A child starts crawling at:
(OPG5th/42) (AP 88)
(a) 5 months
(b) 7 months
(c) 8 months
(d) 9 months
12. An infant begins to sit with support by.....months:
(OPG5th/44) (PGI 88)
(a) 4
(b) 5
(c) 6
(d) 7
13. Which of the following acts can a one-year-old child perform?
(OPG5th/44) (AIIMS 83)
(a) Sits down from standing position
(b) Drinks from cup
(c) Speaks one or two words with meaning
(d) Rings bell purposefully
Developmental Milestones in the First 2 Yr of Life
Gross motor
Head steady in sitting
Runs alone
Pull to sit, no head lag
Walks alone
Hands together in midline
Rolls back to stomach
Sits without support
Asymmetric tonic neck reflex gone

months
2.0
16.0
3.0
12.0
3.0
6.5
6.0
4.0

Cognitive
Stares momentarily at spot where object disappeared
(e.g., yarn ball dropped)
Pretend play with doll (gives doll bottle)
Stares at own hand
Bangs two cubes
Uncovers to (after seeing it hidden)
Egocentric pretend play
(e.g. pretends to drink from cup)
Uses stick to reach toy

17.0
4.0
8.0
8.0
12.0
17.0

Communication and Language


Follows one-step command with gesture
Follows one-step command without gesture
Smiles in response to face, voice
Monosyllabic babble
Speaks 4-6 words
Speaks 10-15 words
Speaks two-word sentences
(e.g., Mommy shoe)
Inhibits to no
(e.g., Give it to me)
Speaks first real word

7.0
10.0
1.5
6.0
15.0
18.0
19.0
7.0
10.0
12.0

Fine Motor
Palmar grasp gone
Grasps rattle
Reaches for objects
Transfers object hand to hand
Scribbles
Builds tower of two cubes
Builds tower of six cubes
Thumb-finger grasp
Turns pages of book

4.0
3.5
4.0
5.5
13.0
15.0
22.0
8.0
12.0

11 C

12 B

13 C

2.0

4 / Pediatric Buster
14. Pincer grasp develops in an infant at:
(OPG5th/44) (PGI 88)
(a) 8 months
(b) one year
(c) 18 months
(d) 2 years
15. Milestones at 10 months of age are:
(OPG5th/44) (PGI 88)
(a) Pincer grasp
(b) Arranging 3 cubes
(c) Walks few steps without support
(d) Says mama dada
16. In a child, one should be worried if:
(OPG5th/44) (PAL 93)
(a) Stammering occurs at 3 years
(b) Lack of toilet control at 2.5 years
(c) Teeth do not erupt by 11 months
(d) Social smile absent by 10 weeks
17. Child recognizes and smiles at:
(OPG5th/44) (AIIMS 98)
(a) 12 months
(b) 6 months
(c) 4 months
(d) 2 months
18. Which about development is incorrect:
(OPG5th/44) (AIIMS 95)
(a) 2 years old can use pleurals
(b) Social smile at 3 months
(c) Pincer grasp at 3 months
(d) Sitting at 6 months
19. An infant is able to grasp a rattle and transfer it from one hand to the other by the age of: (OPG5th/44) (AI 88)
(a) 3 months
(b) 6 months
(c) One year
(d) 2 years
20. Head control is possible in an infant by...months:
(OPG5th/44) (JIPMER 87)
(a) 1
(b) 2
(c) 3
(d) 6
21. A six-week-old infant cannot:
(OPG5th/44) (AIIMS 92)
(a) Grasp dangling objects
(b) Fix gaze
(c) Lift and hold head
(d) Turn head towards sound
22. A 3-year-old can do all except:
(OPG5th/44) (AIIMS 92)
(a) Walk properly
(b) Eat with a spoon
(c) Vacobulary of 20-100 words
(d) Climb stairs running
23. Consider the following factors :
(OPG5th/44)(UPSC/04)
(1) Head control (2) Social smile (3) Crawls (4) Sits up
The correct sequence in the developmental milestones durin the first year of life is:
(a) 1, 2, 3, 4
(b) 2, 1, 4, 3
(c) 1, 4, 3, 2
(d) 2, 1, 3, 4
24. Absence of following sign will be considered normal is 3 months old child:
(OPG5th/44) (AIIMS 2000)
(a) Babbling
(b) Head raising upto 90
(c) Shift red ring from one hand to other
(d) Smile to familiar
faces
25. Regarding the developmental stage of a newborn, which of the following is/are true: (OPG5th/44) (PGI 2000)
(a) He can fix his eyes on a bright object
(b) Can focus on an object 8-12inches away
(c) Turns to one side
(d) Grasps a rattle
(e) Elevates his head on the neck developmental delay
Motor Development. Developmental delay should be suspected if the child is not able to
(i) Pull up to sit by 4th months;
(ii) Roll over by 5 months;
(iii) Sit without support by 7-8 months;
(iv) Stand holding on by 9-10 months;
(v) Walk by 15 months;
(vi) Climb up or down the stairs by 2 years;
(vii) Jump with both feet by 2 1/2 years;
(viii) Stand momentarily on one foot by 3 years;
(viii) Hop by 4 years and walk in a straight line back and forth or balance on one foot for 5-10 seconds
by 5 years.
26. A 3-year-child is expected to do all except:
(OPG5th/46) (AIIMS 94)
(a) Copy a circle
(b) Speak in sentences
(c) Climb up stairs and down stairs
(d) Hobble five steps
27. A normal infant sits briefly leaning forward on her hands, reaches for and grasps a cube and transfers it from
hand to hand. She babbles but cannot wave bye-bye nor can she grasp objects with the finger and thumb. Her
age is:
(OPG5th/46) (UPSC/2K)
(a) 4 months
(b) 7 months
(c) 10 months
(d) 14 months
28. A child climbs with alternate steps, builds a tower of 8-9 cubes, tellsI but not his name and cannot say his
age and sex. The probabale age is:
(OPG/ 6th/46) (AIIMS /MAY/01)
(a) 24 Months
(b) 36 Months
(c) 30 Months
(d) 48 Months
14 A

15 D

16 D

17 D

18 C

19 B

20 C

21 A

22 D

23 B

24 C

25 A

26 D

27 B

28 A

Growth and Development / 5


29. A child can walk up and down the stairs alternating his feet by the age of:
(OPG5th/46) (AI 90)
(a) 12 months
(b) 24 months
(c) 36 months
(d) 48 months.
30. A two-month old child is able to:
(OPG5th/46) (AI/04)
(a) Show a positive parachute protective reflex
(b) Hold head steady in a seated position
(c) Lift head and
chest off a flat surface with extended elbows
(d) Sustain head level with the body when palced in ventral
suspension
31. Four years old child can do following moteractivity:
(OPG5th/46) (AIIMS 98)
(a) Hoping for 15 feets
(b) Standing on one leg for 20 sec
(c) Staircase climbing with alernate leg
(d) None of the above
32. A girl can walk upstairs, alternating feet, and make a tower of 8 cubes. She refers to herself by the pronoun I,
knows her name, but not her age and gender. The most likely chronological age is:
(OPG5th/46)(UPSC/02)
(a) 24 months
(b) 30 months
(c) 36 months
(d) 42 months
33. In children,the following are true of speech development, except:
(OPG5th/47).(UPSC/03)
(a) children pronounce simple sounds such as da-da at 8 months.
(b) children voice single,intelligible words
at 13 months
(c) simple 2- to 3-word sentence sentences are composed at 15 months (d) children can identify
4 to 5 picture cards at 2 years
34. Visual acuity reaches adult acuity by the age:
(OPG5th/47) (JIPMER 81, DNB 89)
(a) 7
(b) 9
(c) 5
(d) 3
35. Consider the following statements:
(OPG5th/48)(UPSC/04)
A 15 months old can
1. Crawl upstairs
2. Make a tower of
3. Name a familiar object
4. Indicate desire by pointing
Which of these statements is/are correct?
(a) 1 only
(b) 1 and 2
(c) 2 and 4
(d) 1, 2, 3 and 4
36. A 3-year-old child is able to perform all of the following activities except:
(OPG5th/48) (UPSC/01)
(a) Climbing stairs with alternate feet
(b) Riding atricycle
(c) Cursive writing
(d) copying a circle
37. A three-year-old child can do which of the following except:
(OPG5th/48) (UPSC/01)
(a) Ride a tricycle
(b) build a tower often cubes
(c) Knows his age and sex
(d) Use scissors to cut out
pictures
38. Handedness develops at:
(OPG5th/48) (UPSC/01)
(a) 12 months
(b) 18 months
(c) 24 months
(d) 36 months
39. IUGR is caused by all except:
(OPG5th/48)
(a) Diabetes
(b) Alcohol
(c) Smoking
(d) Chronic renal failure
40. Intrauterine growth retardation can be caused by all except:
(OPG5th/51) (AIIMS 87)
(a) Nicotine
(b) Alcohol
(c) Propranolol
(d) Phenothiazine
PROBLEMS OF IUGR INFANTS
Problem
Pathogenesis
Intrauterine fetal demise
Hypoxia, acidosis, infection, lethal anomaly
Perinatal asphyxia
Uteroplacental perfusion during labor chronic fetal hypoxiaacidosis; meconium aspiration syndrome
Hypoglycemia
Tissue glycogen stores, gluconeogenesis, hyperinsulinism, glucose
needs of hypoxia, hypothermia, large brain
Polycythemia-hyperviscosity
Fetal hypoxia with erythropoietin production Hypoxia,
Reduced oxygen consumption/
hypoglycemia, starvation effect, poor subcutaneous fat stores
hypothermia
Dysmorphology
Syndrome anomalads, chromosomal-genetic disorders,
oligohydramnios-induced deformations, TORCH infection

29 C

30 D

31 C

32 C

33 C

34 A

35 B

36 C

37 D

38 C

39 A

40 D

Disorders of Growth and


Development

1. The most important anthropometric characteristic of nutritional dwarfism is:


(OPG5th/50) (AIIMS 83)
(a) Normal weight for chronological age
(b) Normal birth weight and subsequent retardation of growth
(c) Normal weight for height
(d) Normal weight for chronological age
2. Characteristic features of growth hormone deficiency include all of the following except:(OPG5th/51) (AIIMS 83)
(a) Short stature since birth
(b) Symptomatic hypoglycaemia
(c) Delayed tooth eruption
(d) Sexual infantilism
Growth Hormone Deficiency is characterized by:
1.
Height below 3rd percentile.
2.
Prepubertal growth velocity less than 4 cm per year.
3.
Bone age below the chronological age.
4.
Abnormal 24-hour GH secretory pattern.
5.
Peak GH levels less than 10 ng/mL during provocative stimulation tests.
6.
Low IGF-1 and IGFBP-3 levels for age.
7.
Resumption of growth following GH administration.
3. Short stature secondary to growth hormone deficiency is associated with:
(OPG5th/51) (AI 88)
(a) Low birth weight
(b) Normal epiphyseal development
(c) Height age equal to skeletal age
(d) Normal body proportions
Short Stature: When child is below 3rd percentile or less than 2-S.D.
Growth hormone deficiency : Child is normal in weight and height at birth.
Symmetrical dwarfism; child gain less than 4 cm in height/yr.
Bone age significantly delayed.
Intelligence normal but gonadal development is infantile with delay in secondary sex character.
Larons dwarf: Due to reduced somatomedin levels with N or GH.
Hypothyroidism: Asymmetrical dwarfism; Bone age delayed.
Genetic short stature: Gain in height is more than 4 cm/yr. Bone age and body proportions are normal.
Psychosocial short stature: Due to emotional deprivation. Symmetrical devarfism Bone age normal or
delay.
4. Deficiency of growth hormone leads to:
(OPG5th/51) (AI 89)
(a) Delayed fusion of epiphysis
(b) Proportionate dwarfism
(c) Acromegaly
(d) Mental retardation
5. Consider the following statements: Disproportionate dwarfism is a feature of:
(OPG5th/52) (UPSC 98)
(a) Hypopituitarism
(b) Hypothyroidism
(c) Achondroplasia
(d) Morquios disease
6. Upper and lower portion of body disproportion with growth retardation characteristically seen in:
(OPG5th/52) (AI 93)
(a) Down syndrome
(b) Turners syndrome
(c) Cerebral palsy
(d) Congenital hypothyroidism
1 B

2 A

3 D

4 B

5 B

6 D

Disorders of Growth and Development / 7


Upper segment
Lower segment ratio
Proportionate
(normal ratio for age)
High ratio
(upper segment > lower segment)
Low ratio
(upper segment < lower segment)
7. Bone age is normal in:
(a) Genetic short stature
deprivation

Probable disorder
Delayed adolescence, Hypopituitarism, Constitutional
dwarfism, Nutritional dwarf
Hypothy, Achondroplasia, Ellis-van Creveld syndrome,
Turner syndrome
Hurlers syndrome, Morquios
syndrome, hypogonadism

(b) Hypopituitarism

(c) Hyperpituitarism

(OPG5th/52) (JIPMER 81, AIIMS 92)


(d) Hypothyroidism
(e) Emotional

Bone age is advance of height age in genetic short stature


Chondrodystrophy
Down syndrome
Turner syndrome
Storage disorder
8. Retardation of skeletal maturity can be caused by all except:
(OPG5th/51) (AI/04)
(a) Chronic renal failure (b) Hypothyroidism (c) Protein energy malnutrition (PEM) (d) Congenital adrenal
hyperplasia.
9. Which is incorrect about thumb sucking:
(OPG5th/55) (JIPMER 91)
(a) Can lead to malocclusion
(b) Is a source of pleasure
(c) Is a sign of insecurity
(d) Must be treated
vigorously in the first year
10. To avoid displacement of permanent teeth finger sucking should be terminated by:
(a) 8 years
(b) 5 years
(c) 3 years
(d) 2 years
11. True breath holding attacks generally do not occur after:
(OPG5th/55) (PGI 80, AIIMS 81,85)
(a) 1 year
(b) 5 years
(c) 4 years
(d) 2 years
(e) 18 months

Breath holding spells are common b/w 6 mon - 5 yr, but if hypoxia continues for 10-15 sec convulsion
may occur. Kindness and understanding is helpful.

12. Consider the following statements:


Breath holding spells in children have the following features:
(OPG5th/55)(UPSC/02)
(1) They are preceded by minor trauma
(2) Child stops breathing and is cyanosed
(3) Child becomes suddenly pale and limb
(4) Frequency becomes less on zinc supplementation
Which of the above statements are correct?
(a) 1, 2 and 3
(b) 1 and 2
(c) 2, 3 and 4
(d) 1, 3 and 4
(OPG5th/55) (PGI 79, AIIMS 78, 81)
13. Colic generally disappears by age:
(OPG5th/56) (UPSC 83, AMC 81)
(a) 1 year
(b) 2 years
(c) 4 months
(d) 8 months
14. The most common cause of enuresis in children is:
(OPG5th/62) (UPSC 96)
(a) Urinary tract infection
(b) Psychologic stress
(c) Diabetes mellitus
(d) Spina bifida
15. Mother comes with problem of bed wetting by her child. What should be given to the baby:
(OPG5th/62) (AIIMS 99)
(a) Imipramine
(b) Amitriptyline
(c) Fluoxetine
(d) Trazodone
16. A mother of 5 years child presented to a pediatrician with complaint of child enuresis all of the following
statement are true except:
(OPG5th/62) (AIIMS/2000)
(a) Reassure the mother
(b) The complaint can be completely cured by medicine
(c) Condition will resolve
spontaneously
(d) It is due to faulty toilet training by parents
7 A

8 D

9 D

10 B

11 B

12 A

13 C

14 B

15 A

16 B

8 / Pediatric Buster
Enuresis (Bed wetting): Diagnosis is made when urine is voided twice a week for at least 3
consecutive months.
Management of enuretic child should begin with behavioral treatment.
17. All of the following are essential features of attention deficit hyperactivity disease (ADHD) except:
(OPG5th/64) (AI/04)
(a) Lack of concentration
(b) Impulsivity
(c) Mental retardation
(d) Hyperactivity
The important clues to dyslexia in school age children include delayed language, trouble rhyming words,
mispronunciations, hesitations and word-finding difficulties.
Attention-Deficit/Hyperactivity Disorder: Characterized by inattention, including increased distractibility and difficulty sustaining attention; poor impulse control and decreased self inhibitory
capacity and motor overactivity and motor restlessness.
ADHD commonly occurs following:
1. Damage to the CNS (prematurity or traumatic brain injury)
2. Toxic exposure (e.g. foctal alcohol syndrome or lead poisoning)
3. Mal development (e.g. Mental retardation syndromes)
4. Abnormalities in the dopamine transporter gene, the D 4 receptor gene and human thyroid
receptor beta gene.
Behaviors must begin before the age of 7 year and must be present for at least 6 months and
impair the childs ability to function normally.
In at least 80% of affected children symptoms of ADHD persist into adolescence and adulthood.
18. One of the following statements regarding childhood autism is false:
(OPG5th/65) (KERALA/94)
(a) It represents a type of childhood neurotic disorder (b) Emergence of speech and social smile is often delayed
(c) Parental attitude and behaviour do not attribute to it
(d) Isolated area of remarkably talent may be
developed
19. Autism refers to:
(OPG5th/65) (PGI 2000)
(a) Neurodevelopmental delay (b) Condition seen following head injury (c) Deafness (d) Impaired reciprocal
social and interpersonal interaction
Autism: Characterized by1. Severe deficiency in social knowledge and reciprocral social interaction
2. Abnormality in language and nonverbal communication.
3. Activities and interests of patients are limited.
4. Patient may develop generalized tonic clonic or psychomotor seizures.
5. 70% patients are retarded.

17 C

18 A

19 A

Adolescents Health and


Development

Adolescents are 22.8% of Indian


WHO definition Adolescent
Youth
Young people

population
10-19 years
15-24 years
10-24 years

1. Order of development of secondary sexual characteristic in male:


(OPG5th/68) (PGI 96)
(a) Testicular development-pubic hairaxillary hairbeard
(b) Pubic hair-testicular developmentaxillary
hairbeard
(c) Testicular developmentbeardpubic hairaxillary hair
(d) Axillary hairbeardpubic
hairtesticular development
2. Increase in muscle mass at adolescence is probably caused by:
(OPG5th/68) (AIIMS 81)
(a) Thyroid hormone
(b) Adrenal hormone
(c) None
(d) Both
3. Peak stage in height growth corresponds to....stage of pubic hair:
(OPG5th/68) (JIPMER 95)
(a) I
(b) II
(c) III
(d) IV
Adolescent stages
Parameter

Early

Middle

Late

Age (years)
SMR stages
Changes

9 to 13
1 to 2
(i) Beginning of rapid growth
(ii) Appearance of secondary
sexual characterstics.

14 to 16
3 to 4
(i) Peak height growth
(ii) Menarche/
spermarche

17 to 20
5
Slower growth

4. A homosexual stage in early adolescence:


(OPG5th/69) (PGI 81, AIIMS 82)
(a) Indicates sibling rivalry
(b) Indicates family tension
(c) Considered to be normal phenomenon
(d) Indicates parental insecurity
5. Earliest sign of puberty in male:
(OPG5th/69) (UP 94)
(a) Testicular enlargement
(b) Pubarche
(c) Moustache
(d) Change in voice
6. Growth spurt in adolescent girl is:
(OPG5th/69) (AIIMS-96)
(a) Pubic hair
(b) Breast enlargement
(c) Axillary hair
(d) Clitoris enlargement
7. Sequence of development in puberty of a female is:
(OPG5th/69) (AI/2000)
(a) Thelarche-Pubarche-Menarche
(b) Thelarche-Menarche-Pubarche
(c) Menache-Thelarche-Pubarche
(d) Pubarche-Thelarche-Menarche

1 A

2 B

3 B

4 C

5 A

6 A

7 A

10 / Pediatric Buster

Sequence of puberty:
In girls:
1. Breast development (Thelarche)
2. Public hair (Pubarche)
3. Peak growth velocity
4. Menarche (2 yr after pubic hair start)
In boys: Testis Penis Public hair Axillary hair appearing
Peak stage in growth corresponds to 2nd stage in pubic hair in girls and to 3rd stage in boys.
In first two yr after menarche; menstruation is irregular; cycles may be anovulatory.

8. In adolescence, maximum gain in height for girls is observed during


(a) Post-menstrual stage
(b) Pre-menstrual stage
(c) Sexual maturity stage-1
stage II-III
ADOLESCENT-FRIENDLY HEALTH SERVICES
The Generic AFHS Menu
Sexual and reproductive health education
Reproductive health services
Contraception
Pregnancy testing and options
Prenatal and postpartum care
Well baby care (adolescent mothers)
Nutritional services
MTP
STD/HIV screening, counseling and treatment
Counseling regarding life skill development
Screening for various disorders
Growth and development monitoring
Anticipatory guidance regarding substance abuse and other risk taking behavior

8 C

(OPG5th/69)(UPSC/03)
(d) Sexual maturity

Nutrition and Nutritional


Disorders

1. Acute hyponatremia in 8-month-old infant may produce all, except:


(a) Diarrhoea
(b) Convulsions
(c) Muscle cramps and weakness

(OPG5th/87) (JIPMER 79)


(d) Vascular collapse

Energy requirement
Age
0-2 mon
116 kcal/kg
2-3 mon
109 kcal/kg
3-4 mon
103 kcal/kg
4-5 mon
99 kcal/kg
5-9 mon
95 kcal/kg
9-12 mon
112 (470 kJ/kg) kcal/kg
1-3 yr
1200 kcal/day
4-6 yr
1700 kcal/day
7-9 yr
1900 kcal/day
10-12 yr
2100 kcal/day
13-15 yr
2400 kcal/day
16- 18 yr
2600 kcal/day
2. The normal calorie requirement for a 5-year old child is:
(OPG5th/92) (PGI 93)
(a) 800 calories
(b) 1000 calories
(c) 1500 calories
(d) 2000 calories
3. The average calorie requirement for infants during the first year of life is:
(OPG5th/92) (PGI 81)
(a) 80 to 90 kcal/kg
(b) 100 to 120 kcal/kg
(c) 120 to 130 kcal/kg
(d) Any of the above depending upon
weight
4. The amount of calories required at 1 year of age are:
(OPG5th/92) (AI 96)
(a) 900 kcal/day
(b) 1000 kcal/day
(c) 1200 kcal/day
(d) 1400 kcal/day
5. Protein efficiency ratio is:
(OPG5th/93) (AI 94)
(a) Gain in weight of young animals per unit of protein consumed
(b) Product of digestibility coefficient and
biological value divided by 100 (c) Percentage of nitorgen retained out of the nitrogen absorbed (d) Percentage
of protein absorbed after digestion
Protein quality
A. Protein efficiency ratio (PER)
B. Bilogical value =

1 A

2 C

3 B

Weight gain (g)


Protein consumed (g)
Retained nitrogen
Absorbed nitrogen

4 C

5 A

100

12 / Pediatric Buster
C. True digestibility =

Absorbed nitrogen 100


Food nitrogen
D. Net protein utilization =
Retained nitrogen 100
(product of biological value and Food nitrogen
digestibility represented as)
6. In the case of a normal full-terms baby, breastfeeding should start
(OPG5th/97) (UPSC/01)
(a) Soon after birth and continue for 4 to 6 months
(b) After 3 days of birth and continue for 9 months
(c) After 48 hours of birth and continue for 6 months
(d) After and days of birth and continue for 3 months
7. Breastfeeding should begin..... hours after delivery.
(OPG5th/97) (KARN/87)
(a) 2
(b) 4
(c) 8
(d) 24
Breastfeeding should be initiated as soon as possible after birth; i.e., within half an hour after normal
delivery and four hours after cesarean section.
Breastfeeding offers lower risk of infection because of presence of secretory lgA, lysozyme,
lactoferrin for enterobacter, low pH, bifidous factor (for E. coli) and viable phagocytic macrophages.
Tetany of the newborn due to functional hypoparathyroidism is not seen in the breastfed infant.
Milk is a poor source of iron; additional supplements are required after 6 mon 0.6 mg/kg/day of
elemental iron.
All artificially fed infants should receive vit C supplement or fresh juice of citrus family.
Breast milk protects against necrotizing enterocolitis, deficiency of vit E. and zinc and celiac
disease
Breast milk offers PABA for protection against malaria, child has less spitting up, eczema and colic.
There is decreased incidence of otitis media and high antipolio titres.
Vitamin K deficiency in breast milk can cause hemorrhagic disease of newborn.
Stools of baby of low pH, golden colour and acidic in reaction.
CMV antibodies present in the breast milk are not protective to child.
Erythroblastosis foetalis is not a contraindication for breastfeeding.
8. Human colostrum contains more of the following nutrients than mature human milk except:
(a) Lactose
(b) Minerals
(c) Proteins
(d) Vitamin A

(KARN 94)

COLOSTRUM
The secretion started in mid pregnancy and for the first two days following delivery is called
colostrum.
- It is deep yellow serous fluid with alkaline reaction
- 10-40 ml is daily secretion
- It contains high protein, vitamin A, sodium and chloride and high specific gravity
- The carbohydrate, fat and potassium are less than breast milk
- It contains antibodies (lgA, lgG, lgM) and humoral factors
- It contains fat globules which gives laxative effect to baby
- Assumes the characteristics of mature breast milk by 3-4 weeks
Selective contraindication for breastmilk are galactosemia and phenylketonuria
9. Human colostrum has a relatively higher concentration of all of the following except: (OPG5th/97) (UPSC/99)
(a) Protein
(b) Fat
(c) IgA
(d) Sodium
10. Breastfeeding is contraindicated if the mother is receiving:
(OPG5th/98) (AIIMS 83)
(a) Streptomycin
(b) Insulin
(c) Antithyroid drugs
(d) Broad spectrum antibiotics
11. Which of the following is the reference standard on which classification of protein energy malnutrition is based:
(OPG5th/102) (AIIMS 83)
(a) Gomez classification
(b) 10th percentile of weight for age of ICMR
(c) 80% of the 50th percentile of
weight of ICMR
(d) 80% of the 50th percentile of weight for age of American standard
6 A

7 A

8 A

9 B

10 C

11 C

Nutrition and Nutritional Disorders / 13


INDIAN ACADEMY OF PEDIATRICS CLASSIFICATION
80% of reference WEIGHT for age is within Normal range
1 degree malnutrition
- B/W 77-80%
IInd degree malnutrition
- B/W 61-70%
IIIrd degree malnutrition
- B/W 51-60%
IVth degree malnutrition
- < 50%
12. Which one of the following anthropometric measurements can be used to assess nutritional status of a child if
mother fails to recall exact chronological age?
(OPG5th/102) (UPSC/03)
(a) Weight for age
(b) Height for weight
(c) Height for age
(d) Head circumference
Anthropometric measurements to assess nutritional status
A. Age Dependent Criteria
(1) Weight for age
(2) Height for Age
B. Age Independent Criteria
(1) Weight for height
(2) Quack stick
(3) Midarm circumference/head circumference ratio
(4) Midarm/height ratio
(5) Chest/head circumference ratio
(6) Skin fold thickness
(7) Mid thigh and calf circumference
13. The earliest indicator of PEM is:
(OPG5th/104) (PAR/416)
(a) Hair changes
(b) Oedema
(c) Underweight for age
(d) Restlessness
14. Kwashiorkor is due to:
(OPG5th/105) (KERALA 94)
(a) Protein deficiency (b) Protein and energy deficiency (c) Energy deficiency (d) Malabsorption of vitamins
15. Flag sign is seen in:
(OPG5th/105) (PAR/416) (UPSC 85, JIPMER 87)
(a) Kwashiorkor
(b) Marasmus
(c) Pellagra
(d) Hypothyroidism
16. Kwashiorkor is characterised by all except:
(OPG5th/105) (UPSC 80)
(a) Dermatitis
(b) Oedema
(c) Flag sign
(d) Alertness
17. Features of kwashiorkor are A/E:
(OPG5th/105) (AI 99)
(a) Oedema (b) Patchy depigmentation of skin (c) Fatty infiltration of lives (d) Fatty infiltration of pancreas
18. Kwashiorkar in Ghanian Ga language means:
(OPG5th/105) ( Kerala 2K)
(a) Condition seen in 2-child
(b) Condition seen in the displaced child
(c) Condition seen in a cousin
(d) Condition seen in the stepchild
(e) Condition seen in the 3-child
Kwashiorkar and marasmus

A. Usual age
B. Essential features
1. Oedema
2. Wasting

13 C

14 B

15 A

Kwashiorkor

0-3 yr

1-3 yr

None

Lower legs,
sometimes face or generalized
Sometimes hidden, sometimes fat
blubbery

Gross loss of
subcutaneous fat
All skin and bone
Obvious
Obvious
Usually apathetic quiet

3. Muscle wasting
4. Growth retardation
5. Mental changes

12 C

Marasmus

16 D

17 D

18 B

Sometimes hidden
Sometimes hidden
Usually irritable moaning, also
apathetic

14 / Pediatric Buster
C. Variable features
1. Appetite
2. Diarrhea
3. Skin changes

Usually good
Often (past or present)
Seldom

4. Hair changes

Seldom

D. Biochemistry/pathology
1. Serum albumin
2. Urinary urea per g
creatinine
3. Urinary hydroxyproline
per g creatinine
4. Serum essential
amino acid index
5. Anemia
6. Liver biopsy

Usually poor
Often (past or present)
Often-diffuse depigmentation
occasional flaky paint or enamel
dermatosis
Often sparse, straight silky,
dyspigmentation, grey or reddish

Usually normal (or low)


Usually normal (or low)

Low
Low

Low

Low

Low

Low

Uncommon
Normal or atrophic

Common
Fatty changes

19. Consider the following:


(OPG5th/105) (UPSC/04)
(1) Growth retardation (2) Skin changes (3) Mental apathy (4) Hair changes
Which of these is/are the constant clinical feature(s) of kwashiorkor?
(a) 1 only
(b) 1 and 3
(c) 2, 3 and 4
(d) 1, 2, 3 and 4
20. All of the following are characteristic features of kwashiorkor, except:
(OPG5th/105) (AIIMS/03)
(a) High blood osmolarity
(b) Hypoalbuminemia
(c) Edema
(d) Fatty liver
21. For kwashiorkor essential criterion is:
(OPG5th/105) (AIIMS 92)
(a) Growth retardation
(b) Hepatomegaly
(c) Flag sign
(d) Skin changes
ESSENTIAL CRITERIA FOR KWASHIORKOR
A. Essential Features
(1) Markedly retarded growth, psychomotor changes
(2) Edema: Edema may be caused by (i) hypoalbuminemia; (ii) retention of fluid and water due to
increased capillary permeability as a result of infection; (iii) free radical induced damage to cell
membranes.
Mental changes: Lethargic, listless and apathetic
B. Other Usual Clinical Features
(1) Hepatomegaly
(2) Hair changes: The hair is thin, dry, brittle, easily pluckable, sparse, and devoid of their normal
sheen.
(3) Skin changes: Flaky paint dermatosis.
(4) Infections: Recurrent episodes of diarrhea, respiratory and skin infections.
22. One-year-old baby exclusively breastfed has a 1 cm hepatomegaly, severe pallor and no spleen. The most
important investigation is:
(AIIMS 89)
(a) B12 estimation
(b) Serum iron estimation
(c) Folic acid estimation
(d) Fetal haemoglobin estimation
23. Kwashiorkor is diagnosed in growth retarded children along with:
(OPG5th/105) (AI 91)
(a) Oedema and mental retardation
(b) Hypopigmentation and anaemia
(c) Oedema and hypopigmentation
(d) Hepatomegaly and anemia

19 C

20 A

21 A

22 B

23 A

Nutrition and Nutritional Disorders / 15


24. Periportal fatty liver is seen in:
(OPG5th/107) (AI 96)
(a) Protein-energy malnutrition
(b) Viral hepatitis
(c) Alcoholic liver disease
(d) Tuberculosis
25. Poor prognostic factors in a child with kwashiorkor include all of the following except:
(a) Hypothermia
(b) Hepatomegaly
(c) Purpura
(d) Dermatosis
(OPG5th/107) (KARNAT 96)
Crireria for admission in severe PEM
Children weighing less than 60 percent for age with
1. Edema
2. Severe dehydration
3. Severe diarrhoea
4. Hypothermia
5. Shock
6. Systemic infection
7. Jaundice
8. Bleeding
9. Age less than one year
10. Persistent loss of apetite
Poor prognostic indicator
1. Serum albumin < 1g/dL
2. Total bilirubin > 6 mg/dL
3. Serum sodium < 120 mmol/L
26. Mostly death in PEM is due to all except:
(a) Hypothermia
(b) CCF
(c) Worm infestation

(OPG5th/107) (TN 89)


(d) Electrolyte imbalance

COMPLICATION OF PEM
S Sugar deficiency i.e., hypoglycemia
H Hypothermia
I Infection and septic shock
El Electrolyte imbalance
DE DEhydration
D Deficiencies of iron, vitamins, and other micronutrients.
27. Moderate to severe malnutrition is indicated by:
(a) Albumin less than 2.1
(b) Body mass index less than 19
(d) Hair changes

(OPG5th/109) (PGI/99)
(c) Serum transferrin less than 160

28. A 10-month-old child weighing 8 kg has Bitots spots in both eyes. Which of the following the most appropriate
schedule to prescribe vitamin A to this child?
(OPG5th/121)(AI/04)
(a) 2 lakh units intramuscular (IM) on day 0,14.
(b) 1 lakh units IM on day 0,14.
(c) 2 lakh units IM on
day 0,1 and 14.
(d) 1 lakh units IM on day 0, 1 and 14.
DOSES OF VITAMIN A
Prevention of vitamin A deficiency: Infants who are not breastfed should receive a 50,000 IU
supplement of vitamin A by 2 months of age (or two doses of 25,000 IU each with 1 month interval
in between) in areas of endemic vitamin A deficiency. Every infant should be administered one dose
of 1 lac units of vitamin A along with measles vaccine at 9 months followed by four more doses of 2
lakh IU each at 18, 24, 30 and 36 months.

24 A

25 B

26 C

27 A

28 D

16 / Pediatric Buster
WHO/UNICEF treatment schedule of xerophthalmia
Children 1 to 6 years and above
Immediately on diagnosis
200,000 IU vitamin A (O)
The following day
200,000 IU vitamin A (O)
Fours weeks later
200,000 IU vitamin A (O)
Children under 1 year and under 8 kg weight at any age
Half the doses as indicated for children 1 to 6 years and above
For night blindness or Bitots Spot
Treat with a daily dose of
10,000 IU vitamin A (O) for 2 weeks
29. All of the following statements are true regarding vitamin A deficiency except:
(NEL17th/180) (KARNAT 96)
(a) Growth retardation is common
(b) Frequent infections can occur
(c) Hydrocephalus is infrequent
(d) Posterior segment of the eye is initially involved
C/F of vitamin A deficiency:
Initially involves posterior segment of eye.
Defective dark adaptation (Night blindness)
XeropthalmiaBitots spot on the temporal side of corneoseleral junction (reversibel)
Keratomalaica
Skin becomes scaly and toad like (hyperkeratosis).
Toad skin is now believed to be due to deficiency of essential fatty acid.
Squamous metaplasia of respiratory mucosa
formation of renal and vesical stones.
Interference with reproduction.
Raised intracranial pressureseparation of cranial bones at sutures may occur (hydrocephalus).
Hypervitaminosis A: Headache; vomiting; raised ICT anorexia; dry skin; painful extremities sparse
hair; hepatosplenomegaly, hypoplastic anemia and benign inracranial hypertension (pseudotumour
cerebri).
30. The most common cause of anemia during childhood is:
(OPG5th/122) (AIIMS 83)
(b) Folic acid deficiency
(c) Pyridoxine deficiency
(d) Iron deficiency
(a) Vitamin B12 deficiency
31. One of the following is not reported to be a clinical manifestation of zinc deficiency state in children:
(a) Dwarfism and hypogonadism
(b) Lever and spleen enlargement
(c) Impaired cell mediated immunity
(d) Macrocytic anemia
(OPG5th/123) (KARN 94)
32. Growth retardation, taste alteration, hepatosplenomegaly, hypochromic microcytic anemia, loss of hair is seen
in defeciency of:
(OPG5th/123) (KARN 95)
(a) Selenium
(b) Copper
(c) Zinc
(d) Iron
Zinc deficiency: Taste alteration and loss of hair; acrodermatitis enteropathica; Night blindness, A
syndrome of growth failure, anaemia and hepatomegaly, and hypogonadism.
33. Perioral rash, pigmentation of skin creases and neuritis is seen in deficiency of:
(OPG5th/123) (AIIMS 89)
(a) Zinc
(b) Copper
(c) Magnesium
(d) Biotin
34. A child with alopecia, psoriatic dermatitis in genitals and mouth and hypogonadism suffering for:
(a) Cu deficiency
(b) Iron deficiency
(c) Zn deficiency
(d) Mg deficiency
(OPG5th/123) (AI 98)
35. Riboflavin deficiency is characterised by:
(OPG5th/125) (PGI 89)
(a) Angular stomatitis
(b) Cheiolosis
(c) Glossitis
(d) All
Riboflavine (B2): Involved in oxidation-reduction mechanism in cellular respiration
C/F of deficiency - Glossitis
- Cheilosis
- Scaly dermatitis
- Circumcorneal vascularization and dermatitis, erythroid hypoplasia.
29 C

30 D

31 D

32 C

33 A

34 C

35 D

Nutrition and Nutritional Disorders / 17


36. Dermatitis may be a clinical manifestation in the deficiency states all of following nutrients except:
(a) Biotin
(b) Niacin
(c) Pyridoxin
(d) Thiamine
(OPG5th/125)
5th
37. Thiamine deficiency causing:
(OPG /125) (UP 97)
(a) Beriberi disease
(b) Angular stomatitis
(c) Cheiolosis
(d) Glossitis
Thiamin (B1):
Three major function of TPP (Thiamin pyrophosphate) are:
1. Oxidative decarboxylation of alpha ketoacids leading to the synthesis of ATP.
2. As a cofactor for transketolase in the pentosephosphate pathway.
3. Maintains neural membane and normal conduction chiefly by peripheral nerves. In its deficiency the
utilization of pyruvic acid is decreased. Pyruvic and lactic acid accumulate in the tissues and their
blood levels are increased. C/F: Beriberi due to polished rice.
Dry Beriberi(Nervous system affected): Irritability, emotional disturbance, polyneuritis, tender
calfmusles and sluggish tendon reflexes. There is polyneuropathy manifested first by numbers and
tingling of the feet and lower legs. Sensory loss also accompanies.
Wet Beriberi(Cardiovascular involvement)- Palpitation, tachycardia, dyspnea and edema. There
may be arteriovenous shunting and high output failure. When the deficiency state occurs particularly
acutely, a fulminant form of heart failure may arise; referred to as Shoshin beriberi.
Infantile BeriberiSeen in breastfed infants of thiamine deficient mothers. Manifested in three ways
1. CVS typeacute onset with rapid course; infant has a shrill cry: Tachycardia; dyspnea; cyanosis
and vomiting.
2. Aphonic typeDyspnea; puffiness and pitting edema.
3. CNS typevomting; tremors and convulsions; ptosis, nystagmus and extraocular muscular
paralysis.
38. Thiamine deficiency can result in all of the clinical condition except:
(OPG5th/125) (UPSC/01)
(a) Peripheral neuropathy
(b) Wernickes encephalopathy
(c) Hepatic failure
(d) Congestive heart failure
39. Signs of pellagra include all of the following except:
(OPG5th/125) (AI 88)
(a) Sore red and swollen tongue
(b) Diarrhoea
(c) Desquamated pigmented dermatitis affecting exposed
areas of skin
(d) Angular stomatitis
40. In niacin deficiency seen in A/E:
(OPG5th/125) (AI 96)
(a) Deafness
(b) Diarrhoea
(c) Dementia
(d) Dermatitis
Niacin (PP factor): Essential component of the coenzyme NAD. Trytophan, an essential amino acid
can be converted to niacin.Two uncommon disorders affecting trytophan metabolism may be
complicated by pellagra.
1. Carcinoid syndrome
2. Hartnups disease
Deficiency is more common in maize eating population
C/F Diarrhoa (3Ds)
Dermatitis-Casals Necklase or crazy pavement.
Dementia
Tongue is red; swollen and beffy
Two most dominant complaints are persistent fatigability and weakness. Uncommon featuresAbdominal pain; dysphagia; proctitis and vaginitis
Long-term use of nicotinic lower LDL and VDL and HDL moreover long-term use may be associated
with hepatic injury activation of peptic ulcer and induction of diabetes.

36 D

37 A

38 C

39 D

40 A

18 / Pediatric Buster
41. A child who bleeds from gums and has swollen knee probably due to:
(a) Haemophilia
(b) ITP
(c) Scurvy
(d) Trauma

(OPG5th/127) (AMU 95)

Vitamin C (Ascorbic acid) : used in oxidation of tyrosine and phenylalanine.


Conversion of proline into hydroxyproline.
Transport of iron.
Enhance action of folic acid.
C/F of deficiency:
Associated folate deficiency.
Child is listless; anorexic and cries (there is icterus, edema, fever, and convulsions).
Breastfed infants are protected as the breast milk contains adequate amounts of vit C.
Bones are tender and picture resembles Sjgrens syndrome.
Hemorrhages occur at the periosteum of long bones; gums; skin and mucous memberane.
Subperiosteal haematomas are present.
The primary defect in the skeletal system is in the formation of osteoid matrix not in mineralization
or calcification.
Gum bleeds are also present.
Costochondral junction become prominent and appear sharp and angular; scorbutic rosary is
separation of epiphyses of ribs and backward displacement of sternum.
A distinctive perifollicular hyperkeratotic; papular rash often appears.
Delayed wound healing and delayed suture closure.
Anemia is common, normocytic normochromic due to bleeding.
42. Early sign of rickets at six months is:
(OPG5th/128) (PGI 87)
(a) Craniotabes
(b) Double malleoli
(c) Bent tibia
(d) Knock knees
43. Which is true of nutritional rickets:
(OPG5th/128) (JIPMER 91)
(a) Decreased production of osteoid
(b) Decreased cortical bone
(c) Decreased calcification of growing bone
(d) Decreased calcification of mature bone
44. All of the following are seen in rickets, except:
(OPG5th/128) (AIIMS/03)
(a) Bow legs
(b) Gunstock deformity
(c) Pot belly
(d) Craniotabes
45. The following are radiographic features of rickets except:
(OPG5th/128) (KARNAT 96)
(a) Increase in width of growth plate
(b) Decreased bone density
(c) Subperiosteal bleeding
(d) Rickety
rosary
46. Vitamin D resistant rickets due to all except:
(OPG5th/129) (AIIMS 92)
(a) 1 hydroxylase deficiency
(b) Renal tubular acidosis
(c) Fanconi syndrome
(d) Drugs
Vit D:
1. Stimulates intestinal absorption of calcium and independently of phosphorus.
2. Collaborates with parathyroid hormone in the mobilization of calcium from bone.
3. Stimulates the paratheyroid hormone-dependent reabsorption of calcium in the distal renal
tubules.
Deficiency results in:
Biochemical changes

Serum Ca and P-34 - falls


Elevation of alkaline
Phosphatase level.

Aminoaciduria
Rickets (Inadequate mineralization and hence excess of unmineralized matrix) The changes are
complicated by inadequate provisional calcification of epiphyseal cartilage deranging enchondral
bone growth.

41 C

42 A

43 C

44 B

45 C

46 D

Nutrition and Nutritional Disorders/ 19


C/F of Vit D deficiency Rickets: Usually manifest in latter half of 1st yr. or IInd It is unusual below
the age of three months.
Craniotabes is the earliest manifestation of rickets.
Frontal bossing and sqaured appearance of head.
Costochondral junction becomes prominent to form ricketic rossary
Pigeon breast
Harrisons groove
Lumbar lordosis and bowing of legs.
Delayed eruption of teeth.
Epiphyses of long bones widened; broadening of wrist and double malleoli.
Pot-belly
Bilateral lamellar cataracts in early infancy.
MANAGEMENT
6 lakh IU orally or IM of vit D.
Evidence of healing is a thin dense line initally near the epiphysis, where preparatory calcification
would be occurring.
Repeat dose if no response in 3-4 wk. If no response is there even to second dose of vit D; diagnosis
of refractory rickets should be entertained.
Vit D resistance rickets is treated by large dose of vit D (200 units) along with 0.5 to 1.5 g of
phosphates and phosphoric acid.
47. Vitamin D resistant rickets has all the following characteristics except:
(OPG5th/129) (UPSC/01)
(a) Hypophosphatemia
(b) X-linked inheritance
(c) Reduced calcium absorption from gut
(d) Renal stones
48. Vitamin D deficiency rickets is confirmed by demonstration of:
(OPG5th/130) (AIIMS 83)
(a) Epiphyseal changes in X-ray
(b) Hypocalcaemia and hypophosphataemia
(c) Raised serum alkaline
phosphatase
(d) Healing with physiologic doses of vitamin D
49. Cause of recent onset genu valgum and metaphyseal widening in a child with serum calcium 11.0 mEq/L serum
parathormone increased and phosphate 2.8 mEq/L is:
(OPG5th/130) (AIIMS/2K)
(a) Nutritional rickets
(b) Azotemic osteodystrophy
(c) Hypophosphatemic rickets
(d) Primary hyperparathyroidism
50. A 2-year-old boy has vitamin D refractory rickets. Investigations show serum calcium 9 mg/dl, phosphate 2.4 mg
dl, alkaline phosphatase 1040 IU. Parathyroid hormone and bicarbonate levels are normal. The most probable
diagnosis is:
(OPG5th/130) (AIIMS/02)
(a) Distal renal tubular acidosis
(b) Hypophosphatemic rickets
(c) Vitamin D dependent rickets
(d) Proximal renal tubular acidosis.
VARIANTS OF RICKETS
Features

Type I
(Calcium deficient)

(1)
(2)
(3)
(4)
(5)

Normal or low
Normal
Usually low
Low
Elevated
Elevated
Present
Absent
(a) Deficiency of vit D
(a) Primary
(b) Liver disease
hypophosphatemia
(c) Anticonvulsant therapy
(d) Deficiency of phosphate

Serum calcium
Serum phosphorus
Alkaline phosphatase
Secondary
Cause

Type II
(Phosphate deficient)

51. Calcium requirement of a growing school age child is about:


(a) 10 gm/day
(b) 0.1 gm/day
(c) 1.0 gm/day
(d) 2.0 gm/day

47 C

48 D

49 C

50 B

51 C

Type III
(End organ resistance)
Low
Normal
Elevated

(a) Vit D dependent


rickets type II

(OPG5th/131) (JIPMER 79, UPSC 86)

20 / Pediatric Buster
Disorders of trace elements
Trace element

Function

Deficiency

Excess

(1) Copper (E)

Transferrin
Hemoglobin
Absorption of iron
Oxidative enzyme
activation-tyrosinase,
catalase, uricase,
cytochrome oxidase,
lysyl oxidase, etc

Indian childhood
cirrhosis
Wilsons disease
Hemolytic anemia
(Fanconi-like anemia)
Acute copper poisioningvomiting, hemolytic anemia
and gastric hemorrage

(2) Fluoride
(E)

Constituent of bone
and tooth

Refractory anemia
Osteoporosis
Neutropenia
Delayed bone age
Pseudoparalysis
Hypercholesterolemia
Menkes disease
Defective hair
pigmentation
Hypothermia
Degenerative changes
inelastin of aorta
Mental deterioration
Retarded growth
Dental caries
Osteoporosis

(3) Iodide
(E)

Part of T3 and T4

Endemic cretinism
Simple goiter

Fluorosis
Vomiting, diarrhea,
Abdominal pain
Tetany
Cardiovascular collapse,
Goiter

Tetany

Anemia, abdominal
colic, muscular weakness
low IQ low attention
encephalopathy
Unknown

Keshan
cardiomyopathy
Kaschin Beck disease
Decreased fertility
Cancer of esophagus

Alopecia,
nail deformity
lassitude,
garlic odor to breath
Hyperuricemia

(4) Lead (NE)

(5) Magnesium
(E)
(6) Selenium
(SE)
(7) Molybdenum
(E)
(8) Mercury
(9) Nickel
(E)-

(10) Manganese (E)

EEssential
SEsemiessential

Constituent of bone
Enzyme activator
Muscle and nerve
irritability cation
Cofactor of
glutathione peroxidase
Constituent of
xanthine oxidases,
Toxic for many enzymes
and RNA
Stabilization of DNA
and RNA
Urease-matalloprotein
contains nickel as
essential constituent
Constituent of
mitochondrial
superoxide dismutase
Structure of bone
Antioxidant

Not known

Increased
prothrombin
and bleeding time

Minimatta disease
Acrodynia
Dermatitis,
nasal and pulmonary
carcinoma,
liver necrosis
Encephalopathy
pneumoconiosis,
syndrome like
Parkinsons disease

Newborn Infants

In India, the average weight of a normal newborn after 40 wk of gestation is around 2.8 kg.
Low Birth weight baby is of 2500 gm or less irrespective of the gestational age or less than 10th
percentile of the weight.
Very low birth weight baby is weighing 1500 gm or less irrespective of gestational age.

Algorithm regarding the brief outline of steps of action as recommended by NRP for the neonatal resuscitation

22 / Pediatric Buster
Certain vital donts In neonatal resuscitation
Dont administer heavy sedation to the mother
Dont do heavy and continuous suction.
Dont let the neonate develop hypothermia.
Dont carry on with tactile stimulation beyond 2 and never beyond 4 flicks.
Dont delay endotracheal intubation in an apneic neonate.
Dont below your lungs into neonates mouth.
Dont use full palmar grasp for giving bag and mask ventilation.
Dont give respiratory stimulants.
Dont suck the nose first and the mouth later. The breathing effort that follows sucking the nose
first may allow secretions in the mouth to be suddenly aspirated into the lower airway
Dont slap the back
Dont squeeze the rib cage
Dont force thighs on the abdomen
Dont dilate the anal sphincters.
1. The primary aim of neonatal resuscitation:
(OPG5th/137) (UP 94)
(a) To maintain breathing
(b) To restore heart rate
(c) To retain colour
(d) To induce spontaneous cry
2. Neonatal period extends up to:
(OPG5th/137) (JIPMER 81)
(a) 21 days of life
(b) 30 days of life
(c) 28 days of life
(d) 35 days of life
3. You are part of a neonatal resuscitation team attending a delivery. A baby weighing 3 kg is born and has not
cried immediately. What sequence of resuscitation steps will you carry out for this baby?
(OPG5th/139)(UPSC/04)
(a) Dry the baby; position the baby; tactile stimulation; suction of mouth and nose; free flow oxygen if required
(b) Dry the baby; free flow oxygen; position the baby; suction of mouth and nose
(c) Position the baby; suction
of mouth and nose; free flow oxygen; tactile stmulation; dry the baby (d) Dry the baby; position the baby; suction
of mouth and nose; tactile stimulation; free flow oxygen if required.
4. Meconium aspiration is done for 3 times but no breathing occurs next step is:
(OPG5th/140) (AI 97)
(a) Chest compression
(b) O2 inhalation
(c) Bag and mask incubation
(d) Trickling
5. A mother is preparing to breastfeed her infant, it would be of value of her, if she were acquainted with:
(a) Moro reflex
(b) Rooting reflex
(c) Grasp reflex
(d) Cough reflex
(OPG5th/145) (AIIMS 830)
Neonatal reflexes: The neonatal reflexes can be best elicited during quite awake state.
Sucking and rooting reflexstrong and well-cordinated at 32 wk
Grasp reflexusually disappears by 12 wk. Persistence beyond 12 wk is indication of brain
damage.
Moros reflexIt is a vestibular reflex elicited after 28 wk of gestation even in premature baby
but the abduction is weak. The reflex is complete after 32 wk of gestation. Infants with cerebral
damage have exaggerated or absent response. An asymmetric response is seen in Erbs palsy; spastic
hemiplegia and fracture of the humerus or clavicle. Response is poor in Down syndrome and
hypothyroidism. Absent in kernicterus and depressed in opioid withdrawal syndrome. The reflex
disappears after the age of 12 wk so persistence is considered pathological.
Glabellar tap reflex.
Cross extensionappears at 32 wk but at 32 wk, there is extension but no adduction.
Automatic walking32 wk.
Tonic neck reflex(Fencing posture)Prominent during 2-4 mon persistence post 6-9 mon.
Persistence past 6-9 mon indicates poor control of motor function or cerebral pasly.
Landau reflex
Parachute reflex
Babinskis sign and Chvosteks sign may be present normally in term neonate.

1 A

2 C

3 D

4 D

5 B

Newborn Infants / 23
PRIMITIVE REFLEXES
Reflex

Age of appearance

Age of disappearance

Stepping
Placing
Moro
Sucking and
rooting
Palmar grasp
Plantar grasp
Tonic neck
Landau
Neck righting
Parachute

Birth
Birth
Birth
Birth

6 weeks
6 weeks
3 months
4 months while awake.
7 months while asleep
6 months
10 months
4 to 6 months
24 months
24 months
Persists throughout life

Birth
Birth
2 months
3 months
4 months
9 months

6. The primitive reflex that persists throughout life is:


(a) Moro reflex
(b) Tonic neck reflex
(c) Palmar grasp reflex

(OPG5th/145)(UPSC/03)
(d) Parachute reflex

Features of normal full term newborn infant


50 cm long with head circumference 34-35 cm and chest circumference 3 cm less.
The upper segment to lower segment ratio is 1.7 and 1.9 to 1.
Cry is vigorous but if it is abnormal to indicate a neurological difficulty.
Normal newborn spends 80% of time in sleeping.
Breast nodule is palpable; Ear cartilage is firm with good elastic recoil.
At least one testis is descended.
The anterior 2/3rd or more of the soles show deep creases.
The heart rate ranges between 120-140/min and while crying may increase to 160/min.
Respiratory rate stabilizes at about 30-40/min.
Peripheral cyanosis may be present for a short time.
94% of newborn pass urine in 24 hr after birth.
The first stool; meconium, black thick and viscid is passed within 24 hr after birth.
Most babies regularize their feeding pattern by the end of the first wk. and demand feed after every
3-5 hr.
The baby loses about 7-8% of the birth wt during the first week.
The baby regains the birth wt by 10 days and then continues to gain weight at the rate of 20-30
g/day for the next 3 months of age.
7. Meconium is excreted by a newborn till...day:
(OPG5th/145) (TN 89)
(a) 2
(b) 3
(c) 6
(d) 4
8. 99% of the neonates void within:
(OPG5th/145) (PGI 79, AMU 88)
(a) 24 hr
(b) 48 hrs
(c) 8 hr
(d) 12 hr
9. At birth the normal heart rate is:
(OPG5th/145) (PGI 81)
(a) 60-80
(b) 80-110
(c) 70-120
(d) 110-150
10. Pulse rate of a neonate varies between:
(OPG5th/145)
(CUGEE 96)
(a) 120 to 140 minutes
(b) 140-160 minutes
(c) 70-90 minutes
(d) 160-180/minutes
11. Normal reflex patterns at birth may include:
(OPG5th/146) (JIPMER 81)
(a) Grasp reflex
(b) Moro reflex
(c) Sucking or rooting reflex
(d) All
12. Which one of the following is not normal in a neonate?
(OPG5th/146)(UPSC/03)
(a) Cutaneous hemangiomata
(b) Caput succedaneum
(c) Icterus neonatorum
(d) Cafe au lait spots
13. Umbilical cord usually falls after:
(OPG5th/146) (PGI 81, DNB 91)
(a) 5-10 days
(b) 15-20 days
(c) 1-5 days
(d) None of the above
6 D

7 A

8 B

9 D

10 A

11 D

12 D

13 A

24 / Pediatric Buster
14. Persistent Moros reflex at 12 weeks indicates:
(a) Normal child
(b) Brain damage
(c) Hungry
15. Moros reflex disappears by:
(a) 1 month
(b) 3 months
(c) 5 months
(d)
16. Which is not a normal finding in the newborn:
(a) Systolic murmur
(b) Pulse rate of 100/min
17. A hymenal tag in a newborn is best treated by:
(a) Steroids
(b) Surgery
(c) Leaving it alone
18. The following are normal features in a neonate:
(a) A vaginal bleeding
(b) Breast engorgement
19. Breast milk at room temperature stored for:
(a) 4 hr
(b) 8 hr
(c) 12 hr
(d) 24 hr

(OPG5th/146) (PGI 87)


child

(d) Irritable child


(OPG5th/146) (UP 94)

7 months
(OPG5th/147) (JIPMER 93)
(c) Peripheral cyanosis
(d) Central cyanosis
(OPG5th/147) (AIIMS 80 PGI 81)
(d) None
(OPG5th/147) (PGI 89)
(c) Palpable liver and kidney
(d) Phimosis
(e) All
(OPG5th/151) (PGI/98)

Expressed break milk can be stored at room temperature for 10 hours; in a refrigerator for 24 hours
and in a freezer at 20C for 3 month
The first 2 weeks of the neonatal period are crucial for establishing breastfeeding, lactogenic
hormones are ineffective in stimulating human breast secretion.

20. Low birth weight baby is defined as:


(OPG5th/155) (AIIMS 84)
(a) Birth weight less than the anticipated from period of gestation
(b) Gestation of less than 37 weeks
(c) Birth weight of less than the average for the country
(d) Birth weight of 2500 gm or less
21. A low birth weight baby is by definition:
(OPG5th/155)
(CUPGEE 96)
(a) Less than the 10th percentile of the expected weight (b) Less than the 15th percentile of the expected weight
(c) Less than the 5th percentile of the expected weight
(d) Less than the 20th of the expected percentile weight
PROBLEMS WITH LOW BIRTH WEIGHT BABIES
Preterm

Small for date babies

Birth asphyxia
Hypothermia
Feeding difficulties
Infections
Respiratory distress due to
hyaline membrane disease
Apneic spells
Intraventricular hemorrhage
Necrotising enterocolitis
Metabolic acidosis
Hyperbilirubinemia

Birth asphyxia
Meconium aspiration
syndrome
Hypothermia
Hypothermia
Infections
Polycythemia

1. Symmetric IUGEarlier onset


associated with fetal cell number; etiologies as chromosomal; genetic, malformation. Teratogenic
or severe hypertensives etiologies
These babies are proportionately smaller in all parameters including the head size.
2. Asymmetrical IUGRLate onset
Brain is not affected and difference in head and chest circumference is > 3 cmPreservation of
doppler wave form velocity to carotid vessels.
Associated with poor maternal nutrition or late onset or exacerbation of maternal vascular disease
(Preeclampsia and chronic Hypertension)
22. In neonatal septicaemia all of the following are used as diagnostic indicators except:
(OPG5th/161) (AI 96)
(a) Leucocytosis
(b) Positive CRP
(c) Immature neutrophils greater than 20% of total
(d) ESR raised

14 B

15 B

16 D

17 C

18 E

19 B

20 D

21 A

22 A

Newborn Infants / 25
23. All of following are true about neonatal sepsis except:
(OPG5th/161) (AI 96)
(a) Preterm babies are predisposed to sepsis
(b) Late initiation of breastfeeding is a predisposition
(c) Premature rupture of membranes predisposes to sepsis
(d) None
24. Laboratory findings in neonatal septicaemia are A/E:
(OPG5th/161) (AI 96)
(a) C-reactive protein
(b) Leucocytosis
(c) ESR increased
(d) Toxic granulated multilobulated nuclei
25. Most common cause of neonatal septicaemia:
(OPG5th/161) (AI 98)
(a) Local nusery environment
(b) Infection through umblical cord
(c) Exclusively breastfed baby
(d) Infection by GIT bacteria
Septicemia in the neonatal period:
Preterm and low birth weight infants are susceptible to infection via the umblical cord.
Predisposing factors.
- Poor hygiene of nursery personnel
- Premature rupture of membrane for 24 hr or more before labor
- Maternal infection fornight prior to delivery
- Repeated vaginal examination during labor.
Host risk factors- Male sex; galactosemia (E.coli); I/M iron, congenital anomalies, immune defects,
second twin and omphalitis.
C/F:
Baby becomes lethargic and unresponsive if sucking normally.
Apneic episodes may be only manifestation in preterm babies.
There may be hepatosplenomegaly; jaundice and abdominal distension.
Marked pallor; ashen grey color; cold extremities and absent pulse with fall in BP indicates shock
There is neutropenia; immature forms of neurophils exceed 20% of the total neutrophils; C reactive
protein is elevated and ESR and haptoglobin also raised.
Limulus amebocyte lysate detects endotoxins of gram-negative bacteria.
Bactericial drugs cefotaxime and ceftriaxone are the antibiotics of choice if organism is not known.
Therapy continues for 3-7 days beyond apparent cure for ordinary case; 3 wk of meningitis and
osteomyelitis and 6 wk for UTI.
26. Neonatal septicaemia is most commonly caused by:
(OPG5th/161) (AIIMS 92)
(a) E. coli
(b) Grp. B Streptococcus
(c) Staph. aureus
(d) Streptococcus viridans
27. Neonatal sepsis and meningitis most common cause:
(OPG5th/161) (PGI/98)
(a) Streptococcus pyogenes
(b) Streptococcus algaetaciae
(c) Enterococcus fecalis
(d) Staphylococcus aureus
DEFINITION OF SIRS AND SEPSIS
SIRS: The systemic inflammatory response to a variety of clinical insults, manifested by 2 or more of
the following conditions:
Temperature instability < 35 C or > 38.5C
Respiratory dysfunction
Tachypnea > 2 SD above the mean for age
Hypoxemia (PaO2 < 70 mm Hg on room air)
Cardiac dysfunction
Tachycardia > 2 SD above the mean for age
Delayed capillary refill > 3 sec
Hypotension > 2 SD below the mean for age
Perfusion abnormalities
Oliguria (urine output < 0.5 ml/kg/hr)
Lactic acidosis (elevated plasma lactate and/or arterial pH < 7.25)
Altered mental status
Sepsis: The systemic inflammatory response to an infectionus process
23 B

24 B

25 A

26 A

27 B

26 / Pediatric Buster
28. In neonatal necrotising enterocolitis all of the following are found except:
(OPG5th/164) (AI 96)
(a) Bradycardia
(b) Blood in stools
(c) Abdominal distention
(d) Bowel sounds increased
Neonatal necrotizing enterocolitis:
More prone to occur in premature infants.
There occurs apnea; bradycardia and cyanosis, hypotension, polycythemia and leucocytosis.
Bowel shows edema aned vascular engorgment. Gas collects in the submucosa and the intestinal
wall becomes necrotic.
Ileum is the most common site and clostridia is suspected to be the cause.
Illness develops in 1 or 2 days with mild abdominal distension; blood in stools, bowel sounds are
diminished, mild acidosis, and mild thrombocytopenia
DIC may occur in last stage and pneumoperitoneum is present on X-ray.
Parenteral alimentation is done with amino acid mixture and 25% glucose.
29. Which of the following infections has the most neurological complications:
(OPG5th/165t) (UPSC 86)
(a) Rubella
(b) Measles
(c) Smallpox
(d) Scarlet fever
30. Rubella maximally infects the foetus if the mother contracts the disease during....of pregnancy:
(a) 5-6 weeks
(b) 9-12 weeks
(c) 12-13 weeks
(d) 20 weeks
(OPG5th/165t) (AIIMS 87)
Congenital Rubella:
Rubella is also called 3 days measles
Risk of fetal infection is 50% or high if mother acquires infection during the first four weeks of
gestation.
But the greatest risk to foetus is when infection develops 3-6 wk post-conception
Most florid fetal illness is manifested in first 8-10 wk of gestation.
Foetus infected after 20 weeks may have isolated deafness as only symptom
31. The following are seen in congenital rubella except:
(OPG5th/165) (PGI 89)
(a) Cataract
(b) Pulmonary stenosis
(c) Salt and pepper retinitis
(d) Bullae and vesicles on skin
32. Heart disease common with rubella infection:
(OPG5th/165) (AI 96)
(a) PDA
(b) VSD
(c) ASD
(d) Essenmenger syndrome
33. Purpura is seen in:
(OPG5th/165) (AI 89)
(a) Miliary tuberculosis
(b) Niemann-Picks diesease
(c) Congenital rubella
(d) CMV infection
34. Rubella syndrome is characterised by A/E:
(OPG5th/165) (AIIMS/99)
(a) Microcephaly with growth retardation (b) Deafness (c) Blindness (cataract) (d) Renal tract anomalies
35. Defect most commonly occurring in congenital rubella is:
(OPG5th/165) (AIIMS/85)
(a) Cataract
(b) Deafness
(c) Microcephaly
(d) Blindness
Defects in congenital Rubella:
M.C. defect is cataract
Unilateral or bilateral sensorineural deafness
CVS malformations such as PDA, pulmonary artery branch stenosis, VSD
Interstitial pneumonia
Increased bone densities of metaphysis of long bones without periosteal reaction
Salt and pepper retinitis; nuclear cataract, glaucoma and microphthalmia
Polycystic kidneys
Esophageal; jejunal atresia and hepatitis
Microcephaly and mental retardation
IUGR.
(OPG5th/165) (AIIMS 91)

36. Congenital rubella does not cause which cardiac anomaly:


(a) ASD
(b) PDA
(c) VSD
(d) PS
28 D

29 A

30 A

31 D

32 A

33 C

34 D

35 B

36 A

Newborn Infants / 27
37. Rubella infection in child is best diagnosed by:
(OPG5th/165) (AIIMS 95)
(a) IgM Ab in fetal blood
(b) IgA Ab in fetal blood
(c) T4 cell count in fetal blood
(d) Fetal haemoglobin
38. Cataract in newborn is seen in:
(OPG5th/165) (UPSC/2001)
(a) Congenital rubella
(b) Human papilloma virus infection
(c) Herpes simplex virus infection
(d) Toxoplasmosis
Congenital Rubella
Late complication

Diabetes
Thyroid disorders
Rubella encephalitis
Psychomotor defects
Dental abnormalities (Enamel hypoplasia)
Active Infection at birth
Hemolytic anemia
Thrombocytopenic purpura
Hepatitis
Encephalitis
Petechial blue bery muffin lesion
Or maculopapular skin lesion
Myocarditis
Pneumonitis
CSF protein raised without pleocytosis
Baby may shed virus upto 18 months
Prevention can be done by immunization of all boys and girls before puberty
39. An 8-year-old female child following URTI developed maculopapular rash on the face spreading onto the trunk
which cleared on the 3rd day without desquamation and tender post auricular and suboccpital lymphadenopathy.
The diagnosis is:
(OPG/ 5th/165) (AIIMS /MAY/01)
(a) Measles
(b) Rubella
(c) Erythema infectiosum
(d) Kawasaki disease (Mucocutaneous LN Syndrome)
40. A child born with microcephaly, chorioretinitis and intracranial calcification. Most likely diagnosis is:
(OPG5th/165t) (JIPMER 90)
(a) Congenital syphilis
(b) Rubella
(c) Toxoplasmosis
(d) Trypanosomiasis
Toxoplasmosis: Infection is transmitted from mother to foetus duing pregnancy in only 30% cases.
Women seropositive before pregnancy usually are protected against acute infection and do not give
birth to congenitally infected babies.
Chances of infection transmitted transplacental is less when acquired 6 months before conception
and increases as interval b/w infection and conception decreases.
Incidence of transplacental infection is lowest during first trimester (15%) but the disease in the
neonate is most severe. While incidence during third trimester is greatest (65%) but the infant is
usually asymptomatic at birth.
Infected infants who are asymptomatic at birth; develop learning disabilites and chronic neurologic
sequelae later in life.
C/F:
Cerebro-occular lesions are the most common; fever; refusal to feed; maculopapular rash;
hepatosplenomgaly and jaundice; meningoencephalitis chorioretinitis, convulsions; hydrocephalus,
microcephay and cerebral calcification.
Thrombocytopenia and IUGR
CSF shows xanthochromia
Tt: Pyrimethamine and sulfadiazine to child Spiramycin has been used for treating maternal
infection to prevent fetal Toxoplasma infection.

37 A

38 A

39 B

40 C

28 / Pediatric Buster
41. Which is not a manifestation of congenital toxoplasmosis:
(OPG5th/165t) (AIIMS 95)
(a) Hydrocephalus
(b) Chorioretinitis
(c) Cerebellar atrophy
(d) Intracranial calcification
42. Most common lesion in fetal toxoplasmosis:
(OPG5th/165t) (AIIMS 95)
(a) Chorioretinitis
(b) Pulmonary HT
(c) Encephalitis
(d) GIT involvement
43. STD which causes foetal abnormality is:
(OPG5th/165t) (AI 94)
(a) Herpes simplex
(b) Hepatitis
(c) Group B Streptococcus
(d) Toxoplasmosis
Herpes Simplex:
Neonatal HSV is almost always acquired during vaginal delivery or by ascending infection by early
rupture of membranes.
Lesion appears on 4-10th day after birth vesicular skin lesions may appear as inital sign.
Infant may have fever, hypothermia; dyspnoea and myocarditis, liver is enlarged; convulsions are
also seen.
CSF shows moderate pleocytosis and disproportionate elevation of proteins.
Breastfeeding is safe.
Vidarabine and acyclovir may help.
Cesarean section is indicated in genital herpes.
44. 6 weeks child presents with B/L cataract with seizure the probable diagnosis is:
(OPG5th/1 65) (AIIMS 96)
(a) Congenital toxoplasmosis
(b) Galactosaemia
(c) Fructosaemia
(d) Gauchers disease
Congenital Syphilis:
Rarely the infection may be contracted
The treponemes do not invade the placental tissue untill the fifth month of gestation.
Early manifestation usually appear within first year of life.
M.C. is a bullous rash; rhinitis snuffles
Hepatospleenomegaly; genealized lymphadenopathy
Coombs negative hemolytic anemia; bleeding diathesis thrombocytopenia; nephrosis or nephritis;
osteochondritis and syphilitic metaphysitis (within one month) are also seen.
Patient may have acute syphilitic leptomeningitis progressive hydrocephalus and cranial nerve palsies.
Patient may have congenital glaucoma and chorioretinitis.
Late manifestations (after 2 yr) are triad of Hutchinsons teeth, interstitial keratitis and nerve
deafness. There may be rhagades; saber-shaped tibia and mulberry molars.
Cluttons joints; painless synovitis of the knee joint is most common lesion around the age of
puberty.
There may be pseudoparalysis of Parrot; olympian brow and Higounenakis sign, Congenital syphilis
is strongly suscepted when VDRL and FTA-ABS lg M are positive in the infant.
Specific Treponema pallidum lgM antibodies and IGM antibodies against 47 kD antigen are usuful
for diagnosis of congenital syphilis
Treatment: if CSF
abnormalpenicillin G
normalprocain Penicillin
Interstitial keratitis is treated with corticosteroid
Nerve deafness is treated with oral steroid and penicillin
Erythromycin is indicated if VDRL test remains positive.
45. In early congenital syphilis which one is not seen:
(OPG5th/165) (UP 96)
(a) Keratitis
(b) Vesicular rash
(c) Chorioretinitis
(d) Rhinitis
46. A neonate suspected to have congenital syphilis, diagnostic test to be done is:
(OPG5th/165) (AI/2001)
(a) FTA ABS IgM
(b) FTA-ABS IgG
(c) TPI
(d) VDRL
47. The clinical sign of hyaline membrane disease generally first appears:
(OPG5th/166) (AMU 88 PGI 81)
(a) In the first 6 hours of life
(b) Between 6 and 12 hours of life
(c) Between 36 and 48 hours of life
(d) After 48 hours of life
41 C

42 C

43 D

44 A

45 A

46 A

47 B

Newborn Infants / 29
48. Hyaline membrane in the lung is seen in:
(OPG5th/166) (PGI 87)
(a) Respiratory distress syndrome
(b) Pulmonary oedema
(c) Pneumococcal pneumonia
(d) Acute viral hepatitis
49. Respiratory distress syndrome in infants occur in:
(OPG5th/166) (PGI 89)
(a) Babies of diabetic mother
(b) Premature rupture of membrane
(c) Birth weight more than 2500 gm
(d) Even in term baby
50. Surfactant production in lungs starts at:
(OPG5th/166) (AI/2001)
(a) 28 weeks
(b) 32 weeks
(c) 34 weeks
(d) 36 weeks
HMD (Hyaline membrane disease or idiopathic respiratory distress syndrome):
Occurs commonly in preterm neonates; babies of diabetic mother and infants delivered by cesarean
section or breech.
Ischemia of the alveolar cells results in effusion of albuminous fluid.
Clinical manifestations usually appear 4-6 hr after birh.
2 or more of the following features if found at examination 1 hour apart are enough to diagnose RDSRespiratory rate > 60 enough xiphoid retraction.
Expiratory grunt
Cyanosis
Biochemical changes show metabolic and respiratory acidosis; pCO2 may rise to even 80 mmHg;
Increased excretion of sodium may cause hyponatrenia.
X-ray show ground glass motting.
1/3 babies die within first 3-days
Glucocorticoid administration to the mother two days before delivery in 32 wk of gestation reduces
the risk.
Lacithin: sphingomyelin ratio less than one is always associated with severe RDS.
Maturation of surfactant can be detected by shake test or bubble stability test.
51. Respiratory distress syndrome in the newborn is commonly associated with all except: (OPG5th/166) (UPSC/01)
(a) Diabetic pregnancy (b) Prematurity (c) Following an elective caesarean section (d) Intrauterine growth
retardation
52. Which of the following is false about hyaline membrane disease:
(OPG5th/166) (ALL INDIA/02)
(a) It occurs when delivery occurs before 34 weeks (b) Diabetes mellitus is a predisposing factor (c) Cyanosis
(d) 100% oxygen is the treatment
SCORING SYSTEM FOR RESPIRATORY DISTRESS SYNDROME
Score
0
1
Respiratory rate
<60
60 to 80
Cyanosis
None in
in 40% oxygen
room air
Retraction
None
Mild
Grunting
None
Audible with
stetho
Air entry (crying)
Clear
Decreased
Note: Score
0 to 4
5 to 7
Over 7

Action needed
less than 40$ oxygen
CPAP
assisted ventilation

53. Grunting respiration in the newborn is suggestive of:


(a) Congestive cardiac failure
(b) Respiratory disease

48 A

49 A

2
80 or apnea
in more than 40%
oxygen
Moderate or severe
Audible without
stetho
Barely audible

50 A

51 D

52 D

53 B

(c) Acidosis

(OPG5th/166) (AIIMS 83)


(d) Choanal atresia

30 / Pediatric Buster
54. Foetal lung maturity is assessed by:
(OPG5th/167) (KERALA 94)
(a) L/S ratio
(b) Bilirubin content of amniotic fluid
(c) Ultrasound
(d) Amniocentesis
55. A child delivered at 31st week of gestation has ground glass appearance of lungs and got a cyanotic attack on
4th day, the features point towards:
(OPG5th/167) (AIIMS/99)
(a) ARDS
(b) Meconium aspiration
(c) Hyaline membrane disease
(d) None of the above
56. Which one of the following is true of hyaline membrane disease of the newborn?
(OPG5th/167) (PGI/98)
(a) Prematurity provides relative protection to the occurrence
(b) Maternal steroid exposure increases severity
of the disease
(c) Phosphatidyl glycerol estimation is a reliable method of diagnosis
(d) Surfactant increases
the surface tension of alveoli
57. Most common cause of respiratory distress after birth in first 24 hours is:
(OPG5th/168) (JIPMER 95)
(a) Neonatal sepsis
(b) Meconium aspiration
(c) Bacterial pneumonia
(d) Air embolism
MECONIUM ASPIRATION
About 13% of all deliveries have meconium staining of amniotic fluid (MSAF). Around 6% of such
neonates aspirate meconium into the lungs in utero.
Complications
Airleak syndromes: Pneumothorax, interstitial emphysema, pneumomediastinum, pneumopericardium, pneumoperitoneum, subcutaneous emphysema.
Persistent pulmonary hypertension of the newborn (vide infra)
Pulmonary/cerebral hemorrhage.
Superadded bacterial sepsis.
Subglottic stenosis (due to endotracheal intubation)
Chest radiograph shows overinflated lungs, flat diaphragm, retrosternal lucency, bilateral
pneumonia and signs of airleak syndromes. Cardiomegaly may also be present.
58. In a child cessetion of breathing for 20 sec with bradycardia is:
(a) Apnoea
(b) Dyspnoea
(c) Cheyne-Stokes respiration
(d) None

(OPG5th/168) (AMU 95)

Transient Tachypnea of the Newborn


Respiratory distress syndrome type II
Usually follows uneventful normal preterm or term vaginal delivery or cesarean.
Characterized by early onset of tachypnea and patients usually recover rapidly within 3 days.
Lungs are clear and X-ray shows prominent pulmonary vascular marking; flat diaphragm and
occasionally pleural fluid.
Hypoxemia; hyercapnia and acidosis are uncommon.
Diagnosis is by sudden recovery of infant and absence of roentgenographic reticulogranular pattern
on air bronchography.
The syndrome is believed to be secondary to slow absorption of fetal lung fluid resulting in decreased
pulmonary compliance.
59. Which one of the following is true of transient tachypnoea of newborn (TTN)?
(OPG5th/168) (UPSC 97)
(a) It is the commonest respiratory disorder caused by absence of surfactant
(b) In premature babies it
is often fatal (c) Onset of respiratory distress is immediately after birth and it rarely lasts beyond 48 hr
(d) It often leads to chronic lung disease
60. Transient tachypnea of newborn, true is:
(OPG5th/169) (PGI/99)
(a) Commonly seen in preterm newborn
(b) Respiratory distress resolve in 7-10 days
(c) Interlobar fissure
effusion
(d) Common in baby of diabetic mother
61. All of the following are cause of bronchopulmonary dysplasia except:
(OPG5th/169) (AI 92)
(a) Oxygen toxicity
(b) Theophylline use
(c) Traumatic damage to lungs
(d) Pulmonary oedema due to
capillary damage

54 A

55 C

56 C

57 B

58 A

59 C

60 C

61 B

Newborn Infants / 31

Bronchopulmonary dysplasia is chronic lung injury due to


1. Positive pressure with oxygen (Barotrauma)
2. Pulmonary edema
3. Infection
4. Prematurity
- It occurs due to imbalance b/w elastase and proteinase inhibitors.
X-ray shows either honeycombing or hyperinflated lungs.
- Sequelae may lead to airway obstruction; hypertension and hypertrophy of ventricle
- Furosemide and salbutamol are helpful in management
- Theophylline and corticosteriods provides a dramatic relief.

62. Which is true of diaphragmatic hernia:


(OPG5th/168) (AIIMS 91)
(a) Vomiting occurs early
(b) Chest X-ray is not diagnostic
(c) More common on right side
(d) Pulmonary hypoplasia seen
63. A newlyborn child with respiratory distress and apgar score 3 in 1 minute is resuscited with IPPR on examination
cyanosis. Apex beat shifted to rightside, and scaphoid abdomen the diagnosis is:
(OPG5th/168) (UP 98)
(a) Pneumothorax
(b) Diaphragmatic hernia
(c) Dextrocardia
(d) TOF
Congenital Diaphragmatic hernia (CDH):
More common in left
FCDH is synonymous with herniation through posterolateral formen of Bochdalek.
Malrotation of intestine and pulmonary hypoplasia occurs in all cases.
Associated anomalies
CNS lesions
Omphalocele
CVS lesions
Down syndrome
Most infants have severe respiratory distress within first 24 hr of life.
Abnormal development of mesechyme is new concept for different implications.
Prenatal diagnosis by ultrasonography is common.
Patient has absence of breath sounds, shift of heart sounds; pneumothorax and scaphoid abdomen
(and apparent dextrocardia).
Diagnosis is confirmed by X-ray.
There may be pulmonary hypertension and hypoplasia. A nasogastric tube should be passed to
decompress the stomach.
Stabilized patient may have a repair at 12-24 hr of age. Extracorporeal membrane oxygenation will
be beneficial in unstabilized patients.
64. A 2-year-old child is brought with history of recurrent bouts of spasmodic cough not associated with fever
examination revels a well-preserved child having respiratory rate of 60 p/m peripheral cyanosis, hyperinflated
chest with bilateral decreased air entry and adventitious sounds. Other system examinations revealed nothing
abnormal. The most likely diagnosis is:
(UPSC/01)
(a) Pneumothorex
(b) Congestive heart failure
(c) Bronchial asthma
(d) Bronchiectasis
65. Breakdown of 1 gm of haemoglobin yields about:
(OPG5th/169) (PGI 81, AIIMS 86)
(a) 20 mg bilirubin
(b) 35 mg bilirubin
(c) 5 mg bilirubin
(d) 1 mg bilirubin
66. Which of the folloiwing is/are true regarding physiological jaundice:
(OPG5th/170) (PGI 2000)
(a) It is seen in the first 6 hours of life
(b) Unconjugated hyperbilirubinemia is seen
(c) Associated with
elevated liver enzymes
(d) Clay colored stools
(e) Increased urobilinogen in urine
67. Jaundice in a newborn at birth or before 24 hours is commonly due to:
(OPG5th/170) (AI 95)
(a) Erythroblastosis
(b) Congenital hyperbilirubinaemia
(c) Biliary atresia
(d) Physiological jaundice
68. Jaundice in a newborn seen after 48 hours of birth with bilirubin not exceeding 5 mg% is:
(OPG5th/170) (KERALA 94)
(a) ABO incompatibility
(b) Haemolytic
(c) Neonatal hepatitis syndrome
(d) Physiological jaundice

62 D

63 B

64 A

65 B

66 B

67 A

68 D

32 / Pediatric Buster
69. Physiological icterus usually appears on the ....and regress by the...:
(OPG5th/170) (AIIMS 83)
(a) First day and third day
(b) Third day and seventh day
(c) Third day and ninth day
(d) Ninth day and eleventh day
Physiological Jaundice of the newborn
Usually appears after 30 hours
Peak level (12 mg/dl) is reached on 4th or 5th day and disappears by 7-14 days; rate of increase is
1 mg/dl/day.
In premature babies; maximum bilirubin level reached 12 to 15 mg/dl on 5th-7th days and
disappears by 14 days to a month.
Physiological jaundice is exaggerated in prematurity
- Hypoxia and circulatory insufficiency
- DrugsNovabiocin, salicylate, Vit K
- Cephalohaematoma (1 gm Hb = 34 mg of billirubin on hemolysis)
- Role of infections
Breast milk jaundice (develops gradually on 2nd or 3rd wk) when conjugation of bilirubin is
interfered by pregnanediol) temporary interruption of nurshing for 3-4 days help.
70. Which one of the following is not associated with jaundice in infancy?
(a) Hereditary fructose intolerance
(b) Maple syrup urine disease
syndrome

(OPG5th/170) (UPSC/04)
(c) Galactosemia
(d) Crigler-Najjar

CAUSES OF UNCONJUGATED (INDIRECT) HYPERBILIRUBINAEMIA


(1) Physiologic
(2) Pathologic
Hemolytic disease of the newborn: Rh isoimmunization, ABO incompatibility, minor blood group
incompatibility.
Hereditary spherocytosis
Nonspherocytic hemolytic anemia: G-6-PD deficiency, pyruvate kinase deficiency, alpha-thalassemia
Acquired hemolysis disorders: Vitamin K3 -induced hemolysis, microangiopathies
Septicemia
Increased enterohepatic circulation: Intestinal obstructions, congenital hypertrophic pyloric
stenosis, meconium ileus paralytic ileus, Hirschsprung disease
Inborn errors of metabolism: Familial nonhemolytic jaundice (Crigler-Najjar syndrome) type I
and II, Gilbert disease
Medications: Vitamin K3
Hormones: Breast milk jaundice, hypothyroidism, hypopituitarism.
CAUSES OF DIRECT HYPERBILIRUBINAEMIA
Increased direct bilirubin
Sepsis
Intrauterine infection
Toxoplasmosis
Cytomegalovirus
Rubella
Herpes
Syphilis
Severe hemolytic disease
Biliary atresia
Giant cell hepatitis
Choledochal cyst

69 B

70 B

Newborn Infants / 33
Cystic fibrosis
Galactosemia
Alpha1-antitrypsin deficiency
Tyrosinemia
71. Which of the following factor is responsible for sporadic cases of jaundice in a breastfed infant?
(OPG5th/171) (AIIMS 81, DNB 90)
(a) Excessive iron
(b) Pregnanediole
(c) Vitamin K1 deficiency
(d) None of the above
72. Commonest cause of pathological hyperbilirubinaemia in India is:
(OPG5th/171) (PGI 89)
(a) ABO incompatibility
(b) Rh incompatibility
(c) Septicaemia
(d) G-6-PD deficiency
73. Unconjugated hyperbilirubinaemia in newborn is caused by all except:
(OPG5th/171) (AIIMS 92)
(a) Galactosaemia
(b) Gilberts syndrome
(c) Spherocytosis
(d) Breast milk jaundice
Pathological jaundiace:
Appears within 24 hr of birth.
Increase in bilirubin level at the rate of 5 mg/100ml per 24 hrs.
Absolute bilirubin level more than 15 mg/100 ml.
Jaundice persists more than one week in mature and two week in premature neonates.
74. All of the following factors may contribute to the development of indirect hyperbilirubinaemia in neonates except:
(a) Drugs
(b) Cephalhaematoma
(c) Hypothyroidism
(d) Choledochal cyst (OPG5th/171) (KARN 94)
75. Administration of large doses of vitamin K to a newborn may result in:
(OPG5th/171) (AI 88)
(a) Hyperbilirubinaemia (b) Bulging of the fontanelle (c) Hypoprothrombinaemia (d) Renal vein thrombosis
76. Neonatal jaundice persisting 2 weeks after birth is not seen in:
(OPG5th/171) (AI 92)
(a) Galactosaemia
(b) Rh incompatibility
(c) Hypothyroidism
(d) Breast milk jaundice
CAUSES OF JAUNDICE BASED ON TIME OF ONSET
First day
Rh and ABO incompatibilities (hemolytic disease of the newborn)
Intrauterine infections like toxoplasmosis and cytomegalic inclusion disease
G-6-PD deficiency
Hereditary spherocytosis
Drug administration to mother (vitamin K, sulfisoxazole, salicylates)
Homozygous alpha-thalassemia
Second and Third Day
Physiologic
Hyperbilirubinemia of newborn
Birth asphyxia
Cephalhematoma
Acidosis
Hypothermia
Hypoglycemia
Drugs
Familial nonhemolytic icterus as in Crigler-Najjar disease, Gilberts disease, Dubin-Johnson syndrome.
Fourth to Seventh Day
Septicemia
Syphilis
Toxoplasmosis
Cytomegalic inclusion disease
Extrahepatic atresia of bile duct
Breast milk jaundice
71 B

72 B

73 A

74 D

75 A

76 B

34 / Pediatric Buster
After First Week
Septicemia
Extrahepatic atresia of bile duct
Hereditary spherocytosis
Neonatal hepatitis
Drug-induced hemolytic anemia
Galactosemia
Persistent Jaundice during First Month
Inspissated bile syndrome
Cretinism
Congenital hypertrophic pyloric stenosis
77. Cirrhosis occurs in A/E
(NEL17th/1129) (UP 96)
(a) Galactosaemia
(b) Wilsons disease
(c) -1 antitrypsin deficiency
(d) Cystinuria
78. Long-standing unconjugated jaundice of newborn is caused by following except:
(NEL17th/1131) (AIIMS 90)
(a) Crigler-Najjar syndrome
(b) Breast milk jaundice
(c) Cretinism
(d) Septicemia
79. Neonatal cholestatis is associated with:
(NEL17th/1314) (AIIMS 98)
(a) Glycogen-storage disease
(b) MPS-1H (Hurlers) syndrome
(c) Alpha-hydroxylase deficiency
(d) Niemann-Picks disease
80. A 2-month-old baby presented with jaundice, there is no urobilinogen in urine, but dark urine stools are pale,
the diagnosis goes in favour of:
(NEL 17th/1317)(AIIMS/99)
(a) Hereditary spherocytois (b) Breast milk jaundice (c) Hypothyroidism (d) Congenital biliary duct atresia
81. Gilberts syndrome are true A/E:
(NEL 17th/1320) (UP 97)
(a) Mild conjugated bilirubinaemia
(b) Normal LFT
(c) Normal liver biopsy
(d) Unconjugated
bilirubinaemia
82. Risk of kernicterus is increased in all except:
(OPG5th/172) (AIIMS 84)
(a) Low level of serum albumin
(b) Prematurity
(c) Acidosis
(d) High level of serum albumin
Kernicterus:
Unconjugated hyperbilirubinemia usually > 20 mg% with necrosis of neurons in basal ganglia,
hippocampus and subthalamic nuclei.
There is lethargy and setting sun sign. Convlusion and rigidity usually develop between 3rd and
7th day.
Spastic or athetoid type of cerebral palsy, brownish staining of teeth; deafness, intellectual
retardation and learning disabilities.
Bilirubin reversibly inhibits phosphorylation of synapsin.
The capacity of serum albumin to bind billirubin is low in newborns.
There is ketoacidosis
Risk increases with acidotic; hypothermic hypoglycemic and preterm babies.
CLINICAL FEATURES OF KERNICTERUS
Acute form
Phase 1 (1st 1-2 days): poor sucking, stupor, hypotonia, seizures
Phase 2 (middle of 1st wk): hypertonia of extensor muscles, opisthotonos, retrocollis, fever
Phase 3 (after the 1st wk): hypertonia
Chronic form
First year: hypotonia, active deep tendon reflexes, obligatory tonic neck reflexes, delayed motor skills
After 1st yr: movement disorders
(choreoathetosis, ballismus, tremor), upward gaze, sensorineural
hearing loss.
83. In kernicterus which is increased:
(OPG5th/172) (TN 89)
(a) Conjugated bilirubin
(b) Unconjugated bilirubin
(c) Both
(d) None
84. Which of the following antimicrobial agents increase the risk of kernicterus in newborn?
(a) Gentamicin
(b) Novobiocin
(c) Cloxacillin
(d) None
(OPG5th/172) (PGI 78, 82)
77 B

78 D

79 A

80 D

81 A

82 D

83 B

84 B

Newborn Infants / 35
85. In kernicterus staining of brain is more intense in the:
(OPG5th/172) (PGI 79, AIIMS 81)
(a) Spinal cord
(b) Corpus callosum
(c) Basal ganglion
(d) Cerebral cortex
86. Chances of developing kernicterus appears to be significant when serum level of unconjugated bilirubin reaches:
(a) 50 mg%
(b) 20% mg
(c) 10 mg%
(d) 5 mg%
(OPG5th/172) (PGI 80, AIIMS 86)
87. In a newborn brain damage may be associated with serum bilirubin protein ratio of:
(a) More than 1
(b) More than 2
(c) More than 3 (d) More than 3.5 (OPG5th/172) (AIIMS 81, AMU 89)
88. In Rh isoimmunisation, exchange transfusion is indicated if:
(OPG5th/173) (AIIMS 89)
(a) Cord blood haemoglobin is less than 10 mg/dl
(b) Cord bilirubin is more than 10 mg/dl
(c) History of previous sibling affected
(d) Hydrops fetalis
89. In infants receiving phototherapy all are seen except:
(OPG5th/173) (AIIMS 80, PGI 83)
(a) Have increased insensible water loss
(b) Generally develop of skin bronzing
(c) Should have their eyes
patched during therapy
(d) Often became constipated
Phototherapy
C/l in conjugated hyperbilirubin
Predominant from of bilirubin IX and isomer
Works upto depth of 2 mm takes 1-3 hours for fresh bilirubin from blood to re-equilibrate to tissues,
turn the baby every 2nd hour.
Bilirubin absorb light b/w 400 and 500 nm isomeration and oxidation is maximal between 450460 nm, cool white light (380-700 nm) with peak at 550 nm are most effective.
Green light least effective (form lumi bilirubin)
Light source Blue light
Cool day light fluorescent lamp
Halogen bulb and now fibroptic system under blanket
Level of bilirubin: Normal is 1% of body weight, i.e. 10 mg/dl in 1000 gm baby and 20 mg/dl in
2000 gm baby
Hemolysis lowers the threshold to start phototherapy
Blood bain barrier matures beyond 7 days of life
For Rh immunized babies, begin phototherapy at birth
ADR of phototherapyrash, dehydration, diarrhoea, acute hemolysis, hypocalcemia, bronzing of
skin and retinal image
Control of severe hyperbilirubinemia in full term newborn is with inhibition of bilirubin production
by sn mesoporphyrin, concentration 15 mg/dl 18mg/dl reached in 48-96 hours.
90. Most common cause of cholestatic jaundice in newborn:
(OPG5th/174) (AI 97)
(a) Hypoplasia of biliary tract
(b) Neonatal hepatitis
(c) Choledochal cyst
(d) Physiological cyst
91. A clue to the diagnosis of the following disorder is obtained from simple chemical tests on urine:
(a) Galactosaemia
(b) Cretinism
(c) Neonatal tetany
(d) Mucoviscidosis
(OPG5th/172) (AIIMS 83)
92. Bronze baby syndrome is a complication of:
(OPG5th/173) (JIPMER 89)
(a) Exchange transfusion
(b) Chloramphenicol toxicity
(c) Phototherapy
(d) 100% oxygen therapy
93. Hazards of exchange transfusion include all except:
(OPG5th/174) (PGI 81, AIIMS 83)
(a) Oligaemic shock
(b) Citrate tetany
(c) Cardiac arrest
(d) Hypokalaemia

Exchange Transfusion Indication


- Elevation of uncojugated bilirubin above 18 mg/dl
- Serum bilirubin protein ratio of more than 3.5
- Positive direct Coombs test
- Cord hemoglobin is less than 10 mg/100 ml
- Cord bilirubin is > 5 mg/100 ml.
Rise in bilirubin at a rate faster than 0.5 mg/100 ml/hr.
Rise in bilirubin 15 mg/100 ml in 48 hr
Maternal antibody titre is above 1 : 64.
25% solution of salt free albumin (1 mg/kg) is given before exchange transfusion.

85 C

86 B

87 D

88 A

89 D

90 A

91 A

92 C

93 D

36 / Pediatric Buster
94. A neonate is presented to the hospital with hepatitis and clay coloured stools. Biopsy of the liver shows giant
cells. The diagnosis is:
(OPG5th/174) (AIIMS/NOV/01)
(a) Neonatal hepatitis with extrabiliary atresia
(b) Physiological jaundice
(c) Extrabiliary atresia
(d) Neonatal hepatitis with physiological jaudice
95. In neonatal cholestasis, if the serum gammaglutamyl transpeptidase is more than 600 IU/L the most likely
diagnosis is:
(OPG5th/174) (AIIMS/02)
(a) Neonatal hepatitis
(b) Choledochal cyst
(c) Hypothyroidism
(d) Biliary atresia
96. Phototherapy in non haemolytic hyperbilirubinemia is indicated when the unconjugated bilirubin level is above:
(a) 12 mg%
(b) 15 mg%
(c) 9 mg%
(d) 18 mg%
(OPG5th/174) (AIIMS 99)
Approach to Indirect Hyperbilirubinemia in Healthy Term Infants without Hemolysis
Treatment Strategies

Age (hr)
<24
24-48
49-72
>72

Phototherapy

Intensive
Phototherapy and
Preparation for
Exchange Transfusion

Exchange
Transfusion if
Phototherapy
Fails

15-18
18-20
20

25
30
30

20
25
25

INDICATORS OF PHOTOTHERAPY
Birth weight
Serum bilirubin at which
phototherapy is indicated
2500 g
15 mg/dl
2000-2500 g
12 mg/dl
1500-2000 g
10 mg/dl
1000-1500 g
7 mg/dl
< 1000 g
5 mg/dl
97. A 2 months baby presents with history of jaundice, turmeric coloured urine, and pale stools since birth.
Examination reveals liver span of 10 cm, firm in consistency and spleen of 3 cm. The most specific investigation
for establishing the diagnosis would be:
(OPG5th/174) (AI/03)
(a) Liver function tests
(b) Ultrasound abdomen
(c) Preoperative cholangiogram
(d) Liver biopsy
98. 8 days old neonate with extensor posture suggests:
(OPG5th/175) (TN/99)
(a) Cerebral palsy
(b) Hypoxic-ischemic encephalopathy
(c) Malnutrition
(d) Infection
Neuropathological Lesions in Hypoxic-ischemic Encephalopathy
Infants

Lesions

Term

Parasagittal injury, selective neuronal,


focal and multi-focal ischemia, status
marmoratus

Preterm

Long-term sequalae

Proximal spastic quadriparesis, spastic


cerebral palsy, spastic hemiparesis,
cognitive defects, seizures, extrapyramidal
cerebral palsy
Periventricular leukomalacia, intraventricular Spastic diplegia, visual impairment
hemorrhage

99. Neonatal seizures in the following disorders is associated with poor prognosis except:
(OPG5th/176) (AI 91)
(a) Hypoglycaemia
(b) Intraventricular haemorrhage
(c) Hypocalcaemia
(d) Meningitis
100. CSF examination in a one day old term male infant with convulsions reveals cell count-10, RBCs/HPF-50 cells,
protein-70 mg/dl, sugar-40 mg/dl. The child is likely to be suffering from:
(OPG5th/177) (KARNATAKA/98)
(a) Meningitis
(b) Intracranial bleed
(c) Hypoglycemia
(d) None of the above

94 A

95 D

96 B

97 C

98 B

99 C

100 C

Newborn Infants / 37
101. Treatment of choice for neonatal seizures:
(a) Phenytoin
(b) Valproate
(c) Diazepam
(d) Barbiturates
102. A newborn is screened for hypoglycaemia in following except:
(a) Large for date
(b) Birth asphygia
(c) Rh incompatability

(OPG5th/177) (AIIMS 97)


(OPG5th/177) (AIIMS 92)
(d) Baby of hypothyroid mother

CAUSES OF HYPOGLYCEMIA
Common
Inadequate substrate, especially if feeding is delayed or is sub-optimal: small for dates and preterm
babies.
Relative hyperinsulinism in infants of diabetic mothers.
Secondary to polycythemia.
Secondary to stressful conditions such as hypothermia, sepsis, asphyxia and respiratory distress.
Rare
Hyperinsulin states: Beta cell hyperplasia (nesidioblastosis), adenoma of beta cells.
Deficiency of hormones such as glucagon, hGH, epinephrine, adrenal and ACTH.
Metabolic diseases such as glycogen storage disease, fructose intolerance, ketotic hypoglycemia,
maple syrup urine disease, etc.
103. Hypoglycemia in late infant and child occurs if blood glucose level is:
(OPG5th/177) (UPSC/01)
(a) 40 mg/dl
(b) 60 mg/dl
(c) 10 mg/dl
(d) 20 mg/dl
104. Absent stomach bubble on antenatal ultrasonography is an important finding for antenatal diagnosis of:
(OPG5th/178) (UPSC 96)
(a) Congenital heart disease in the foetus
(b) Oesophageal atresia in the foetus
(c) Omphalocele in the
foetus
(d) Spina bifida in the foetus
105. Consider the following statements: Tracheo-oesophageal fistula in the neonatal period is characterised by all
except:
(OPG5th/178) (UPSC 98)
(a) Associated hydramnios in the mother
(b) Associated pre-eclamptic toxaemia in the mother
(c) Pneumonia in the infant
(d) Excessive oro-pharayngeal secretions
106. In live births the incidence of congenital malformations of oesophagus is:
(OPG5th/178) (AIIMS 78, 81)
(a) 1 in 1000
(b) 1 in 50,000
(c) 1 in 4000
(d) 1 in 7,00,000
Incidence of oesphageal anomalies is 1 in 4000 live births, the commonest being where the upper part of
oesophagus ends blindly and lower part is connected to trachea by fistula.
1. There may be associated poly-hydramnios and single umbilical artery. The baby has excessive
drooling, saliva is frothy and there is chocking and cyanosis with the first feed.
2. On X-ray air bubble is seen in stomach and lung show atelectasis or pneumonia in the right upper
zone.
107. A newborn baby had normal Apgar score at birth and developed excessive frothing and choking on attempted
feeds. The investigaton of choice is:
(OPG5th/178) (AIIMS/03)
(a) Esophagoscopy
(b) Bronchoscopy
(c) MRI chest
(d) X-ray chest and abdomen with the red rubber
catheter passed per orally into esophagus.
108. A newborn has dribbling after feeds. He has respiratory distress and froths at the mouth. Diagnosis is:
(OPG5th/178)(AI/2001)
(a) Tracheoesophageal fistula
(b) Tetralogy of Fallot
(c) Oesophageal atresia
(d) Laryngeal webs
109. Neonatal hypocalcemia to produce seizures is below serum level ofmg%:
(OPG/178) (ORISSA/98)
(a) 7
(b) 8
(c) 9
(d) 10
110. A neonate presents with acute obstruction with multiple fluid levels. Diagnosis is:(OPG5th/178) (ALL INDIA/02)
(a) Pyloric stenosis
(b) Duodenal atresia
(c) Ileal atresia
(d) Lazzs band
111. Baby of a diabetic mother show all except:
(OPG5th/180) (PGI 85)
(a) Hyaline membrane disease
(b) Large baby
(c) Tremulous baby
(d) Hyperglycaemia
112. Child born to diabetic mother can have:
(OPG5th/180) (PGI 89)
(a) Hyperglycaemia
(b) Hypocalcaemia
(c) Growth retardation
(d) Mental retardation
101 D

102 D

103 C

104 B

105 B

106 C

107 D

108 A

109 A

110 C

111 D

112 B

38 / Pediatric Buster
113. Which of the following is not seen in infants of diabetic mother?
(NEL17th/614) (PGI 89)
(a) Hairy pinna
(b) Large tongue
(c) Hypoglycaemia
(d) Asymmetrical ventricular septal hypertrophy
114. Long-term complication of infants born to IDDM mother is:
(OPG5th/180) (AI 95)
(a) DM
(b) Ketotic hypoglycaemia
(c) Obesity
(d) Blindness
Infant of Diabetic Mother:

Complications

Death during last trimester


Preterm delivery
Macrosomia
Holoprosencephaly
Asphyxia
IUGR
Agenesis of lumbar sacral duodenal anorectal region
Neonatal respiratory distress
Asymmetrical septal hypertorophy
Hypoglycemia
Hypocalcemia
Hyperbilirubinemia
Polycythemia
Risk of cardiovascular defects and open
Cardiomyopathy and persistent pulmanary hypertension
Lazy left colon syndrome
Diabetic risk in later life
RV thrombosis and renal agenesis

115. The most common cause of death in patient with congenital diaphragmatic hernia is: (OPG5th/168) (AIIMS 98)
(a) Pul. hypoplasia
(b) Intestinal obstruction
(c) Associated malformations
(d) Septicaemia
116. A newborn baby has been referred to the casualty as a case of congenital diaphragmatic hernia. The first clinical
intervention is to:
(NEL17th/1354)(OPG5th/168)(AIIMS/03)
(a) Insert a central venous pressure line
(b) Bag and mask ventilation
(c) Insert a nasogastric tube
(d) Ventilate with high frequency ventilator.
117. X-ray appearance in hyaline membrane disease in:
(OPG5TH/167) (KERALA 94)
(a) Homogenous ground glass appearance (b) Reticulonodular shadow (c) Normal X-ray (d) Air bronchogram

113 B

114 A

115 A

116 C

117 D

Immunity and
Infectious Diseases

1. Which one of the following immunoglobulins is transmitted to the fetus through placenta?
(a) lgG
(b) lgA
(c) lgM
(d) lgD
(OPG5th/184) (UPSC/02)
2. The most common anomaly seen in the foetus of a mother taking lithium carbonate is:
(UP 97)
(a) Cardiac deformities
(b) Neural tube defect
(c) Limb reduction
(d) Genitourinary deformities
AGENTS AFFECTING FOETUS
Drug

Effect on Fetus

Alcohol

Congenital cardiac, CNS, limb anomalies; developmental delay; attention


deficits; autism.
Azathioprine
Abortion
Busulfan
Stunted growth; corneal opacities; hypoplasia of ovaries, thyroid, and
parathyroids
Carbamazepine
Spina bifida, possible neurodevelopment delay
Carbon monoxide
Cerebral atrophy, microcephaly, seizures
Chorionic villus sampling
Probably no effect, possibly limb reduction
Cigarette smoking
Low birthweight for gestational age
Cocaine/crack
Microcephaly, LBW, IUGR, behavioral disturbances
Lithium
Ebsteins anomaly, macrosomia
Methyl mercury
Minamata disease, microcephaly, deafness, blindness, mental retardation
Misoprostol
Arthrogryposis, cranial neuropathies (Mobius syndrome), equinovarus
Penicillamine
Cutis laxa syndrome
Phenytoin
Congenital anomalies, IUGR, neuroblastoma, bleeding (vitamin K deficiency)
Prednisone
Oral clefts
Quinine
Abortion, thrombocytopenia, deafness
Stilbestrol (diethylstilbestrol Vaginal adenocarcinoma in adolescence
[DES])
Tetracycline
Retarded skeletal growth, pigmentation of teeth, hypoplasia of enamel,
cataract, limb malformations.
Thalidomide
Phocomelia, deafness, other malformations.
Valproate
Vitamin D
Warfarin (Coumarin)

1 A

2 A

Spina bifida, impaired neurologic function


Supravalvular aortic stenosis, hypercalcemia
Fetal bleeding and death, hypoplastic nasal structures

40 / Pediatric Buster
3. True about measles vaccine is that it is:
(OPG5th/195) (AMU 95)
(a) Thermolabile
(b) Thermostable
(c) Both
(d) None
4. Which one of the following is a conjugated vaccine?
(OPG5th/193)(UPSC/04)
(a) Hepatitis B
(b) Rubella
(c) Haemophilus influenzae b
(d) Pertussis
5. Toxic shock syndrome is due to:
(OPG5th/195) (UP 98)
(a) BCG vaccine
(b) DPT vaccine
(c) Measles vaccine
(d) Hepatitis-B vaccine
DISEASES AGAINST WHICH VACCINES ARE AVAILABLE
(a) Diseases preventable by environmental intevention or by immunization: Japanese encephalitis,
tuberculosis, yellow fever, typhoid fever, rabies, rotavirus diarrhea, cholera, hepatitis A, malaria.
(b) Diseases preventable only by immunization: Poliomyelitis, diphtheria, measles, tetanus, rubella,
pertussis, mumps, meningococcal meningitis (A and C serotypes), influenza, H influenzae
meningitis, chickenpox, S. pneumoniae pneumonia, hepatitis B.
6. The diluent used for measles vaccine should be stored at:
(a) 20C temperature
(b) 20C to 8C temperature
temperature
7. Zero dose of OPV is given at:
(a) Birth
(b) 1 month
(c) 3 months
(d) 9 months

(OPG5th/195) (UPSC/2K)
(c) +2C to +8C temperature
(d) Room
(OPG5th/200t) (AMU 95)

NATIONAL IMMUNIZATION SCHEDULE


Birth

BCG and OPV zero dose


(for institutional deliveries)

6 weeks
10 weeks
14 weeks
9 months
18-24 months
5 years
10 yr and 16 yr
For pregnant women

:
:
:
:
:
:
:
:

BCG (if not given at birth), DPT-1 and OPV-1


DPT-2 and OPV-2
DPT-3 and OPV-3
Measles
DPT and OPV (1 booster)
DT
TT
Early inpregnancy TT-1, after 1 month: TT-2

8. The schedule universal programme of immunization in a child begins at birth and end at:(OPG5th/200) ((AI 88)
(a) 3rd year
(b) 4th year
(c) 5th year
(d) 10th year
Chickenpox: Caused by varicella zoster virus
Cowdry type A eosinophillic intranuclear inclusions are seen in cells
Induction period is usually between 14 and 16 days.
The rash is centripetal in distribution appearing on the first day of the illness.
Crusts do not contain viable virus and hence noninfective
9. Chickenpox in children is characterised by all of the following except:
(OPG5th/205) (AI 88)
(a) Presence of live virus in crusts
(b) Lesion appears in crops
(c) Centripetal distribution of rash
(d) Rash shows rapid progression from macules to vesicles
10. Pleomorphic rash is a feature of:
(OPG5th/205)
(DELHI 96)
(a) Chickenpox
(b) Smallpox
(c) Erythema infectosum
(d) Erythema subitum

3 A

4 D

5 C

6 D

7 A

8 C

9 A

10 A

Immunity and Infectious Diseases / 41


Complications
1. Secondary bacterial infection of skin and mucosal lesions
2. Varicella pneumonia
3. Thrombocytopenia
4. Purpura
5. Post-chickenpox encephalitis
6. Arthritis
7. Hepatitis
8. Glomerulonephritis
9. Aseptic meningitis
10. Reyes syndrome.
11. The clinical sign in measles which appears last is:
(a) Kopliks spots
(b) Fever
(c) Rash
(d) Coryza
12. Which one of the following is the infective period of Measles?
(a) 4 days before the onset of symptoms and 5 days thereafter
the onset of rash (d) 30 days from the onset of rash

(OPG5th/207) (JIPMER 88)


(OPG5th/207)(UPSC/03)
(b) Few hours to 5 days
(c) 16 days from

Measles:
Caused by RNA paramyxovirus
There is generalized lymphoid hyperplasia.
There is intranuclear and intracytoplasmic inclusion and multinucleated giant cell are also seen.
On 1st day there is fever2-3 day - Kopliks spot on the inner side of cheek, opposite lower molar teeth.
A maculopapular rash is seen on 4th-5th day
Coombs sign is present
A line of conjunctival inflammation on upper margin
13. Which one of the following is the most complication of measles infection in children? (OPG5th/207)(UPSC/04)
(a) Otitis media
(b) Bronchopneumonia
(c) Encephalitis
(d) Diarrhoea
14. In which one of the following conditions is SSPE a late complication?
(OPG5th/207)(UPSC/03)
(a) Enteric fever
(b) German measles
(c) Herpes zoster
(d) Measles
15. Complication of measles are all except:
(OPG5th/207) (KERALA 95)
(a) Myocarditis
(b) Appendicitis
(c) SSPE
(d) Pancreatitis
Complication:
Most common complication is diarrhoea
Others are:
- Cervical lymphadenopathy
- Bronchopneumonia (young age)
- Encephalitis
- Otitis media
- Myocarditis
- Interstitial pneumonia
- Hepatitis
- Glomerulonephritis
- Achlorhydria
- Appendicitis
- Malnutrition
- Transverse myelitis
- Bleeding manifestation
11 C

12 A

13 B

14 D

15 D

42 / Pediatric Buster
-

Subacute sclerosing (SSPE), panencephalitis (older age)


Primary cause of the death is respiratory complication
Associated vit A deficiency can lead to keratomalacia

16. Treatment of post-measles bronchopneumonia is:


(OPG5th/207) (JIPMER 89)
(a) Antibiotics and oxygen
(b) Only oxygen
(c) Steroids
(d) No treatment is necessary
17. Consider the following statements:
(OPG5th/207)(UPSC/04)
(1) Vitamin A administration reduces the severity and mortality due to measles
(2) Kopliks spots appear on buccal mucosa on 3rd day of appearance of rash.
(3) Neurological complications are the most common complications following measles
(4) Children with measles infection should also be actively immunized to protect against future episodes.
Which of these statements is/are true with regard to measles?
(a) 1 only
(b) 1 and 3
(c) 2 and 4
(d) 1, 2, 3 and 4
18. If a mother has had measles, her newborn baby is immune to this disease for:
(OPG5th/207) (AIIMS 83)
(a) 4 to 6 months
(b) 6 to 12 months
(c) 1 to 3 years
(d) 3 to 6 years
19. A child with fever and coryza for last 3 days developed maculopaplar erythematous rash which lasted for 48 hours
leaving behind pigmentation. The cause is:
(NEL17th/1069)(AIIMS 2000)
(a) Roseola infantum
(b) Measles
(c) Fifth disease
(d) Typhoid
EXANTHEM SUBITUM (ROSEOLA INFANTUM)
High fever and irritability lasting 2-4 days
Fever comes down with appearance of rash (Transient rash)
Febrile convulsions are usual
The rash lasts for 24 hr, spares face and there is occipital and postauricular lymphadenopathy
Etiological agent is herpes virus 6 and child is infected during 6 mon - 3 yr of age.
20. Fever stops and rash begins is diagnostic of:
(OPG5th/208) (JIPMER 95)
(a) Fifth disease
(b) Roseola infantum
(c) Measles
(d) Toxic shocks syndrome
21. Transient rash seen in:
(OPG5th/208) (UP 96)
(a) Sudamen
(b) Scarlet fever
(c) Exanthema subitum
(d) Chickenpox
22. A 3-year-child develops upper respiratory tract infection with high fever on 4th day maculopapular rash appears.
Sudden disappearance of rash and fever are found in:
(OPG5th/208) (UP 98)
(a) Infectious mononucleosis
(b) Measles
(c) Exanthema subitum
(d) Rubella
23. A child with high fever and coryza for last 3 days developed maculopapular erythmatous rash lasted 48 hours,
leaving behind no pigmentation is suffering from:
(OPG5th/208) (ALL INDIA/02)
(a) Measles
(b) Fifth disease
(c) Typhoid
(d) Roseola infantum
24. Most common manifestation of HPV infection in children:
(NEL17th/1085)(DNB 2001)
(a) Single papilloma
(b) Multiple papillomatosis
(c) Osteoma
(d) Morphine
Lesions of Parvovirus B-19:
Erythema infectiosum
Aplastic crisis
Idiopathic thrombocytopenia
Hydrops foetalis
Acute arthralgia
MUMPS
Caused by paramyxovirus (single stand RNA)
Most cases are b/w 5-15 yr
Saliva is highly infective
Period of infectivity is 7 days prior and 9 days after appearance of swelling
Isolation period is until swelling subsides
The virus is localized is salivary glands in 6 to 10 days
One infection provides life long immunity.
16 A

17 B

18 A

19 A

20 B

21 C

22 C

23 D

24 B

Immunity and Infectious Diseases / 43


25. Which of the following is not transmitted transplacentally:
(a) Mumps
(b) Syphilis
(c) Rubella
(d) Toxoplasma
26. The most common complication of mumps in children is:
(a) Orchitis
(b) Aseptic meningitis
(c) Pancreatitis
(d) Myocarditis
27. Isolation period for mumps is:
(a) Until fever subsides
(b) Until pain subsides
(c) Until swelling subsides
28. Sternal oedema is seen in:
(a) Measles
(b) Mumps
(c) Diphtheria
(d) Varicella

(OPG5th/209) (TN 90)


(OPG5th/209) (AI 88)
(OPG5th/209) (KERALA 94)
(d) Until headache subsides
(OPG5th/209) (TN 98)

Complications of Mumps
Edema over manubrium and upper chest due to lymphatic obstruction
Meningoencephalitis
Facial neuritis and auditory nerve damage laeading to deafness
Mumps is probably one of the most common causes of aseptic meningitis
Epididymitis and orchitis are unusual in children but are seen frequenctly in adolscent
Bilateral involvement of testes may rarely lead to ssterilit
Pancreatitis may rarely be followed by diabetes mellitus.
Oophoritis; nephritis; myocarditis, thyroiditis; mastitis; arthritis; dacryoadenitis; uveokeratitis
and thrombocytopenic purpura.
29. Ocular manifestation of mumps is:
(OPG5th/209) (AIIMS 97)
(a) Dacryoadenetis
(b) Chorioretinitis
(c) Anterior uveitis
(d) Memb. conjunctivitis
30. The most frequent complication of mumps infection in young children is:
(OPG5th/209)(UPSC/03)
(a) Meningoencephalitis
(b) Orchitis
(c) Oophoritis
(d) Migratory polyarthralgia
31. The most common form of extrasalivary gland involvement in mumps is:
(OPG5th/209) (UPSC/01)
(a) Encephalomyelitis
(b) Pancreatitis
(c) Aseptic meningitis
(d) Oophritis
32. Acute stage of poliomyelitis lasts for:
(OPG5th/211) (PGI 80, DNB 91)
(a) 10 to 15 days
(b) 1 to 5 days
(c) 1 to 4 weeks
(d) 1 to 2 years
33. All of the following are features of poliomyelitis except:
(OPG5th/211) (ICS/98)
(a) Acute onset
(b) Progressive course
(c) Intact sensation
(d) Febrile onset
34. Diagnosis of poliomyelitis is confirmed by:
(OPG5th/211) (AIIMS 99)
(a) Presence of organism in stool
(b) 4-fold rise in antibody titre
(c) More than 60 days of paralysis
(d) CSF showing organism
35. Death in poliomyelitis is usually due to:
(OPG5th/212) (AI 92)
(a) Hypertension
(b) Respiratory paralysis
(c) Aspiration
(d) Arrhythmias
ACUTE FLACCID PARALYSIS (AFP)
What is AFP?
Acute flaccid paralysis means that paralysis is of acute onset (<4 weeks) and the affected limb or
limbs are flaccid, i.e. floppy or limp. Tone is diminished and deep tendon reflexes are diminished
or absent.
Rapid onset of weakness/paralysis of one limb or limbs in a child < 15 years.
Weakness may include muscles of respiration/swallowing.
Affected limbs are flaccid, i.e. tone is diminished.
There is a reflexia or diminished deep tendon reflexes.
Paralysis may be sudden in all limbs or may extend from one limb and progress to maximum
severity.
Sensory loss may/may not be there.
Common causes of AFP:
Poliomyelitis
Polio like illness caused by other neurotropic enteroviruses.
25 A

26 B

27 C

28 B

29 A

30 B

31 C

32 A

33 B

34 A

35 B

44 / Pediatric Buster

Guillain-Barre syndrome (GBS)


Traumatic neuritis
Transverse myelitis

Other causes:
Post-diarrhoeal hypokalemia
Acute stage of hemiplegia
Tick paralysis
Botulism
Cases of AFP are confirmed as polio if:
They are associated with isolation of wild polio viruses from the stools of the case; or
They have residual neurologic sequalae at 60 days after the onset of paralysis; or
They died before FU could determine whether residual neurologic sequalae was present at 60 days
They were lost before FU could determine whether compatible neurologic sequalae was present at
60 days
Why AFP (not polio) must be reported?
A clinical diagnosis of polio may not be immediately apparent or may be inaccurate
Virus shedding is brief and a virological culture report takes 4-6 weeks to come
The sensitivity of surveillance can be measured only by the AFP rate (1 per 100,000)
Eradication (the ultimate goal) requires sensitive surveillance even in the absence of polio.
Each AFP case:
Case investigation to be done within 48 hours with all demographic and clinical information
2 stool samples to ref. lab collected 24 hrs apart within 14 days of onset of paralysis to be sent
to the lab on ice to be cultured to isolate the polio virus
Community investigation to identify additional cases
Outbreak control additional dose of OPV to children < 5yr of age
Entire village rural area
Municipal ward urban area
Follow up examination 60 days after onset
Purpose of AFP surveillance:
To identify high risk areas or groups where child polio virus continues to spread
To target areas for supplemental immunisation
To monitor progress to assess whether strategies are implemented effectively or not
To certify a country polio free
To detect importation of wild polio virus
Role of laboratory in AFP surveillance:
To confirm polio by virus isolation
To trace the origin of a case by maintaining a reference bank of molecular structure of known viruses
from different geographical areas.
To certify that polio has been eradicated
To assess vaccine potency
Standard performance indicators for AFP surveillance:
AFP rate
1. Target>1/00,00 children < 15 yrs of age. This is an indicator of surveillance sensitivity thus a rate
of < 1/00,000 means surveillance is not complete and cases are probably being missed

Immunity and Infectious Diseases / 45


2. Completeness of monthly reporting (Target > 90%)
No. of monthly reports received
% Complete = 100
No. of monthly reports expected
3. Reported AFP cases investigated < 48 hrs of reporting (Target > 80%)
4. Reported cases with 2 stool specimen collected 24 to 48 hrs apart apart and < 14 days since onset
(Target > 80%)
5. Stool specimen arriving at National Laboratory within 3 days of being sent (Target > 80%)
6. Specimens arriving in good condition at National Lab. (Target > 80%). This is an indicator of the
quality of reverse cold chain.
Good condition at arrival means:
There is ice in the container
The specimen volume is adequate (> 8 gm)
There is no evidence of leakage/desiccation
Appropriate documentation is completed
(Lab request form/reporting form)
7. Specimens with a turn around time < 28 days (Target > 80%). Turn around means time between
specimen receipt and reporting of results
8. Stool specimen from which NPEV was isolated (Target > 10%). If the rate is lower than either is
problem with late or poor stool collection, inadequate reverse cold chain or poor laboratory
technique

36. The average incubation period of infectious hepatitis:


(OPG5th/213) (AIIMS 83)
(a) 7 days
(b) 15 days
(c) 25 days
(d) 35 days
37. Incubation period of hepatitis A is:
(OPG5th/213) (AMU95)
(a) 2 weeks-2 months
(b) 6 weeks-6 months
(c) 6 days-6 weeks
(d) 2 days-2 weeks
38. All of the following statements regarding perinatal transmission of hepatitis B in the newborn are true, except:
(OPG5th/214) (UPSC/01)
(a) HB vaccine can be delayed for newborn beyond 48 hours if mother is HbsAg positive
(b) Maximum
transmission of hepatitis B from mother to child occurs at the time of delivery
(c) HbeAg positivity in
mother increases the risk of transmission
(d) 70% of newborn infected at the birth can become chronic
carriers
39. In acute hepatitis B infection, the earliest abnormality detected is:
(OPG5th/215) (AIIMS 83)
(a) Raised serum transaminases
(b) Antibodies to hepatitis B antigen
(c) Detection of surface antigen
(d) Anorexia and lassitude
40. Hepatitis C infection in the neonate is closely related to:
(OPG5th/216) (PGI/2000)
(a) Time of infection (b) Method of delivery (c) Duration of the infection (d) Association with HIV infection
41. Epstein-Barr virus is the aetiological factor for:
(OPG5th/219) (AIIMS 83)
(a) Congestive cardiac failure
(b) Infective mononucleosis
(c) Hodgkins lymphoma
(d) Lymphocytic choriomeningitis
INFECTIOUS MONONUCLEOSIS
Caused by Epstein-Barr virus (DNA virus of herpes groups)
Virus elicits both humoral and cell-mediated immunity
There are many atypical T lymphocytes
Children are prone to B cell lymphoma
I.P. is 4-5 week

36 C

37 A

38 A

39 C

40 A

41 B

46 / Pediatric Buster

C/F- Fever; malaise; sore throat, loss of apetite; lymphadenopathy and maculopapular rash
Hepatosplenomegaly with elevated SGOT and SGPT levels
Frank jaundice is rare.
Rash usually develops on ampicillin intake
Patient may have aseptic meningitis or meningoencephalitis.
The disease is self limiting (2 mon)

Complications
Splenic rupture
CNSconvulsions, GB syndrome; peripheral neuropathy
Immune hemolytic anemia with thrombocytopenia
Myocarditis and hepatitis
Diagnosis is accomplished by absolute increase in lymphocytes which are large and atypical
Heterophile antibodies are tested by positive Paul Bunnel test (often negative in under 4 yr of child)
A screening test (monospot) has been developed for rapid diagnosis.
No specific treatment but acyclovir may be used in chronic infectious mononucleosis and EBV
associated polyclonal lymphoproliferation.
42. All of the following statements regarding infectious mononucleosis are true except:
(OPG5th/219) (AI 90)
(a) The onset is acute with fever, sore throat and lymphadenopathy
(b) Atypical lymphocytes are seen in
peripheral blood smear
(c) Acute lymphoblastic leucaemia is a common sequel
(d) Heterophile antibody
test is useful in diagnosis.
HIV
Perinatal HIV-1 transmission in the leading cause of pediatric AIDS
Most children are diagnosed b/w 2 months and 3 year of age.
Clinically present as prolonged diarrhoea
Failure to thrive
Oral candidiasis
Hepatosplenomegaly
Septicemia with Strepto Strep Pneumoneae or Salmonella
Chronic sinusitis may also be the major sign.
LIP is a distinctive presentation of HIV-1 infection and is probably the most common clinical
presentation in children.
LIP is defined as reticulonodular pulmonary infiltrates composed or CD8 + lymphocyte persisting
for 2 month or more with or without higher adenopathy that donot respond to antimicrobials.
The most common and highly lethal opportunistic infection is P. carinii pneumonia.
M. tuberculosis and malaria are also major opportunistic pathogens in AIDS other are Candida and
Toxoplasma.
Neoplasms are relatively uncommon in pediatric HIV-1 infected patients.
The age at the initial clinical presentation and CD4 + lymphocyte count are the major factors in
prognosis.
Drugs to be used in HIV-1 infected children arezidovudine and dideoxylnosine.
43. Percentage of HIV infection in child of a HIV +ve mother is:
(OPG5th/220) (KERALA 97)
(a) 20-30%
(b) 10-20%
(c) 70-80%
(d) 100%
44. A 2-year-old girl presents with recurrent episodes of diarrhoea (mucus and blood at times ) hepatosplenomegaly,
perianal excoriation and resistant oral thrush. She weighs 6 kg and measures 78 cm in hieght. The most likely
diagnosis is:
(OPG5th/222) (UPSC/01)
(a) Necrotising enterocolitis
(b) Lactose intolerance
(c) HIV infection
(d) Campylobacter jejuni infection
45. All of the following methods are used for the diagnosis of HIV infection in a 2-month-old child, except:
(a) DNA-PCR
(b) Viral cultue
(c) HIV ELISA
(d) p 24 antigen assay
(OPG5th/222) (AIIMS/03)
42 C

43 A

44 C

45 C

Immunity and Infectious Diseases / 47


46. Manifestations of AIDS in children are all except:
(OPG5th/222) (AIIMS 97)
(a) Hypergammaglobulinaemia
(b) Thrombocytopenia
(c) Kaposi sarcoma common
(d) Cutaneous anergy
47. Feature of HIV infection in children includes one of the following:
(OPG5th/222) (AIIMS 98)
(a) Diarrhoea due to Cryptococcus
(b) Kaposi sarcoma frequent
(c) Recurrent chest infection with atypical
organisms
(d) Recurrent candidiasis
48. A 4-month-old HIV positive child following URTI, developed sudden onset of breathlessnes. The chest X-ray shows
hyperinflation. The O2 saturation was greater than 90%. The treatment of choice is:
(a) Nebulized acyclovir (b) Ganciclovir (c) Ribavirin (d) Cotrimoxazole
(OPG5th/223) (AIIMS /MAY/01)
49. Colour of diphtheritic membrane is:
(OPG5th/225) (JIPMER 95)
(a) Grey
(b) White
(c) Yellow
(d) Cream
50. Prophylaxis for household contact of diphtheria is:
(OPG5th/226) (KERALA 95)
(a) Ampicillin
(b) Rifampicin
(c) Erythromycin
(d) Tetracycline
51. The cell chemistry in a 2 years old child presenting with history of diarrhoea and inadequate water intake will
be:
(AIIMS/2000)
(a) Raised ECF with rise in sodium concentration
(b) Raised ECF with fall in sodium concentration
(c) Decreased ECF with rise in sodium concentration
(d) Decreased ECF with fall in sodium concentration
52. Congenital passive immunity is not observed in:
(OPG5th/227) (AIIMS 83)
(a) Rubella
(b) Mumps
(c) Poliomyelitis
(d) Whooping cough
53. Incubation period of pertussis is:
(OPG5th/227) (AIIMS 97)
(a) 7 days
(b) 7-14 days
(c) 15 to 28 days
(d) 28 days
Pertusis is the only disease in which child is not protected by antibodies from the mother.
Clinical disease is associated with encapsulated, phase 1 strains
Source of infection in case, carrier state does not exist
Most infectious during catarrhal stage and complications are more in paroxysmal stage.
Infective period extends from a week after exposure to about 3 weeks after onset of paroxysmal
stage
There is no life long immunity
IP is 7-14 days and case fatality is 4-15% in infants
Fever is absent and post tussive vomiting is characteristic
Chief complications are bronchitis, broncho pneumonia (Strepto pneumoniae and haemophillus) and
bronchiectasis
Others are- Convulsions and encephalopathy
- Hernia; rectal prolapse
- Subconjunctival haemorrhage
Diagnosis is by fluoroscent antibody staining of laryngeal swab and culture on Bordet gengou
media.
54. All are complications of pertussis except:
(OPG5th/227) (AIIMS 97)
(a) Subconjunctival haemorrhage
(b) Hernia
(c) Bronchitis
(d) Encephalopathy
55. The antibiotic of choice of pertussis is:
(OPG5th/228) (AIIMS 79, AP 90)
(a) Ampicillin
(b) Gentamicin
(c) Erythromycin
(d) Penicillin
56. DPT is contraindicated in:
(OPG5th/228) (AI 91)
(a) Family H/o convulsion and neurological illness
(b) Acute upper respiratory infection
(c) Malnutrition
(d) Evolving neurological illness
57. Contraindication to DPT is all except:
(OPG5th/228) (AI 95)
(a) Local reaction to previous DPT
(b) High fever
(c) Infantile spasm
(d) Seizures
58. Excessive crying is seen after which vaccination:
(OPG5th/228) (AMU 95)
(a) Polio
(b) DPT
(c) BCG
(d) Measles
59. The following statements regarding typhoid fever in children is true:
(OPG5th/229) (AI 90)
(a) Mild splenomegaly is usual
(b) Leucocytosis is characteristic
(c) Encephalitis is a common complication
(d) Culture of urine is usually positive in the first four days of fever.
46 C
59 A

47 C

48 C

49 A

50 C

51 D

52 D

53 B

54 C

55 C

56 D

57 A

58 B

48 / Pediatric Buster
Typhoid fever:
Incubation period is 14 days
Temperature rises in a step ladder pattern
Constipation is frequent
There is bradycardia
Rash comes on 6th day in crops as rose spots which fade on pressure.
Adbomen is distended and a tympanic note is heard on tapping.
Spleen is palpable 1 or 2 cm below the costal margin.
Basal rales are heard over lungs
Child has muttering delirium and pick at bed clothes peculiar of typhoid state.
Hemorrhage and perfoation may occur in 2nd or 3rd wk.
There may be chills, myalgia, anorexia, cough vomiting, arthralgia etc.
Complications: Parotitis; bronchitis; pneumonia; myocarditis, fatty liver; encephalitis; peripheral
neuritis; aphasia; cerebral vein thrombosis; osteomyelitis, uveitis etc.
Widal agglutination test shows a diagnostic titre of 1/80 or more for O antigen of S. typhi.
A four-fold rise in agglutinin titre of nonimmune individual is diagnostic
Elevation of antibody titre against O (somatic) antigen has a better diagnostic value.
Anamnestic reaction means the rise of titre of H antibody with non-tyhoid fevers but the titre of
O antibodies does not rise.
Blood counts show below normal leucocyte count with a moderate neutropenia and relative
lymphocytosis.
Chloramphenicol is useful drug; others are ampicillin, cotrimoxazole and ciplox (avoided in children
due to possible risk to growing cartilage)
Prevention is by Ty 21 a (Typh oral) 3 capsules on 1, 3, and 5 days.)
60. Ghons focus is:
(OPG5th/232) (TN 90)
(a) Primary complex
(b) Post-primary TB
(c) Miliary TB
(d) TB lymph nodes
61. All of the following statements with reference to childhood tuberculosis are correct except:
(OPG5th/232) (UPSC/2001)
(a) Primary complex is mostly symptomatic (b) Mantoux test is useful to detect infection
(c) Early morning
gastric lavage for AFB is a useful test (d) Chest X-ray is the most important tool in diagnosis of miliary
tuberculosis
62. False-negative tuberculin reaction occurs in children in the following except:
(OPG5th/232) (AI 91)
(a) Very severely malnourished child
(b) Patient on corticosteroid therapy
(c) 4-6 weeks following measles
(d) Atypical mycobacteriosis.
63. Mantoux test reading of less than 5 mm indicates:
(OPG5th/234) (JIPMER 91)
(a) Tuberculous infections
(b) Disseminated TB
(c) Susceptibility to TB
(d) Immunity to TB
64. Dose of rifampicin in mg/kg body weight/day for treatment of tuberculosis in a child is:
(a) 5
(b) 10
(c) 15
(d) 20
(OPG5th/236) (AIIMS 81, 91)
65. A poverty stricken mother suffering from active tuberculosis delivers a baby. Which one of the following advices
would be the most appropriate in her case?
(OPG5th/237) (UPSC 96)
(a) Breastfeeding and BCG immunization
(b) Breastfeeding and isoniazid administration
(c) Expressed breast milk and BCG immunization
(d) Stop feeds and isoniazid administration

Indications of treatment of primary TB in children with INH and r-cin for 9 mon are:
1. Positive tuberculin reactors
2. Under age of 3 yrs
3. Recent conversion significant
4. Radiological evidence
5. Presence of tuberculous toxemia
6. Children suffering from or recently recovered from measles or whooping cough.

60 A

61 A

62 D

63 C

64 D

65 B

Immunity and Infectious Diseases / 49

All patients with positive tuberculin reaction do not need treatment.


Chemoprophylaxis with 5-10 mg/kg/day of isoniazid everyday is given to newborn infants born to
mother with sputum positive for acid-fast bacilli. The infant should be on chemoprophylaxis until
the mothers sputum becomes bacteriologically negative. Thereafter chemoprophylaxis is stopped
and BCG vaccination is given.

66. Basanti age 29 years from Bihar presents with tuberculosis and she delivers baby. She will be instructed all
of the following except:
(OPG5th/237) (AI/2001)
(a) Give baby BCG
(b) Withhold breastfeeding
(c) Give ATT for 2 years
(d) Proper disposal of sputum
67. A mother has active TB and recently delivered a child; what is to be done to newborn:(OPG5th/237)(AI/2000)
(a) Give BCG and isolate the baby
(b) Give BCG, INH and RMP and continue breastfed
(c) Give BCG and
no breastfeeding and no isolation
(d) Give BCG and breastfeeding is allowed
Baby born to mother with TB (diagnosed in 3rd trimester or during delivery)
Breastfeeding must be continued.
BCG vaccine should be given at birth
If chest X-ray is normal, then 6 HR
If chest X-ray is abnormal, then 2 HRZ/7 HR
Congenital tuberculosis: 2 HRZ/7HR4
68. Which of the following are true about cerebral malaria:
(OPG5th/242) (PGI 2000)
(a) Quinine is used in the treatment
(b) It is seen more in the hyperendemic areas
(c) Most common
presentation is with seizures
(d) Associated with hypoglycemia
(e) It is more common in infants
MALARIA
Sickle cell trait; thalassemia; G6PD deficiency and altered ATP in erythrocytes are relatively
immune from malaria.
Chronic progressive nephrotic syndrome is only observed in P. malarial infections.
Cerebral malaria is rare in children with protein calorie malnutrition.
Individuals infected with malaria respond poorly to other antigens such as tetanus toxoid.
69. The mechanism of diarrhoea in giardiasis is prevention of absorption of fat by:
(a) Mechanical coating of the walls of intestine
(b) Secondary bacterial infection
(d) Production of endotoxins
70. Treatment of giardiasis is:
(a) Mebendazole
(b) Piperazine
(c) Metronidazole
(d) Bephenium
71. Which of the following is not a usual feature of ascariasis:
(a) Abdominal pain
(b) Urticaria
(c) Anemia
(d) Loefflers syndrome

66 B

67 A

68 A

69 A

70 C

71 C

(OPG5th/251)
(CMS 85)
(c) Production of exotoxins
(OPG5th/251) (AIIMS 83)
(OPG5th/254) (AIIMS 92)

Gastrointestinal System

1. A child with hypertrophic pyloric stenosis will have:


(OPG5th/264) (ALL INDIA/02)
(a) Hyperchloremic acidosis with aciduria
(b) Hypochloremic alkalosis with paradoxical aciduria
(c) Hypochlorimic acidosis with paradoxical alkalosis
(d) Normochloremic aciduria
2. The metabolic derangment in congenital pyloric stenosis is:
(OPG5th/264) (AIIMS/02)
(a) Hypocholoremic alkalosis
(b) Hyperchloremic alkalosis.
(c) Hyperchloremic acidosis
(d) Hypochloremic
acidosis
3. Regarding congenital hypertrophic pyloric stenosis all are correct except:
(OPG5th/264)
(a) Weight loss
(b) Diarrhoea
(c) Visible peristalsis
(d) Ramsteds operation is done

Congenital pyloric stenosis is characterized by:


Vomiting after every feed
Visible peristalsis
A pea size tumour
There is also hypochloremic alkalosis
Ramstedt (pyloromyotomy) is treatment of choice

4. The first and most common symptom of intussusception is:


(a) Pain
(b) Vomiting
(c) Diarrhoea
(d) Abdominal distention

(OPG5th/268) (AIIMS 83)

Intussusception in infants is suspected by severe colicky abdominal pain; blood stained stools like
current jelly and empty right iliac fossa.

5. Infective diarrhoea is caused by:


(a) Rotavirus
(b) Calycivirus
(c) Flavivirus
(d) Enterovirus
6. The most common virus causing diarrhoea in infants is:
(a) Rotavirus
(b) Myxovirus
(c) Adenovirus
(d) Rhabdovirus

(OPG5th/270) (TN 97)


(OPG5th/270) (TN 89, PGI 88)

Viral agents of gastroenteritis:


In order of frequency
- Rotavirus
- Astrovirus
- Enteric adenovirus
- Norwalk virus (M.C. cause of gastroenteritis outbreaks in older children and adults)
1 B

2 A

3 B

4 A

5 A

6 A

Gastrointestinal System / 51

Rotavirus has 5 group (A; B; C; D; E) group A has no antigenic relationship to other.


Rotavirus strains are species specific.
Subgrouping is determined by antigenic structure of the inner capsid protein Vp6
It is most common in winter months in children b/w 3 mon - 2yr.
Virtually all children have got serological evidence by 4-5 yr.
Incubation period is fever than 48 hr with fever and vomiting followed by onset of watery stools.
Vomiting and fever abate during the 2nd day of illness but diarrhoea after continuous for 5-7 days.
Almost all newborns infected with rotavirus outbreaks of necrotizing enterocolitis are seen with
newstrain.
Isotonic dehydration with acidosis is seen with severe viral enteritis.
Stools are free of blood and leukocytes.
Commercial radioimmunoassay offers 90% specificity and senstivity for group A rotavirus and
enteric adenovirus.

7. Infective diarrhoea in infancy is commonly due to:


(OPG5th/270) (AIIMS 83)
(a) Pseudomonas
(b) E. coli
(c) Klebsiella
(d) Shigella
8. Most common cause of diarrhoea in children is:
(OPG5th/270) (AI 95)
(a) Vibrio cholerae
(b) E. coli
(c) Rota virus
(d) Pneumococcus
9. Content of citrate in ORS is......millimoles:
(OPG5th/272) (JIPMER 95)
(a) 20
(b) 25
(c) 30
(d) 35
10. Molar concentration of oral rehydration fluid A/E:
(OPG5th/272) (PGI 87)
(a) Sodium 30
(b) Potassium 20
(c) Bicarbonate 30
(d) Glucose 111
11. The function of glucose in ORS:
(OPG5th/272) (PGI 96)
+
(a) Increase Na absorption by co-transport
(b) Gives sweet taste to ORS
(c) Increase osmolality of ORS
(d) Increase Na+K+ pump activity
12. Which one is not used in ORS:
(OPG5th/272) (KERALA 94)
+
+
(a) Na
(b) K
(c) Glucose with salt
(d) Glucose without salt
13. Correct regarding concentration of elements of WHO ORS solution:
(OPG5th/272) (PGI 2000)
(a) Glucose 111 mg
(b) Sodium 80 mEq
(c) Potassium 30 mEq
(d) Chloride 80 mEq
(e) Osmolality
is less than that of plasma
Composition of recommended ORS solution for severely malnourished (Reosmal)
Component

Reosmal (mmol/L)

Standard ORS (mmol/L)

Glucose
Sodium
Potassium
Chloride
Citrate
Magnesium
Zinc
Copper
Osmolarity

125
45
40
70
7
3
0.3
0.045
300

111
90
20
80
10
311

14. Treatment in 6-month-old child with acute watery diarrhoea without signs of dehydration is:
(a) Mothers milk and household fluids
(b) ORS and antibiotics
(c) Mothers milk and antibiotics
(d) Mothers milk and ORS
(OPG5th/273) (JIPMER 95)
15. A 2 years old child 6.5 kg in weight presents with vomiting and diarrhoea for 2 days and skin over abdomen when
lifted takes seconds to come back, the diagnosis goes in favour of:
(OPG5th/273) (AIIMS 2000)
(a) Mild dehydration
(b) Severe dehydration
(c) No dehydration
(d) Dehydration cannot be detected in
this case from abdominal skin
7 B

8 C

9 C

10 A

11 A

12 D

13 A

14 A

15 B

52 / Pediatric Buster
Assessment of Dehydration
Mild dehydration (3-5%): Normal or increased pulse, decreased urine output thirsty, normal physical
examination.
Moderate dehydration (7-10%): Tachycardia, little or no urine output, irritables/lethargic, sunken
eyes and fontanel, decreased tears, dry mucous membranes, mild tenting of the skin, delayed capillary
refill, cool and pale.
Severe dehydration (10-15%): Rapid and weak pulse, decreased blood pressure, no urine output, very
sunken eyes and fontanel, no tears, parched mucous membranes, tenting of the skin, very delayed
capillary refill, cold and mottled.
16. A two old girl has weight of 6.7 kg. She suffered an attack of diarrhoea. If skin is pinched and get back in seconds.
Which of the folowing is true about her:
(OPG5th/273) (ALL INDIA/02)
(a) Severe dehydration
(b) Mild Dehydration
(c) Moderate dehydration
(d) No comment can be made
on these data
17. A child suffering from acute diarrhea is brought to the casualty and is diagnosed as having severe dehydration
with pH of 7.23, serum Na-125, serum K-3, HCO316. The best IV fluid of choice is:
(OPG/5th/274) (AIIMS /MAY/01)
(a) 3 % saline
(b) Normal saline
(c) N/3 saline + 10% dextrose
(d) N/3 saline + 5% dextrose
18. A 5-yr-old boy passed 18 loose stools in last 24 hr and vomited twice in last 4 hours. He is irritable but drinking
fluids. The optimal therapy for this child is :
(OPG5th/ 274) (AI/03)
(a) Intravenous fluids
(b) Oral rehydration therapy
(c) Intravenous fluid initially for 4 hr followed by oral
fluids
(d) Plain water ad libitum.
19. Treatment of choice in severe dehydration is:
(OPG5th/274) (KERALA 94)
(a) Isolyte-P
(b) Normal saline
(c) Plasma
(d) Ringer lactate
Composition of intravenous solution
Fluid

[Na+]

[Cl-]

[K+]

[Ca2+]

[Lactate-]

Normal saline (0.9% NaCl)


Normal saline (0.45%NaCl)
Normal saline (0.225% NaCl)
Ringer lactate

154
77
38.5
130

154
77
38.5
109

28

20. When severe dehydration in a neonate occurs, amount fluid replacement in l hour:
(a) 20-40 ml/kg
(b) 5-10 ml/kg
(c) 10-15 ml/kg
(d) 15-20 ml/kg
(OPG5th/274) (CUPGEE 2001)
21. A 6-month-old child having severe dehydration comes to the casualty with weak pulse and unrecordable BP.
Repeated attempts in gaining IV access has failed. The next best step is:
(OPG5th /274)(AIIMS /MAY/01)
(a) Venesection
(b) Jugular vein catheterisation
(c) Intravenous IV fluids
(d) Try again
22. Coeliac disease is due to sensitivity to:
(OPG5th/281) (AIIMS 83)
(a) Milk
(b) Maize
(c) Wheat
(d) Meat
23. Which one of the following cereals is not harmful in the case of gluten sensitive enteropathy ?
(a) Rye
(b) Maize
(c) Wheat
(d) Barley
(OPG5th/281) (UPSC 98)

Coeliac disease is characterized by excretion of fat in stools and serum xylose levels less than
20 mg/10 ml. Treatment is exclusion of all cereals contaning gluten (wheat; rye, barley and oats)
from the diet.

24. True in cystic fibrosis are A/E:


(OPG5th/282) (UP 96)
+
(a) Sterility
(b) Steatorrhoea
(c) Dry mouth
(d) Increased sweat Na concentration
25. In cystic fibrosis Pseudomonas aeruginosa is most common organism. The next most common is:
(a) Streptococcal
(b) Klebsiella
(c) Pneumococci
(d) Staphylococcus aureus (OPG5th/282) (JIPMER 95)

16 D

17 B

18 B

19 D

20 A

21 C

22 C

23 B

24 C

25 D

Gastrointestinal System / 53
26. Cystic fibrosis is:
(OPG5th/282) (UP 96)
(a) Autosomal dominant
(b) Autosomal recessive
(c) X-linked dominant
(d) X-linked recessive
27. The following can cause recurrent respiratory infection:
(OPG5th/283) (AIIMS 84)
(a) Tetralogy of Fallot
(b) VSD
(c) Cystic fibrosis
(d) TGV without pulmonary stenosis
Cystic Fibrosis:
Autosomal recessive transmission.
Disease is characterized by chronic airway infection that ultimately leads to bronchiectasis and
bronchiolectasis; exocrine pancreatic insufficiency abnormal sweat gland function and urogenital
dysfunction
There is impaired digestion and absorption; salty skin due to increased secretion of chlorides in the
sweat; Intestinal obstruction; biliary cirrhosis of liver, intussusception; meconium ileus; chronic
diarrhoea and recurrent respiratory tract infection.
Analysis of sweat and chloride is a reliable diagnostic test level of Cl > 70 mEq/L.
There is decrease vital capacity and decreased lung compliance.
Haemophilus and S. aureas are often the infective agents. Mycobacerium tuberculosis is rare.
Earliest chest X-ray change in lung is hyperinflation.
Pneumothorax is common; respiratory failure and cor pulmonale are prominent features in cystic
fibrosis.
Treatment is with pancreatin; suitable antibiotics, humidification of air.
28. A young child has eaten icecream and got the symptoms of abdomen distension, diarrhoea and flatulence. Same
episode is seen with ingestion of icecream/milk also, the child is suffering from deficiency of:
(a) Lactase
(b) Sucrase
(c) Salivary amylase
(d) Pancreatic amylase
(OPG5th/282) (AIIMS/99)
29. A mother kissing her baby finds that the babys skin is salty, the diagnosis is:
(OPG5th/283) (TN 92)
(a) Fanconi syndrome
(b) Thalassaemia
(c) Cystic fibrosis
(d) Niemann-Pick disease
30. Mucoviscidosis is most commonly related to:
(OPG5th/283) (AI 96)
(a) Fibrocystic disease of pancreas
(b) Duodenal atresia
(c) Diaphragmatic hemia
(d) Annular pancreas
31. Cause of meconium ileus is:
(OPG5th/283) (AI 96, 98)
(a) Fibrocystic disease of pancreas
(b) Liver aplasia
(c) Malnutrition
(d) Cirrhosis
32. Abetalipoproteinaemia is first manifested clinically by:
(OPG5th/283) (JIPMER 78, PGI 80)
(a) Hypohydrosis
(b) Tremors
(c) Steatorrhoea
(d) Convulsions

Abetalipoproteinaemia is autosomal recessive disorders characterized by steatorhoea and


acanthocytosis and there is no effective treatment.

33. Portal hypertension in children in India is commonly due to:


(a) Indian childhood cirrhosis
(b) Extrahepatic portal venous obstruction
(d) Hepatic out flow tract obstructions.

(OPG5th/294) (AIIMS/03)
(c) Idiopathic portal hypertension

Indian Childhood Cirrhosis:


Onset at first year of life
There is firm hepatomegaly with sharp leafy margin.
More common in Hindus; rare in Muslims and Christians.
Early weaning may be a cause
Etiology may behigh levels of copper (serum and tissue copper are elevated).
- Aflatoxin B
- Viral infection
- Immunologic
- Nutritional
Surface of the liver is finely nodular
Mallory hyaline is seen in hepatocytes.
26 B

27 C

28 A

29 C

30 A

31 A

32 C

33 B

54 / Pediatric Buster

Bile duct proliferation is absent.


There is chalky pasty stools hepatosplenomegaly and distension of abdomen.
A peculiar garlic odour is present in patient with impending liver cell failure.
Treatment
- Steroids and gammaglobin
- Penicillamine to chelate copper
Supportive therapy for anemia and hypoproteinemia.

34. The most common cause of abdominal pain in children is:


(a) Porphyria
(b) Worm colic
(c) Lead poisoning
(d) Appendicitis

34 B

(OPG5th/266) (PGI 87)

Haematological Diseases

Hemoglobin:
Six hemoglobins are detected normally in humans as = gower -1
- Embryonic =
gower - 2 Poctland
- Fetal
=
Hb F
Hb A
- Adult
=
Hb A2
In embryos of 4-8 wk gestation the gower hemoglobin predominate but by the 3rd month they have
disappeared.
After 8th gestational wk HbF is the predominant hemoglobin; in the 6 months old fetus it
constitutes 90% of the total hemoglobin then a gradual decline; so that at birth Hbf averages 70%
of the total.
HbA can be detected as early as 16-20 wk of gestation. By the 6 months of gestation there is about
5-10% of HbA and at term 30% by 6-12 months of age the normal adult hemoglobin pattern appears.
Iron deficiency Anemia:
With term pregnancy 900 mg of iron is lost by mother to fetus; placenta and PPH.
Normal infant has 250-300 mg of iron at birth.
Pallor is major symptom of deficiency.
Patients suffer from frequent infections; splenomegaly cardiac enlargement and systolic flow
murmurs and protein losing enteropathy.
Some children develop a liking for nonedible substance (PICA) and are at risk of lead poisoning.
Sequence of changes are
- Decreased storage iron
- Decreased ferritin < 10 mg/ml
- Increase TIBC > 350 mg/dl
- Decreased serum iron
- Increased reticulocyte count
- FEP/Hb > 2.8 mg/g
Elemental iron is given in optimal dose of 5 mg/kg clinical improvement is seen within 24 hr.
Initial bone narrow response is seen in 48 hrs.
Rise in reticulocyte count occurs by 2-3 days and peaks on 10th day of starting the therapy.
1. Child with microcytic-hypochromic anemia not responding to iron the disease to be excluded is:
(a) Thalassaemia
(b) Haemophilia
(c) Renal failure
(d) Sickle cell anemia
(OPG5th/298) (AI 93)
2. Microcytic-hypochromic anemia is seen in all of the following conditions except:
(OPG5th/298) (AI 90)
(a) Beta thalassaemia
(b) Lead poisoning
(c) Iron deficiency anemia
(d) Chronic renal failure.
1 A

2 D

Hamatological Diseases / 56
Microcytic-hypochromic anemia:
- Iron deficiency anemia
- Thalassemia
- Pyridoxine responsive anemia
- Lead poisoning
- Dyserythropoietic anemia
Macrocytic anemia:
- Cobalamin deficiency (Vit B12)
- Folic acid
- Cirrhosis of liver
- Hereditary orotic aciduria
- Di Gugliemos syndrome (AML + Anemia)
- Thiamine responsive
- Hyperthyroidism
- DrugsPAS; colchicine; neomycin DHFRase inhibitors, alcohol 5-FU; hydroxyurea, acyclovir;
zidovudine and Phenytoin.
3. The haemoglobin to appear first in the fetus is:
(NEL 17th/1602) (AIIMS 83)
(a) Hb A
(b) Hb A2
(c) Hb F
(d) Hb Gowers
4. Fetal Hb equals adult Hb at:
(NEL17th/1602) (TN/99)
(a) At birth
(b) 2 months
(c) Oxygen
(d) Ventilation
(e) Digoxin
5. Macrocytosis is seen in all of the following disorders except:
(NEL 17th/1612)(NEL/1379) (UP 97)
(a) Hypothyroidism
(b) Thalassaemia major
(c) Folic acid deficiency
(d) B12 deficiency
6. Megaloblastic anemia secondary to folate deficiency is best corrected by administering folate:
(OPG5th/303) (JIPMER 81, AMU 87)
(a) Orally 2 mg daily
(b) IM 10 mg daily
(c) IV 10 mg daily
(d) 1 gm IM
Megaloblastic Anemia:
Most common in infants feeding on Goats milk (poor source of folate)
Predisposing factors are- Chronic diarrhoea
- Malabsorption
- Recurrent Infections
Patient is pale; irritable; fail to thrive, Increased pigmentation; tremors adn developmental
retrogression is also seen.
Nutritional megaloblastic anemia is unsual after 3-4 yr.
Lab findings are- Macrocytosis (MCV>100fL)
- Polymorphonuclear hypersegmentation
- Fenestrated pattern of chromatin
- Decreased megakaryocytes in bone marrow.
Dose of folic acid to be administred is 2-5 mg/day.
Hematological Disorders:
Transtition from the proerythroblast to the most nature normoblast involves 4 cell divisions over
a 4-day period.
Reticulocyte stays in bone marrow for 2.5-3 days and for another 24 hr in general circulation before
assuming morphologic appearance of a mature RBC.
During the last 3 months of gestation, gamma chain synthesis switches to chain synthesis.
Normal infant has 250-500 gm of iron at the time of birth.
- The level of free haptoglobin decreases.
Plasma hemopexin decreases

3 D

4 B

5 B

6 A

57 / Pediatric Buster

In intravascular hemolysis there is hemoglobinemia, hemoglobinuria methemalbuminemia and


hemosiderinuria.
Rise in serum LDH and carboxyhemoglobin
Increased level of indirect bilirubin and urobilinogen in urine (not very sensitive)
Increased reticulocyte is indirect indicator of hemolysis denoting compensatory erythropoiesis.
Hemolytic Anemias:
Peripheral Blood smear show changes in red cell morphology. Presence of fragmented RBCs
indicates intravascular hemolysis.
- Spherocytes
:
Hereditary spherocytosis, autoimonune hemolytic anemia
- Target cells
:
Thalassemia, HbC, HbS.
- Schistocytes
:
Thalassemia, HbC, HbS.
- Sickle cells
:
Sickle cell anemia
- Acanthocytes
:
Sever Liver disease (Spur cell anemia)
- Burr cells
:
HUS; TTP
- Heinz bodies
:
Unstable hemoglobin G6PD deficiency
7. Splenectomy is usually indicated in children with:
(a) Glucose-6- phosphate dehydrogenase deficiency
(d) Myeloid metaplasia.

(b) Thalassaemia major

(OPG5th/308) (AI 91)


(c) Spherocytosis

Hereditary Spherocytosis:
Autosomal dominant disorder where spectrin is deficient.
Characterized by anemia; splenomegaly and jaundice.
Hepatosplenomegaly indicates extramedullary hematopoiesis.
Jaundice may be intermittent and tends to be less pronounced in childhood.
Pigment gallstones are common even in childhood.
Chronic leg ulcer is infrequent.
Parvovirus crisis is sometimes encountered.
MCH is usually N or slightly decreased but MCHC is high.
Osmotic fragility is increased.
Coombs test in negative (If +ve suggests immune hemolytic anemia)
Splenectomy inproves the picture but deferred until the child is 4-5 yr.
8. Which of the following is not characteristic of thalassaemia major?
(OPG5th/309) (AIIMS 83)
(a) Hypochromic-microcytic anaemia
(b) Splenomegaly
(c) Target cells
(d) Decreased serum iron
9. In beta thalassemia, there is:
(OPG5th/309) (AIIMS/MAY /01)
(a) Increase in beta chain, decrease in alpha chain
(b) Decrease in beta chain, increase in alpha chain
(c) Decrease in beta chain, decrease in alpha chain
(d) Increase in beta chain, increase in alpha chain
10. A child presented with anemia, which was microcytic hypochromic, and hepatosplenomegaly with target cell.
There was a history of similar illness in the family the best investigation to establish the diagnosis would be:
(OPG5th/310) (AIIMS/NOV/01)
(a) G6PD assay
(b) Electrophoresis
(c) Bone marrow examination
(d) Coombs test
11. A 5-year-old girl came with history of progressively increasing pallor since birth and hepatosplenomegaly. Which
of the folowing is the most relevant test for achieving diagnosis:
(OPG5th/310) (AI/04)
(a) Hb electophoresis
(b) Peripheral smear examination
(c) Osmotic fragility test
(d) Bone marrow
examination.
Beta-thalessemia Major:
Also called cooleys or Littoral anemia,
Most severe form of congenital hemolytic anemia.
Manifests after first 4-6 months of life as failure to thrive
Severe anemia
7 C

8 D

9 B

10 B

11 A

Hamatological Diseases / 58

Skeletal abnormalities due to expansion of erythroid marrow (chipmunk facies or malocclusion


of jaw)
There is hepatosplenomegaly; bossing of skull; prominent frontal and parietal eminences with
flattened vault and hypertrophy of maxilla.
Patients have a peculiar skin color owing to increased melanin deposition.
Recurrent infections are also there.
HbA is reduced with compensatory increase in HbF and HbA2.
RBC counts are low; reticulocyte count is elevated; hamatocrit is reduced and MCV, MCH or MCHC
are low.
Marked basophilic stippling and variable number of target cells.
Osmotic fragility is decreased.
Serum iron level is high but free erythrocyte porphyrin level in N.
There is a peculiar association with hypertriglyceridemia.
Earliest bony changes occur in small bones of hand with a rectangular appearance.
Diploic spaces in the skull are widened.
Hair on end appearance in X-ray of skull.
Prenatal diagnosis is possible.

Beta-thalessemia Minor
Heterozygotes for thalassemia are protected from malaria.
Resembles more or less a iron deficiency anemia.
A characteristic finding is increase in HbA2 (4-8% of Total Hb) with normal levels of HbF.
(Alpha-thalassemia HbH)
Four beta peptide chains polymerize to a tetrameric form forming HbH.
Alpha-thalessemia is most prevalent in the countries of south-east Asia.
No hepatosplenomegaly and if present appears later.
Iron deficiency is severe because HbH is not bound to haptoglobin and lost in urine.
Treatment of Thalassemias:
1. Blood transfusion (Hypertransfusion protocal to keep Hb between 10.5-11gm%) not to rise above
16 gm% and fall below 10.5 gm.
2. Chelation by desferrioxamine (Cataracts are reported on long term use).
3. Spleenectomy (in cases with hypersplenism)
4. Bone marrow transfusion
5. Pharmacological upgrading of HbF production by 5-azacytidine or hydroxyurea.
6. Gene therapy.
12. Ideally children with thalassaemia should be transfused with:
(OPG5th/312) (AIIMS 83)
(a) Packed RBC
(b) Platelet rich plasma
(c) Washed RBC
(d) Blood of polycythaemia
13. The most appropriate drug used for chelation therapy in beta thalessemia major is:
(OPG5th/312) (AI/03)
(a) Oral desferrioxamine
(b) Oral deferiprone
(c) Intramuscular EDTA
(d) Oral succimer
14. Spleen is palpable in all haemolytic anemias in a child of 8 years of age, except in: (OPG5th/314)(UPSC/02)
(a) Thalassemia
(b) Spherocytosis
(c) Sickle-cell anemia
(d) Haemolysis in malaria
Sickel cell anemia:
It is substitution of valine for glutamic acid at the sixth position of b chain produces HbS.
Common in black Africans.
Tendency of sickling depends on the amount of HbS and its interaction with other hemoglobin chain
in the cell.
Sickling increases with
1. HbS > HbC > HbD > HbF
2. Dehydration and increased MCHC
3. Fall in pH.
12 A

13 A

14 C

59 / Pediatric Buster

Coexistence of thalassemia lessens the severity.


Clinical manifestation pertains to severe, anemia; vaso-occlusive complications and predisposition
to infection and chronic hyperbilirubinemia.
Physical growth is retarded; Most patients have clubbing of fingers; heart is enlarged and there may
be CHF with systolic ejection murmur.
Liver is enlarged and spleen is palpable.
Fish mouth vertebra is pathognomic.
There is aseptic necrosis of head of femur, dactylitis and chronic osteomyelitis (Salmonella).
Ankle ulcers, pigment gallstones, retinal infarcts, retinal detachment with sudden monoccular
blindness is also seen.
Renal concentration ability is loss; more prone to papillary necrosis.
Emotional disturbance and neurological deficits, cerebral thrombosis and hemiplegia is also seen.
Anemia and rheumatism in infancy is often suggestive of sickle cell anemia.
ESR is not affected and sickling can be demonstrated by metabisulphide test.
Pathological finding are
- Fatty change is heart; liver and tubules of kidney
- Hyperplastic bone marrow
- Crew hair cut in X-ray skull
- Gamma gandy bodies and autosplenectomy,
Hemoglobin E Disease
- Glutamic acid is replaced by lysine
- Nearly normal O2 binding
- HbC is not associated with anemia.
- But there is mild hypochromias
- Osmotic fragility is decreased.
- Characterized by striking target cells.
15. A nine-month-old boy of Sindhi parents presented to you with complaints of progressive lethargy, irritability and
pallor since 6 months of age. Examination revealed severe pallor. Investigations showed Hb- 3.8 gm%; MCV-58
fl; MCH- 19.4 pg/cell. Blood film shows osmatic fragility is normal (target cells and normoblasts). X-ray skull
shows expansion of erythroid marrow. Which of the following is the most likely diagnosis? (OPG5th/315)(AI/04)
(a) Iron deficiency anemia
(b) Acute lymphoblastic anemia
(c) Hemoglobin D disease
(d) Hereditary
spherocytosis.
Hemophilia:
Congenital deficiency of factor VIII (hemophilia A)
Inheritance is X-linked recessive.
Hemophilia is characterized by bleeding into soft tissues, muscles and weight bearing joints (knee
joint is commonest).
Clinical manifestation depends on the severity of the illness.
<1%
Severe (Spontaneous bleeding)
1-5%
Moderate
>5%
Mild (Prolonged bleeding following severe injury)
Partial thromboplastin time is prolonged.
Treatment is cryoprecipitate.
16. Haemophilic A have following diagnostic features except:
(a) Decrease VIII factor
(b) Increase PTT
(c) Increase PT

(OPG5th/327)(AI 97)
(d) BT is normal

Bleeding Time Increased


1. Platelet count decreased: Idiopathic thrombocytopenic purpura; Aplasia; leukemia
2. Platelet count Normal: Anaphylactoid purpra
15 C

16 C

60 / Pediatric Buster
Clotting Time Increased:
Only PT prolongedFactor VII or fibrinogen deficiency
Only PTT prolongedCongenital deficiency of first stage factors, i.e. XII, XI, IX and VIII and von
Willebrands disease
Both PT and activated PTT prolonged-vit K deficiency, severe liver disease and congenital deficiency
of 2nd stage factor; X and V.
17. Aminocaproic acid would be recommended for a haemophiliac child with:
(OPG5th/328) (PGI 79, AIIMS 81)
(a) Epistaxis
(b) Haematuria
(c) Oral bleeding
(d) Haemarthrosis
18. Treatment of neonatal idiopathic thrombocytopenia is:
(NEL17th/1671) (AIIMS 91)
(a) Immunoglobulins
(b) Pulse methylprednisolone
(c) Dexamethasone
(d) Platelets transfusion
19. Most frequent cause of neonatal thrombocytopenic purpura is:
(NEL 17th/1672) (PGI 80, AIIMS 82)
(a) Infection
(b) Erythroblastosis
(c) Drug idiosyncrasy
(d) Large haemangiomas
20. True about idiopathic thrombocytopenic purpura is A/E:
(NEL 17th/1670) (AIIMS/99)
(a) Joint haemorrhage is often seen (b) Increase megakaryocytes in bone marrow (c) Most common in children
(d) Males commonly affected in chronic ITP
Idiopathic Thrombocytopenic purpura:
Characterized by petechial hemorrhages; thrombocytopenia; normal or increased no. of mega
kacyocytes in bone marrow.
There is platelet associated immunoglobulin G (GAIgG).
The disease is due to interaction of platelets and immune complex formed during antibody response
to a viral infection, especially of respiratory tract.
- There may even be neonatal thrombocytopenia.
- Peak age of incidence is 2-8 yr.
- hematemesis, melena and bleeding in joints are unusual.
- Anemia is proportionate to degree of bleeding.
- Spleen is often not palpable.
- Bleeding time prolonged.
- Capilllary fragility test +ve
- Test of coagulation factors normal.
- Short course of prednisolone is effective in reducing bleeding even before platelet count returns
to normal.
- High does I/V lg and immuno suppresants also used. (Azathioprine improves platelet count in
HIV-induced thrombocytopenia)
- Splenectomy is done as last resort.
- All children should receive meningococcal, H. influenzae and pneumococcal vaccine 3 weeks prior
to splenectomy.
- Colchicine and danazol are also being tried in chronic ITP.
- Treatment of chronic ITP is based on symptomatology rather than on the platelet count.
Other Causes of Purpura
- HUS
- TTP
- DIC
- Aplastic anemia
- Cytotoxic therapy
- Infections
- Giant hemangiomas
- Wiskot Aldrich syndrome.
- Anaphylactoid purpura
- Thromboesthenia (Normal number of platelets with defective function)

17 C

18 A

19 A

20 A

Diseases of Ear, Nose, Throat


and Respiratory System

Breathing movements are identified as early as 11 wk but are irregular till 20th wk.
By 24th wk the surfactant precussor appears as inclusion bodies in alveolar lining cells.
Intrapleural negative pressure required for the first breath is 40-100 cm of H20
The normal newborn requires about 7 ml of oxygen/min/kg.
Commonest cause of stridor in newborn is laryngomalacia also caused by cystic hygroma and
avascular rings.
Epiglottitis is caused by Haemophilus influenzae B.
Infections croup (croup syndrome) is usually caused by parainfluenzae type 1.
Bacterial superinfections are unusual.
Spasmodic croup occurs between 1-3 yr.
Antibiotics help epiglottitis but not croup as it viral in origin.
Otitis media in children < 6 yr Hemophilus influenzae > 6 yr is pneumococci
Salicylates are responsible for dyspnea in childhood.

1. In a 4-year-old child with ASOM, the infecting organism is likely to be:


(OPG5th/331) (AI 89)
(a) Pneumococcus
(b) H. influenzae
(c) Streptococcus
(d) Staphylococcus
2. Recognised cause of stridor in newborn:
(OPG5th/339) (PGI 79, AIIMSA 80)
(a) Cystic hygroma
(b) A vascular ring
(c) Laryngomalacia
(d) All
3. Croup syndrome is usually caused by:
(OPG5th/339) (PGI 80, AIIMS 83)
(a) Rhinoviruses
(b) Coxsackie A virus
(c) Coxsackie B virus
(d) Parainfluenza
4. The most common cause of stridor in a newborn is:
(OPG5th/339)
(DELHI 81, 92)
(a) Laryngomalacia
(b) Foreign body
(c) Meconium aspiration
(d) Recurrent laryngeal nerve palsy due
to birth injuries
Laryngomalacia:
Manifest by end of the first week or during the second week after birth.
Stridor is intermittent; aggravated by crying or feeding.
Respiratory distress and chest retraction is absent.
Disappears spontaneously by the age of 6 months - 1 yr.
5. Croup is associated with A/E:
(a) RSV
(b) Influenza virus
(c) Adenovirus
(d) Streptococcus
6. Combination chemotherapy is not indicative in:
(a) Primary complex
(b) Acute epiglottis
(c) Laryngotracheobronchitis
patients
1 B

2 B

3 D

4 A

5 D

6 C

(OPG5th/339) (UP 98)


(OPG5th/340) (JIPMER 95)
(d) Immunological suppressed

Diseases of Ear, Nose, Throat and Respiratory System / 62


7. Ten days old child presents with intermittent stridor todays. Most likely cause is:
(OPG5th/340) (AI/2001)
(a) Laryngotracheobronchitis
(b) Laryngomalacia
(c) Respiratory obstruction
(d) Bronchiolitis
8. 4-year-old child has a seal barking like croupy cough. Management includes all except: (OPG5th/340) (PGI 2K)
(a) Oxygen inhalation
(b) Antibiotics
(c) Hydration
(d) Morphine
9. A 4-year-old child presents with a history of hoarseness croup cough and aphonia. The child has dyspnoea with
wheezing. The most probable diagnosis is:
(OPG5th/341) (JIPMER 80, PGI 82)
(a) Asthmatic bronchitis
(b) Laryngeal foreign body
(c) Bronchopneumonia
(d) Retropharyngeal abscess
10. Foreign body inhaled in infant next management is:
(OPG5th/341) (AI 97)
(a) Bronchoscopy
(b) I.P.P.V and incubation
(c) Steroid
(d) Tracheostomy
Pneumonia: Acute onset with high fever, dyspnea, grunting respiration with R.R. always > 60/min
Apical pneumonia may sometimes be associated with meningismus and convulsions.
Pneumococcal pneumoia is less common in infants under 1 yr. of age; pencilillin C is drug of choice.
Staphylococcal pneumonia occurs in infancy usually after epidemics of influenza and complication
of measles and cystic fibrosis. Pneumatoceles are pathognomic. But also seen in Klebsiella.
Empyema in child below 2 yr is nearly always of staphyloccocal origin. The illness usually follows
URTI or pyoderma.
Abdomen is usually distended due to septicemia and ileus.
Complication pyo-pneumothorax and pericarditis.
Pneumonia in immunosuppressive patient with AIDS is due to Pneumocystis caranii with evidence
of B/L diffuse disease. X-ray shows symmetrical perihilar ground glass appearance. Trimethoprin
and pentamidine are useful.
11. Most indicative sign of pneumonia in one-year-old child with cough and fever is: (OPG5th/348) (JIPMER 95)
(a) Intercostal retraction and fever
(b) Grunting and respiratory rate more than 60
(c) Temperature 39.
5 and 60 respiratory rate
(d) None
12. In staphylococcal pneumonia, acute respiratory distress is due to:
(OPG5th/349) (TN 89)
(a) Pneumatocele formation
(b) Tension pneumothorax
(c) Pleural effusion
(d) Spreading pneumonia
13. A 2-year-old child presented with history of difficult breathing, respiratory rate 50/m and bilateral crepts but
there is no chest retraction, the diagnosis will be:
(OPG5th/352) (AIIMS/2000)
(a) No pneumonia
(b) Not severe pneumonia
(c) Severe pneumonia
(d) Very severe pneumonia
14. A 2-year-old child present with fever (38.5C ) and respiration rate 50 per minute without chest retraction. He
is suffering from:
(OPG5th/352) (ALL INDIA/02)
(a) Very severe pneumonia
(b) Severe pneumonia
(c) Pneumonia
(d) Simple fever
15. A child aged 24 months was brought to the Primary health centre with complaints of cough and fever for the past
2 days. On examination, the child weighed 11 kg, respiratory rate was 38/min, chest indrawing was present. The
most appropriate management for this patient is :
(OPG5th/352) (AI/03)
(a) Classify as pneumonia and refer urgently to secondary level hospital
(b) Classify as pneumonia, start
antibiotics and advise to report after 2 days.
(c) Classify as severe pneumonia, start antibiotics and refer
urgently
(d) Classify as severe pneumonia and refer urgently.
16. A 3-year-old male child is referred from primary health centre to the paediatric emergency with the diagnosis of very
severe pneumonia. The following criteria are essential for diagnosis of very severe pneumonia as per ARI programme
except:
(OPG5th/352)(UPSC/03)
(a) Respiratory rate 48/minute
(b) Central cyanosis
(c) Inability to feed
(d) Extensive crepitations on
chest auscultation
Acute respiratory infections in children
Children aged 2 months to 5 years with cough or difficult breathing; clinical classification to facilitate
treatment decision.

7 B

8 D

9 B

10 A

11 B

12 A

13 B

14 C

15 C

16 D

63 / Pediatric Buster
Clinical category
Very severe pneumonia

Severe pneumonia
Pneumonia

Cough or cold

Essential feature
Central cyanosis, or not able
to breastfeed or drink, or convulsions, or
lethargy or unconsciousness, or severe
respiratory distress (e.g. head nodding)
Lower chest indrawing or nasal flaring, and
no signs of very severe pneumonia
Fast breathing, i.e. Age RR/min 2 m up to
12 m 50 12m up to 5y 40 and
No indicators of severe or very severe
pneumonia
No fast breathing, and no indicators of
severe or very severe pneumonia

Treatment strategy
Inpatient care IM or IV,
chloramphenicol or ampicillin plus
gentamicin; give oxygen
Inpatient care IM or IV benzyl
penicillin or ampicillin
Home care; oral cotrimoxazole or
amoxycillin or ampicillin or IM
procaine penicillin
Home care; home remedy for cough;
paracetamol for fever

Signs indicating need for admission in children who have pneumonia


Central cyanosis
History of convulsions
Lethargy or unconsciousness
Severe respiratory distress (e.g. head nodding)
Lower chest indrawing
Nasal flaring
Stridor in a calm child
Severe dehydration or shock
Severe malnutrition or severe anemia: visible severe wasting, severe palmar pallor or edema of both
feet.
Clinical classification of sick young infants aged 0 to 2 months
Not able to drink or breastfeed
Clinical category
Possible serious bacterial
infection; pneumonia,
septicemia or meningitis

Local bacterial infection


No evidence of possible bacterial infection

Essential feature
Presence of any of the following in a young
infant, thought to be sick by the mother
Convulsions
Bulging fontanelle
Lethargic or unconscious
Less than normal movement
Fever (axillary temperature > 37.5C) or low
body temperature (axillary temperature < 35.5C)
Many or severe skin pustules
Umbilical redness extending to the skin
Fast breathing (respiratory rate 60 per minute)
Severe chest indrawing
Nasal flaring
Respiratory grunting
Cough and severe undernutrition (weight < 2.0 kg in
the first month, or presence of visible severe wasting)
Not able to feed at all, or no attachment with breast
at all, or no sucking at all
Red umbilicus or umbilicus draining pus, or
Skin pustules
No features suggestive of possible serious bacterial
infection or local bacterial infection

Diseases of Ear, Nose, Throat and Respiratory System / 64


The indications for oxygen therapy in infants and children with ALRI are;
Central cyanosis
Inability to drink or breastfeed
Severe lower chest indrawing head nodding
Grunting with every breath (in young infants)
Respiratory rate of 70 breaths per minute or more
17. Acute bronchiolitis is commonly caused by:
(a) Respiratory syncytial virus
(b) Adenovirus
(c) Para-influenza virus type 3
18. In a child the most common cause of respiratory infection with wheeze is:
(a) RSV
(b) Influeza virus
(c) Adenovirus
(d) Para-influenza

(OPG5th/352) (UP 97)


(d) Mycoplasma
(OPG5th/353) (PGI 95)

Bronchiolitis:
Recognized cause of wheezing in infants below the age of 2 yr
Usually occur in winter or spring and caused by respiratory syncytial virus.
It is a self limiting disease.
25% cases may progress to bronchial asthma in later life.
Hypoxemia and respiratory acidosis; increase eosinophils in blood respiratory secretion is there.
Respiratory distress out of proportion to extent of disease.
Treatment is symptomatic with cold humidification oxygen.
Antibiotics are generally unnecessary.
Sedatives should not be used.
Ribavarin by aerosol is of immense help.
Best is steroids in case of emergency.
19. A 9-month-old infant presents with a 2-day history of fever, cough and breathlessness following an upper respiratory infection. She is febrile and has a respiratory rate of 80/min. intercostal and subcostal retractions and
extensive rhonchi on ascultation. A chest X-ray reveals a hyperinflated chest. The most likely diagnosis is:
(OPG5th/353) (UPSC/99)
(a) Bronchial asthama
(b) Foreign body aspiration
(c) Bacterial pneumonia
(d) Bronchiolitis
20. A child presents with fever, bilateral crepts and rhonchi, most likely diagnosis is infection with:
(a) Rhinovirus
(b) RSV
(c) Echovirus
(d) Measles
(OPG5th/353) (AIIMS/2K)
5th
21. Which of the following is/are true about bronchiolitis in children:
(OPG /353) (AIIMS /MAY/01)
(a) Caused by respiratory syncytial virus
(b) Hyperinflation of chest
(c) Pleural effusion
(d) May lead
on to bronchial asthma later in life
22. All of the following is true of bronchiolitis except:
(OPG5th/353) (PGI 88)
(a) Caused by RSV
(b) Occurs in school age
(c) Leads to asthma
(d) Treatment is oxygen
23. All of the following statements are true about bronchiolitis except:
(OPG5th/353) (AI 90)
(a) It is caused by respiratory syncytial virus
(b) It shows seasonal variation
(c) It needs antibiotic therapy
(d) It is common below 1 year of age
24. Following are used in treatment of bronchiolitis except:
(OPG5th/353) (AI 93)
(a) Humidified oxygen
(b) IV fluids
(c) Antiviral drugs
(d) Antibiotics
25. The treatment of bronchiolitis is:
(OPG5th/353)
(a) Humidified oxygen
(b) Antibiotics
(c) Steroids
(d) All of the above
26. In a child with exercise induced asthma, which is done:
(OPG5th/354) (JIPMER 80, DELHI 93)
(a) Prophylaxis with steroids
(b) Prophylaxis with beta agonist
(c) Prophylaxis
(d) Breathing exercise
27. Which is not a feature of child hood asthma:
(OPG5th/354) (AIIMS 94)
(a) Raised IgE levels
(b) History of atopic dermatitis
(c) Absence of wheezing after excercise
(d) Improves with age
28. All of the following are beneficial in the management of acute attacks of bronchial asthma in children except:
(a) High concentration of humidified oxygen
(b) Salbutamol nebuliser
(c) Intravenous hydrocortisone
(d) Sodium cromoglycate nebuliser
(OPG5th/359) (AI 91)
17 A

18 A

19 D

20 B

21 A

22 B

23 C

24 D

25 A

26 B

27 C

28 D

Cardiovascular System / 65
29. Treatment of asthma in children used in A/E:
(a) Steroids
(b) Adrenaline
(c) Aminophyllines

(OPG5th/359) (AI 92)


(d) Sedatives

Status asthmaticus:
Severe persistent dyspnea
Respiratory rate of over 30 per minute
Prolonged expiration
Pulse rate > 120/minute
Pulsus paradoxus
Cyanosis; silent chest, altered consciousness and pneumothorax
Treatment:
- Humidified O2
- Nebulized b2 agonist
- Corticosteroid
- I/V theophylline
- Avoid sedatives
1st drug for child with severe bronchospasm is s.c. adrenaline
Childhood asthma is characterized by increase lgE, history of atopic dermatitis and improvement
with age.

29 D

10

Cardiovascular System

Congenital heart disease:


Incidence is 6-8% per 1000 live births.
Accounts for 17 percent of all cardiac cases in OPD.
Basic development of heart takes place in first 16 wk of embryogenesis.
Cyanotic congenital heart disease are(5Ts)
- Tetralogy of Fallot
- Transposition of great arteries
- Truncus arteriosuspersistent
- Tricuspid atresia
- Taussing-Bing anomaly
Late or tardive cyanotic congenital heart diseases are:
- ASD
- VSD
- PDA
Right sided lesions are more common in females than left sided in males.
Strongest familial tendency is known in atrial septal defect associated with bony abnormalities
(Holt Oram syndrome)
Associated extracardiac anomalies are:
- Down syndrome
ASD of endocardial cushion type (ostium prium)
- Turner syndrome Coarctation of aorta
- Rubella syndrome PDA, VSD or pulmonary stenosis
- Marfans syndrome Regurgitation
1. The most common cyanotic heart disease manifesting as congestive cardiac failure during first week of life is:
(OPG5th/369) (AI 89)
(a) Pulmonary stenosis
(b) Fallots tetralogy
(c) Tricuspid atresia
(d) Hypoplastic left heart syndrome
2. Congenital heart disease which causes death in the first week of life:
(OPG5th/369) (AI 88)
(a) VSD
(b) TOF
(c) Epsteins anomaly
(d) Hypoplastic left ventricle
3. The most common cause of heart failure in infants is:
(OPG5th/369) (KERALA 91)
(a) Myocarditis
(b) Rheumatic fever
(c) Fluid over load
(d) Cardiomyopathy

1 D

2 D

3 A

67 / Pediatric Buster
Congestive cardiac failure:
M.C. causes in infants is myocarditis and primary myocardial disease.
Depending on time on onset.
- Birth - 72 hr

Atresias (mainly aortic)


- 4 days - 1 wk

Hypoplastic left and right hart syndrome


- 1 wk - 4 wk

Coarctation of aorta
- 1-2 mon

ASD; VSD; PDA


- 2-6 mon

Total anomalous pulmonary connection, aortic stenosis; coarctation


of aorta.
Arrhythmia is important cause of CCF in infants; other causes may be
- Obstructive lesion
- Infections
- Hypoglycemia; hypocalcemia and neonatal asphyxia
- CCF occurring without cardiac enlargement
- Congenital mitral stenosis
- Cor-triatriatum
- TAPVC (Obstructive)
- Symptoms of CCF
Poor weight gain
Difficulty in feeding
Breathes too fast and better when held
against the shoulder irritability;
excessive perspiration and restlessness.
Puffiness of face
Rarely oedal edema
Left sided failuretachypnea and tachycardia
Right sided failurehepatomegaly and facial puffiness
- Sign of CCF
Basal rales rarely audible
Examination of neck veins not helpful in small babies.
4. Congestive cardiac failure is diagnosed in an infant by:
(OPG5th/371) (AI 88)
(a) Basal crepts
(b) Elevated JVP
(c) Pedal oedema
(d) Liver enlargement
5. Excessive sweating in a young infant may be manifestation of:
(OPG5th/371) (AIIMS 79, 81)
(a) Polycythaemia
(b) Phenylketonuria
(c) Heart failure
(d) Anxiety
6. The most consistent early clinical sign of congestive cardiac failure in infants is:
(OPG5th/371) (AI 89)
(a) Oedema of feet
(b) Basal rates
(c) Hepatomegaly
(d) Raised jugular venous pressure.
7. Which of the following is not a characteristic of right sided failure?
(OPG5th/371) (JIPMER 80, PGI 83)
(a) Pulmonary oedema
(b) Ascites
(c) Oliguria
(d) Dependent oedema
8. Digitalis toxicity can be diagnosed if ECG shows:
(OPG5th/373) (AIIMS 83)
(a) Prolonged PR interval
(b) T wave inversion
(c) Shortening of QT interval
(d) Ventricular bigeminy
9. Compared to the oral digitalizing dose parenteral dose of digoxin should be:
(OPG5th/373) (AIIMS 81)
(a) 1/4
(b) 1/2
(c) 1/3
(d) 2/3
10. Carey Coombs murmur of rheumatic carditis is:
(OPG5th/376) (AIIMS 81 UPSC 89)
(a) An apical pansystolic murmur
(b) An apical mid diastolic murmur
(c) An apical early diastolic murmur
(d) A basal ejection systolic murmur
11. Essential feature of diagnosis of a acute rheumatic fever is:
(OPG5th/376) (AI 93)
(a) Recent sore throat infection
(b) Erythema marginatum
(c) 1 major and 2 minor Jones criteria
(d) Prior H/o of rheumatic fever
Acute Rheumatic fever:
An immunological disorders initiated by group A hemolytic Streptococcus
Incidence of rheumatic fever following steptococcal throat infection is 0.3%.
Age group involve is 5-15 years.
4 C

5 C

6 A

7 A

8 A

9 D

10 B

11 A

Cardiovascular System / 68

Mitral valve disease and chorea are more common in females whereas aortic valve involvement is
seen more in males.
- Major criteria are
- Carditis
- Arthritis
- Subcutaneous nodules
- Chorea
- Erythema marginatum
- Minor criteria are
- (Acute phase reactants) Fever
- Arthalgia precious episode
- Increased ESR and CRP
- Prololonged PR interval in ECG
- Essential criterion is
- Evidence of recent streptococcal infection as indicated by:
Increased ASO titer
Positive throat culture
Recent scarlet fever

12. Acute rheumatic fever is characterised by:


(OPG5th/376) (UP 94)
(a) Erythema marginatum
(b) Atypical rugling pansystolic murmur
(c) Tender nodule
(d) Haemoptysis
13. Major manifestations of rheumatic fever are A/E:
(OPG5th/376) (UP 97)
(a) Streptococcal antibody
(b) Polyarthritis
(c) Subcutaneous nodule
(d) Chorea
14. All of following are recognized manifestation of acute rheumatic fever except:
(OPG5th/376) (AIIMS 95)
(a) Abdominal pain
(b) Epistaxis
(c) Chorea
(d) Subcutaneous nodules

Carditis is usually pancarditis and occurs within first two weeks of the onset of rheumatic fever.
Myocarditis is charactrised bycardiac enlargement of first heart sound; protodiastolic (S3) gallop
congestive cardiac failure and a delayed diastolic Carey Coombs murmur.
Mitral regurgitation is the commonest manifestation of acute as well as previous rheumatic
carditis.
Rheumatic arthritis is a fleetin polyarthritis involving large joints and there is no residual damage
of the joint.
Patients who have subcutaneous nodules almost always have carditis. the nodules appear around
6 week on bony prominences and are nontender.
At time of chorea signs of inflammation as elevated ESR have returned to normal.
Erythema marginatum, an early manifestation, is predominantly seen on trunk.
Patient is treated with pencillin, aspirin or steroid along with diet control and rest.
Subcutaneous nodules tends to disappear faster with used of steroid as compared to aspirin.
Patient having carditis with CHF steroid use is must.
Carditis without CHFuse either steroid or aspirin but we almost always use steroid.
If patient does not have carditis it is preferable to use aspirin.
Aspirin and steroids are supposed not to have a place in the management of chorea.

15. The diagnosis of rheumatic fever is best confirmed by:


(OPG5th/378) (PGI 88)
(a) Throat swab culture
(b) Raised ESR
(c) ASLO titre
(d) ECG changes
16. Rheumatic fever is suggested by the presence of:
(OPG5th/378) (PGI 79, AMU 85)
(a) Symmetrical polyarthritis
(b) Nail clubbing
(c) Anemia
(d) ECG evidence of prolonged PR interval
17. An 8-year-old male child is admitted with a diagnosis of rheumatic fever with arthritis, carditis and congestive
heart failure. with reference to this case, consider the following as initial lines of management:
(OPG5th/379) (UPSC/03)
(1) Eradication of remnant streptococcal infection. (2) Administration of an anti-inflammatory drug.
(3) Institution of decongestive therapy (4) Institution of graded and gradually increasing exercise.
Which of these should actually consist of initial line of management?
(a) 1, 2 and 3
(b) 1, 2, 3 and 4
(c) 2, 3 and 4
(d) 1 and 4

12 A

13 A

14 B

15 C

16 D

17 A

69 / Pediatric Buster
18. Bacterial endocarditis is most commonly seen in:
(OPG5th/387) (UP 94)
(a) VSD
(b) PDA
(c) ASD
(d) AS
19. Infective endocarditis not seen in:
(OPG5th/387) (AI 94)
(a) ASD
(b) TOF
(c) VSD
(d) MR
20. A wide and fixed split second heart sound occurs in:
(OPG5th/400) (AIIMS 83)
(a) Mitral stenosis
(b) Atrial septal defects
(c) VSD
(d) Coarctation of aorta
21. Which one of the following does not produce cyanosis in the first year of life :
(OPG5th/400) (AIIMS/03)
(a) Artrial septal defect
(b) Hypoplastic left heart syndrome
(c) Truncus arteriosus
(d) Double outlet
right ventricle.
22. True about ASD:
(OPG5th/400)
(a) Patient foramen ovale
(b) Increase pulmonary artey flow leads to left parasternal heave
(c) S2 widen
and variable
(d) Systolic murmur due to rapid flow of blood
ASD (Atrial septal defect)
Most common congenital cardiac anomaly recognized in adults
More common in females
Ostium secondum type (90%) more common than primum type (5%); 5% is sinus venoustype.
Congestive cardiac failure is rare
There is wide and fixed split of second heart sound
Ejection systolic murmur is heard at pull area and a delayed diastolic nurmur at lower left sternal
border.
Rheumatic mitral stenosis with an ASD is called lutembachers syndrome
Infective endocarditis is rare because of low pressure gradient.
ECG of ostium secondum type have right axis deviation with right ventricular hypertrophy
Presence of left axis deviation beyond30 suggests the ostium primum type of atrial septal defect.
ASD is repaired in 3-6 yr.
23. VSD shunt reversal is called:
(OPG5th/402) (AIIMS 89)
(a) Eisenmengers syndrome
(b) Eisenmengers complex
(c) Ebsteins anomaly
(d) None of the above
24. Natural course of events in untreated ventricular septal defects except:
(OPG5th/403) (PGI 81, DELHI 85)
(a) Spontaneous closure of defect (b) Development of pulmonary insufficiency (c) Subacute bacterial endocarditis
(d) A normal life without symptoms
25. A young boy had developed congestive failure and was found to have membranous VSD. He Spontaneously showed
improvement in his condition. The is most likely to be due to:
(OPG5th/403) (AIIMS/NOV/01)
(a) Perimembranous closure of VSD
(b) Development of AR
(c) Pulmonary vascular changes
(d) Infective
endocarditis
Ventricular Septal Defect:
Most common congenital cardiac lesion
90% are located in membranous part of ventricular septum
There is pansystolic murmur at 3rd or 4th ICS and a delayed diastolic murmur starting with 3rd
sound at apex.
Patient are born with right, ventricular hypertophy
Patient may have hyperkinetic pulmonary arterial hypertension
May develop CHF in infancy
50% close spontaneously
M.C. congenital lesion complicate by infective endocarditis
Small defects less than 0.5 cm in diameter are known as Rogers disease
Operative treatment consists closure of the VSD with use of a dacron patch.
26. Differential cyanosis and clubbing is seen in:
(a) PDA and reversal of shunt
(b) TOF
(c) VSD + ASD
18 A

19 A

20 B

21 A

22 B

23 B

24 D

25 C

(OPG5th/405) (UP 94)


(d) MS + MR
26 A

Cardiovascular System / 70
27. Pure left sided failure may be seen with:
(OPG5th405) (AIIMS 78, PGI 81)
(a) ASD
(b) Aortic atresia
(c) Patent ductus arteriosus
(d) Pulmonary valvular obstruction
28. One-year-old child with PDA; which is true:
(OPG5th/405) (AIIMS 95)
(a) Symptoms similar to artopulmonary window
(b) Chances of spontaneous closure high
(c) Indomethacin
may help in closure
(d) Indocarditis is rare
29. Preterm infant with PDA is given...to effect closure of the PDA:
(OPG5th/406) (AP 88)
(a) Corticosteroids
(b) Indomethacin
(c) Aspirin
(d) Hyperbaric oxygen
30. The great danger of patients with patent ductus arteriosus is:
(OPG5th/406) (JIPMER 80)
(a) Syncope
(b) Convulsions
(c) Arrhythmia
(d) Bacterial endocarditis
Patient Ductus Arteriosus:
Communication between the pulmonary artery and the aorta just distal to left subclavian artery.
It closes functionally and anatomically soon after birth.
There is continuous harsh murmur and loud S1.
Wide pulse pressure is found along with left ventricular hypertrophy and pulmonary artery
dilatation.
The cyonosis affect the toes and lower extremities, but not the fingers.
Indomethacin, a prostaglandin synthetase inhibitor can be given orally to close the ductus.
The risk of operation increases after age of 15 yr.
Patients are considered inoperable if a right to left shunt has appeared because of pulmonary
arterial hypertension.
31. All are characteristic of Fallots tetralogy except:
(OPG5th/408) (PGI 85)
(a) Infundibular stenosis
(b) VSD
(c) Overriding aorta
(d) Left ventricular hypertrophy
32. Which one of the following congenital heart disease has cyanosis without cardiomegaly and /or congestive heart
failure?
(OPG5th/408) (UPSC 98)
(a) Transportation of great arteries
(b) Fallots tetralogy
(c) Congenital mitral regurgitation
(d) Congenital pulmonary stenosis
33. Anoxic spells in tetralogy of Fallots are precipitated by:
(OPG5th/408) (PGI/90)
(a) Fever
(b) Exertion
(c) Crying on feeding
(d) All
34. The congenital heart disease that has least chance of heart failure in infants:
(OPG5th/408) (AI 93)
(a) TOF
(b) VSD
(c) TAPVC
(d) ASD
35. Cyanosis and breathlessness in a child is seen in:
(OPG5th/408) (AIIMS 89)
(a) Transposition of great vessels
(b) Tetralogy of Fallot
(c) Ventricular septal defect
(d) Atrial septal
defect
36. Which of the following conditions does not present with CHF in neonate:
(OPG5th/408) (AIIMS 92)
(a) Total anomalous pulmonary venous connection
(b) Coarctation of aorta
(c) Tetralogy of Fallot
(d) Transposition of great vessels
37. In TOF:
(OPG5th/408) (AIIMS 98)
(a) Cynotic spells are due to arrhythmias
(b) CXR and ECG are typically normal
(c) Wide split 2nd HS
on inspiration
(d) Central cyanosis with clubbing
Tetralogy of Fallot:
Commonest cyanotic congenital heart disease in children above 2 yr.
It consists of ventricular septal defect.
- Infundibular pulmonic stenosis
- Overriding or dextroposed aorta
- Right ventricular hypertrophy
The more severe the pulmonic stenosis, the shorter the ejection systolic murmur and the more the
cyanosis.
Congestive failure never occurs in TOF except when complications are present.
Right ventricular hypertrophy is concentric without cardiac enlargement.
27 C

28 A

29 B

30 D

31 D

32 B

33 D

34 A

35 B

36 C

37 D

71 / Pediatric Buster

Patient may present with syncope; anoxic spells (paroxysmal attacks of dyspnea).
Cyanosis may present from birth.
Commonest symptoms are dysnea on exertion and excercise intolerance.
Patient assumes squatting posture on dysneic feeling.
TOF is commonest congenital lesion where squatting is noted.
Patient have cyanosis; clubbing and slightly prominent a waves.
X-ray shows boot shaped heart with upturned apex.
On X-ray there is absence of main pulmonary artery segment (coren sabot). Lung fields are oligemic.
Patient are prone to infective endocarditis; thrombotic diasthesis, paradoxical embolism and
cerebral abscess.
Operation performed are:
Blalock Taussing shunt
Potts shunt
Watersons shunt

38. A blue infant was found to have oligemic lung fields with normal sized heart. The diagnosis is:
(OPG5th/408) (AIIMS/NOV/01)
(a) Transposition of great vessels
(b) Tricuspid atresia
(c) TOF
(d) Pulmonary stenosis
39. Which one of the following is the most common cause of cyanotic congential heart disease?
(OPG5th/408) (UPSC/01)
(a) Dextrocardia
(b) Fallots tetralogy
(c) Atrial septal defect
(d) Coarctation of aorta
40. Pulmonary plethora is seen in all except:
(OPG5th/408) (PGI/89)
(a) VSD
(b) ASD
(c) Fallots tetralogy
(d) PDA
41. Blalock Taussings operation involves:
(OPG5th/409) (AIIMS 92)
(a) Right pulmonary artery with aorta
(b) Left pulmonary artery with ipsilateral subclavian A
(c) Right
pulmonary artery with descending aorta
(d) Left pulmonary artery with ascending aorta
42. Potts shunt is running from:
(OPG5th/409) (AI/2001)
(a) Right subclavian artery to right pulmonary
(b) Descending aorta to left pulmonary
(c) Left subclavian
to left pulmonary
(d) Ascending aorta to right pulmonary
43. Tricuspid atresia all are true except:
(UP/2000) (OPG5th/410) (AIIMS 98)
(a) R.V. hypoplasia
(b) Left axis deviation
(c) Split S2 in inspiration
(d) Pulmonary oligaemia
44. A child with central cyanosis and enlarged left ventricle the probable diagnosis is:
(OPG5th/410) (AI 91)
(a) Tricuspid atresia (b) Eiesenmengers syndrome (c) Tetralogy of Fallot (d) Anomalous pulmonary artery
45. Left axis deviation with left ventricular hypertrophy is seen:
(OPG5th/410) (UP 96)
(a) Tricuspid atresia
(b) Tetralogy of Fallot
(c) Coarctation of aorta
(d) Ventricular septal defect
Tricuspid atresia:
Congenital absence of tricuspid valve
Right venticle hypolastic and pulmonary-stenosis
Patent foramen ovale or atrial septal defect
ECG shows left axis deviation and left ventricular hypertrophy
X-ray shows left ventricular cofiguration
Patient are cyahotic from birth
Anoxic spells and squatting may be present
Fontan operation is done at 4 yr. or more (Atriopulmonary connection)
46. True of tricuspid atresia are A/E:
(OPG5th/410) (UP 97)
(a) Right axis deviation
(b) Left axis deviation
(c) Left ventricular hypertrophy
(d) Severe cyanosis
47. The best position for examination of cardiac murmurs in a child is:
(KERALA 90)
(a) Sitting
(b) Standing
(c) Right lateral
(d) Recumbent
48. Which of the following congenital cyanotic disease presents with gallop rhythm, left parasternal murmur, pericardial friction rub:
(OPG5th/411) (ORISSA/98)
(a) Ebstein anomaly
(b) TA PVC
(c) TGA
(d) Eissenmengers complex
38 C

39 B

40 C

41 B

42 B

43 C

44 A

45 A

46 A

47 D

48 A

Cardiovascular System / 72
Ebstein anomaly:
Diminished pulmonary blood flow resulting from an abnormality of the tricuspid valve.
Cyanosis; effort intolerance, fatigue and paroxysmal attacks of tachycardia
Clubbing is present.
Systolic thrill may be palpable at the left sternal border.
Triple or quadruple sounds heard along with a loud pansystolic murmur.
ECG shows P pulmonale and P mitrale with RBB
WPW type B conduction abnormality may be seen in ECG
There is right ventricular enlargement
Intracardiac ECG is diagnostic of Ebstein anomaly
49. In transposition of great vessels, all are true except:
(OPG5th/413) (AP 97)
(a) Aorta arises from the right ventricle
(b) Mitral valve is continuous with the aortic valve
(c) Causes
jaundice immediately after birth
(d) None of the above
50. Emergency treatment for TGV:
(OPG5th/413) (TN/99)
(a) Balloon septostomy
(b) Oxygen
(c) Ventilation
(d) Digoxin
51. Congestive heart failure, LVH and systolic heart murmur in a 29-day-old child would suggest:
(a) TGA
(b) VSD
(c) Rheumatic fever
(d) TOF
(OPG/5th413) (AIIMS/2K)
52. A neonate has central cyanosis and short systolic murmur on the 2nd day of birth. The diagnosis is:
(a) TGV
(b) TOF
(c) VSD
(d) ASD
(OPG/ 5th413) (AIIMS /MAY/01)
53. A five-day-old. Full term male infant was severely cyanotic at birth. Prostaglandin E was administered initially
and later balloon atrial septostomy was done which showed improvement in oxygenation. The most likely
diagnosis of this infant is:
(OPG5th/413)(AI/04)
(a) Tetralogy of Fallot
(b) Transposition of great vessels
(c) Truncus arteriosus
(d) Tricuspid atresia
Transposition of Great Vessels:
Particulary common in offespring of diabetic mother
Patent ductus is present
There is right ventricular hypertrophy
CHF around 4-10 week of age
On X-ray cardiac silhouette is like egg on side
Right upper lung fields are plethoric
Thymic shadow absent
Ballon septostomy helpful only upto 6-12 wk
< 2 wk-Jatenes switch operation
Other operation of complete TGA is mustard or senning operation
Taussing-Bing anomaly is a variant where aorta comes off the right ventricle and there is a VSD
directly beneath an overriding pulmonary artery.
Associated anomalies with complete TGA are VSD with or without PS, it sided ebstein anomaly
of TR and arterioventricular conduction abnormalities.
54. A neonate is found to have central cyanosis and a cardiac murmur at 30 hours of age. The most probable diagnosis
is.
(OPG5th/413)(UPSC/03)
(a) Endocardial cushion defect
(b) Ventricular septal defect
(c) Transposition of great vessels
(d) Patent
ductus arteriosus
55. True in total anomalous pulmonary connection are A/E:
(OPG5th/414) (AI 97)
(a) The total pulmonary venous blood reaching the right atrium
(b) Always associated with VSD
(c) The oxygen saturation of the blood in the pulmonary artery is higher to that in the aorta
(d) Infracardiac type is always obstructive
56. Figure of eight appearance is seen in:
(OPG5th/414) (AI/2000)
(a) Partial anomalous pulmonary vessels
(b) Total anomalous pulmonary vessels
(c) Truncus arteriosus
(d) Transposition of great vessels
49 C

50 A

51 A

52 A

53 B

54 C

55 B

56 B

73 / Pediatric Buster
57. Neonate has recurrent abdominal pain and diaphoresis on feeding, with murmur, which may lead to MI, diagnosis
is:
(OPG5th/414) (AI/2001)
(a) ASD
(b) VSD
(c) TOF
(d) Coronary anomalous pulmonany venous connection
TAPVC:
Nonobstructive type is more common than obstructive type.
ECG shows right axis deviation and right ventricular hypertrophy
In obstructive type P pulmonale is common.
X-ray showssnowman or figure of 8 configuration in the supracardiac TAPVC.
Obstructive type have ground glass appearance in the lungs.
58. Infantile hypercalcaemia syndrome is most frequently associated with: (OPG5th/416) (AIIMS 80, AIIMS 84)
(a) ASD
(b) VSD
(c) Patent ductus arteriosus
(d) Supravalvular aortic stenosis
59. Sequence of development of Eisenmengers syndrome:
(OPG5th/416) (AIIMS/2K)
(a) Left to right shuntright ventricular hypertrophyright to left shuntpulmonary hypertension
(b) Left
to right shuntpulmonary hypertensionright ventricular hypertrophyright to left shunt
(c) Left to right
shuntright ventricular hypertrophypulmonary hypertensionright to left shunt
(d) Right to left shunt
left ventricular hypertrophypulmonary hypertensionleft to right shunt
60. Paradoxically split second heart sound signifies severe:
(OPG5th/417) (AIIMS 83)
(a) Pulmonary stenosis
(b) Mitral stenosis
(c) Aortic stenosis
(d) Tricuspid stenosis
61. Most common cause of death from aortic stenosis in children is:
(OPG5th/418) (JIPMER 81, PGI 83)
(a) Myocardial infarction
(b) Pulmonary oedema
(c) Atrial flutter
(d) Ischaemic heart disease with
ventricular fibrillation
62. Rib notching of 4-9th ribs with double bulging is seen in:
(OPG5th/419) (AI 98)
(a) Aortic aneurysm
(b) Aortic dissection
(c) Coarctation of aorta
(d) Diaphragmatic hernia
Coarctation of Aorta
Located at junction of arch with the descending aorta distal to left subclavian artery near insertion
of ligamentum arteriosus.
Males are more affected; although females with Turners syndrome frequently have coarctation.
Accompanying anomalies may be PDA; bicuspid aortic valve; congenital aortic stenosis; ASD; VSD;
MR and Berry aneurysm of the circle of willis
Coarctation may be preductal or postductal
Only symptoms are intermittent claudication, pain and weakness of legs and dyspnea on running.
Femorals are impalpable w.r.t. strong brachial ones.
Barium swallow shows characteristic E. sign.
X-ray shows characteristic notching of the lower border of the ribs which tend to appear beyond the
age of 10 years
Deaths are mostly due to CHF; other causes may be intracranial hemorrhage; dissection of aorta,
hypertension, cerebral aneurysm, LVH, rupture aorta or I.E.
63. An infant with severe dehydration secondary to diarrhoea suddenly presents with protein and blood in urine. The
most probable diagnosis is:
(PGI 78, UPSC 89)
(a) Renal vein thrombosis
(b) Pyelonephritis
(c) Acute glomerulonephritis
(d) Lower nephrosis
64. Rib notching is present in:
(OPG5th/419) (UP96)
(a) Coarctation of aorta
(b) Ventricular septal defect
(c) Atrial septal defect
(d) Tetralogy of Fallot
65. Myocarditis can be caused by A/E:
(OPG5th/423) (PGI 88)
(a) Pertussis
(b) Measles
(c) Diphtheria
(d) Scorpion sting
66. Infantile myocarditis and pericarditis is due to:
(OPG5th/423) (TN/99)
(a) Coxsackie A
(b) Coxsackie B
(c) Mumps
(d) Pox virus
67. Sustained severe hypertension in children is commonly suggestive of:
(OPG5th/432) (AI 95, 97)
(a) Coractation of aorta
(b) Pheochromocytoma
(c) Renal parenchymal disease
(d) Drug induced
57 D

58 D

59 B

60 C

61 D

62 C

63 A

64 A

65 B

66 B

67 C

74 / Pediatric Buster
68. The average BP of a 1-year-old child is:
(OPG5th/432) (PGI 78, AMC 81)
(a) 120/80
(b) 75/50
(c) 95/65
(d) 60/30
69. The most common cause of systemic hypertension in children is:
(OPG5th/433) (AIIMS 89)
(a) Coarctation of aorta
(b) Acute glomerulonephritis
(c) Nephrotic syndrome
(d) Congenital adrenal hyperplasia
Hypertension in children:
Average pressure in children from 1-8 years in 95 5 systolic and 65 5 diastolic
Chronic renal disease especially chronic glomerulonephritis is most common cause of hypertension
in children.
2% had coartation and pheochromocytoma as cause of hypertension
Weight loss is an important finding in pheochromocytoma.
70. The most common cause of secondary hypertension in children is:
(OPG5th/433) (PGI
(a) Renal artery stenosis
(b) Renal disease
(c) Systemic vasculitis
(d) Adrenal tumours
71. The most common cause of hypertension in the newborn is:
(NEL17th/1593) (AIIMS
(a) Polycystic kidney
(b) Neuroblastoma
(c) Anaemia
(d) Coarctation of aorta
72. What is true of hydralazine therapy in hypertension:
(NEL 17th/1597) (PGI 80, AIIMS
(a) Drowsiness
(b) Lupus syndrome
(c) Positive Coombs test
(d) Syncope
73. The treatment for symptomatic sick sinus syndrome is:
(NEL 17th/1563) (AIIMS
(a) Atropine only
(b) Only bed rest
(c) Pacemaker implantation
(d) Propranolol only
74. The most common cause of painful pericarditis is:
(NEL17th/1580) (AIIMS 80 BIHAR
(a) Viral
(b) Tuberculous
(c) Uraemia
(d) All of these

68 C

69 B

70 A

71 B

72 B

73 C

74 A

93)
79)
87)
81)
91)

11

Kidney and Urinary Tract

Kidneys develop between 3-4 weeks of fetal age and by 22 wk medulla and cortex are clearly
demarcated.
Nephron induction ceases at about 26 wk of gestation; full complement of nephrons is present by
36 wk.
Urine formation is believed to start around 10th week of gestation.
GFR is about 1/3rd of that of adult, i.e. 30 ml/min in 1st wk of life;
2/3 rd of that of adult i.e. 60ml/min in 2nd-3rd month of life;
100-110 ml/min/1.73 m by the age of 2 yr;
Compared to adults, sodium reabsorption in the newborn is very low. Therefore pH of urine of a
newborn is inappropriately high for degree of acidemia.
A healthy infant excretes 15-30 ml/kg/24 hr of urine on the first two days of life and 25-120
ml/kg/24hr during the next 4 weeks.
Oliguria is defined as urine volume of less than 1 ml/kg/hr.
An infant can concentrate his urine to a maximum of 700-800 mOsm/kg whereas the older child
can achieve 1200-1400 mOsm/kg.
IVP- dose of contrast depends on the body wt.
Causes of mild proteinuria are
UTI
Hydronephrosis
Renal T.B.
Absencce of proteinuria does not rule out congenital anomalies of urinary tract.
Microscopic hematuria is found in:
- Idiopathic hypercalciuria.
- Benign familial haematuria
- IgA nephropathy; Alport syndrome
- Membranoproliferative glomerulonephritis.
Isolated proteinuria is found in heavy exercise; fever or dehydration
Asymptomatic proteinuria
- Orthostatic proteinuria
- Chronic glomerular disease
- Reflux nephropathy
- Renal hypoplasia
- Renal tubular disorders
Posterior urethral valves, occurring in males; are recognised cause of Potter facies.
U/L renal agenesis should be suspected in newborns with single umbilical artery.

Kidney and Urinary Tract / 76


1. Urine production begins at:
(OPG5th/441) (JIPMER 81, BIHAR 88)
(a) 6 weeks gestation
(b) 10 weeks gestation
(c) At term
(d) Soon after birth
2. The neonatal kidney achieves concentrating ability equivalent to adults kidney by:
(OPG5th/441) (AI/04)
(a) One year of age
(b) Eighteen months of age
(c) Three to six months of age
(d) Just before puberty.
3. An 8-month-old male baby is reffered for evaluation following a urinary tract infection. Ultrasound examination
of the abdomen is normal. The most appropriate investigation to evaluate his lower urinary tract is:
(OPG5th/444)(UPSC/03)
(a) Radionuclide cystogram
(b) Micturating cystourethrogram
(c) Intravenous pyelogram
(d) Cystoscopy
4. Which is incorrect about Hench-Schnlein purpura:
(OPG5th/448) (AIIMS 91)
(a) Arthritis
(b) Nephritis
(c) Thrombocytopenia
(d) Centrifugal rash
Hench-Schnlein Purpura (Anaphylactoid):
Self limited type of vasculitis usually in boys of 4-16 yr.
Syndrome may be preceded by upper respiratory infection or streptococcal pharyngitis or drug or food
allergies.
Vessel lesions contain IgA and complement deposits.
Patients develop purpuric or urticarial rash on extensor surface of arms of legs and on buttocks;
polyarthralgias or arthritis; colicky abdominal pain and hematuria.
Proteinuria and hypertension are also present due to focal GN.
Coagulation tests are normal.
Glucocorticoids benefit pain and arthralgia but not renal involvement.
5. The most common leukocytoclastic vasculitis affecting children is:
(OPG5th/448) (AIIMS/03)
(a) Takayasu disease
(b) Mucocutaneous lymph node syndrome (Kawasaki disease)
(c) Hench Schnlein
purpura
(d) Polyartertis nodosa.
6. True statement regarding SLE in children:
(OPG5th/449) (PGI 96)
(a) Skin pigmentation more common than adults
(b) No sex difference
(c) Renal involvement more common
(d) CNS involvement more common
7. Most common cause of hemolytic-uremic syndrome is:
(OPG5th/449) (AI 96)
(a) E. coli
(b) Shigella
(c) Salmonella
(d) Pseudomonas
8. Which is incorrect about haemolytic-uraemic syndrome:
(OPG5th/449) (JIPMER 91)
(a) Always fatal
(b) Burr cells are present
(c) Acute renal failure
(d) Viral prodrome
9. A 9-years-old child with H/o diarrhoea has hematuria, no RBC casts in urine. Blood examination shows
fragmented RBC, diagnosis is:
(OPG5th/449) (AI/2000)
(a) HUS
(b) TTP
(c) HSP
(d) Glomerulonephritis
Hemolytic-uraemic syndrome:
Characterised by Microangiopathic hemolytic anemia, thromocytopenia; acute renal insufficiency
and arthritis
Onset may be viral caused either by upper resp. infection or acute diarrohoea caused by verotoxic
E. coli or Shigella dysentery 1 in india.
A neutrophilic leucocytosis is usually present.
Kidneys often have flea bitten appearance (also in malignant HT).
Major site of pathology are the small renal arteries and afferent arterioles; marked intimal
hyperplasia and fibrin deposits in subintimal region.
Bilateral cortical necrosis may occur.
C.N.S. may be affected.
Thrombotic angiopathy (hyperkalemic thrombi) is seen with presence of Burr cell
Severe renal failure often marked by oliguria and hypertension
Coombs test is negative.
Barium contrast enema shows colonic spasm and transient filling defect.
The prognosis in children is better than that in adults.
1 B

2 B

3 B

4 C

5 C

6 C

7 A

8 C

9 A

77 / Pediatric Buster
10. A 10-year-old boy with hematuria-5 days, oliguria-1 day, BP 170/120, pedal edema present; possible diagnosis:
(a) Nephrotic disease
(b) AC GN
(c) HUS
(d) Renal vein thrombosis
(OPG5th/449) (PGI 2000)
11. Diarrhea-related hemolytic-uremic syndrome is characterized by all except:
(OPG5th/449)(UPSC/02)
(a) Acute renal failure
(b) Coombs positive hemolytic anemia
(c) Thrombocytopenia
(d) Neutrophilic
leukocytosis
12. In Shigella dysentery associated haemolytic-uraemic syndrome the false statement is:
(OPG5th/449)
(a) Leucocytosis
(b) Neurological abnormalities
(c) Hepatic failure
(d) Thrombotic angiopathy
13. In nephrotic syndrome the essential feature is:
(OPG5th/450) (AIIMS 83)
(a) Proteinuria
(b) Hypoalbuminaemia
(c) Hyperlipidaemia
(d) Oedema
Nephrotic Syndrome
Characterized by
Massive proteinuria
Hypoalbuminemia and edema
Also accompaniments are hyperlipidemia and hypertension.
Heavy proteinuria is the basic abnormality.
Hyperlipidemia is because of increased hepatic synthesis of beta lipoproteins and decreased
lipoprotein lipase activity.
During childhood, it is mostly idiopathic.
Minimal lesion (lipid nephrosis) accounts for 85 percent cases of nephrotic syndrome in children.
There is selective proteinuria of albumin but besides this urinary loss also accounts for loss of
- Throxine binding globulin
- Cholecalciferol binding protein leading to secondary hyperparathyroidism
- Transferrin (Iron resistant anemia)
- loss of metal binding protein (zinc and copper deficiency)
There is also hypercoagulable state due to deficiency of antithrombin III and reduced activity of
protein S or protein C and hyperfibrinogen.
Some have severe lgG deficiency.
Electrophoresis reveals, diminished albumin with increase of and globulins
14. Nephrotic syndrome in a child, all are seen except:
(OPG5th/450) (JIPMER)
(a) Hypoproteinaemia
(b) Hyperlipidaemia
(c) Uraemia
(d) Increased alpha globulins
15. Which of the following is not seen in nephrotic syndrome?
(OPG5th/450) (TN 95)
(a) Hematuria
(b) Massive proteinuria
(c) Hypoalbuminaemia
(d) Hypercholesterolaemia
16. The most common type of renal lesion in children is:
(OPG5th/450) (TN 95)
(a) Lipoid nephrosis
(b) Membrane proliferative glomerulonephritis
(c) Focal glomerulonephritis
(d) Diffuse glomerulosclerosis
17. A 5-year-old child is suffering from nephrotic syndrome is well responding to steroid therapy. What would be the
finding on light microscopy:
(OPG5th/451) (AI/2001)
(a) No finding
(b) Basement membrane thickening
(c) Podocyte lesions
(d) Fusion of foot process
18. Best response to steroids is observed with:
(OPG5th/451) (AI 96)
(a) Focal glomerulonephritis
(b) Lipoid nephrosis
(c) Membranous GN
(d) Membranoproliferative GN
Lipoid nephrosis:
Diffuse epithelial foot process effacement; serum complements are normal.
There is abnormality of T lymphocyte function with deposits of IgM and C3.
Besides anasarca; ascites; hydrothorax and hydrococele is also present.
Heavy proteinuria; hyaline and granular casts, low albumin with rise in 2 globulin and Normal
Blood urea and creatinine values are often encountered.
ARF is rare.
Peritonitis (Pneumococcal) is usually the complication of the NS.
Thromboembolic manifestation may occur but renal vein thrombosis is rare.
10 C

11 A

12 C

13 A

14 C

15 A

16 A

17 A

18 B

Kidney and Urinary Tract / 78

Spontaneous remission and relapse of heavy proteinuria occur in 30-40%.


Over 95% respond completely to 8 wk of prednisolone therapy.
Other agents used are
- Cyclophosphamide
- Chlorambucil
- Cyclosporin A.

19. Steroids are useful in:


(OPG5th/451) (AI 89)
(a) Poststreptococcal glomerulonephritis
(b) Membranous glomerulonephritis
(c) Rapidly progressing
glomerulonephritis
(d) Minimal change glomerulonephritis
20. A 9-year-old child has steroid dependent nephrotic syndrome for the last 5 years. He has received corticosteroids
almost continuously during this period and has cushingoid features. The blood pressure is 120/86 mmHg and
there are bilateral subcapsular cataracts. The treatment of choice is:
(OPG5th/452) (AIIMS/03)
(a) Levamisole
(b) Cyclophosphamide
(c) Cyclosporin A.
(d) Intravenous corticosteroids
21. Children have membrane proliferative glomerulonephritis + nephritic syndrome, treatment is:
(OPG5th/453) (UP 97)
(a) Corticosteroid + cyclophosphamide
(b) Sympathomimetics
(c) Steroid + antiplatelet
(d) Diuretic
22. Minimal change nephropathy progresses to:
(OPG5th/453) (AI 95)
(a) Membranous GN (b) Mesangioproliferative GN (c) Focal glomerulonephritis (d) Focal glomerulosclerosis
23. Common cause of UTI in children:
(OPG5th/455) (AIIMS 97)
(a) Always associated with septicaemia
(b) Vesicoureteric reflux
(c) Calculus
(d) Wilms tumour
Vesicoureteric refulx
Vesicoureteric reflux (VUR) refers to the retrograde flow of urine from the bladder to the upper urinary
tract.
Present in 20-35% of children with febrile UTI and is a risk factor for acute pyelonephritis and reflux
nephropathy
Detection of VUR is by radiocontrast MCU and isotope radionuclide cystrography
Operative correction of VUR is indicated in patients with
- Persistent severe (grade IV or V) reflux
- Non-compliance or intolerance to medication
- Appearance of new renal scars or deterioration of renal function during medical therapy
- Multiple recurrent UTI despite prophylaxis
Standard surgical procedures of ureteric reimplantation have cure rates of 95-97%
24. A 12-month-old child with abdominal distention, high fever, urine shows 5 WBC per hpf, severe leukocytosis with
98% neutrophils, next step in management is:
(OPG5th/456) (AIIMS/2K)
(a) Urine c/s with oral antibiotics
(b) Urine c/s with IV antibiotics
(c) Wait for c/s report
(d) Do
ultrasonography and give chloroquine
25. Investigation of choice for VUR is:
(OPG5th/457) (AIIMS 98)
(a) IVP
(b) MCU
(c) DTPA scan
(d) USG
26. The treatment of choice for primary grade V vesicoureteric reflux involving both kidneys in a 6-month old boy
is:
(NEL17th/1793)(OPG5th/458)(AIIMS/03)4
(a) Antibiotic prophylaxis
(b) Ureteric reimplantation
(c) Cystoscopy followed
(d) Bilateral ureterostomies
27. Renal tubular acidosis is associated with all of the following biochemical abnormalities except:
(OPG5th/465) (AI 90)
(a) Hypochloraemia
(b) Hypokalemia
(c) Decreased PCO2 and serum pH
(d) Normal serum calcium and
phosphorus.
Renal Tubular Acidosis (RTA):
Renal excretion of acid is reduced out of proportion to reduction in GFR as a result there is
hyperchloremic acidosis and inappropriately high urine pH. The unmeasured anion gap is normal.

19 D

20 B

21 C

22 D

23 B

24 B

25 B

26 A

27 A

79 / Pediatric Buster

There are 4 types; type 3 is rare and type 4 is associated with hyporenineic hypoaldosteronism with
hyperkalemia or tubular hyperresponsiveness to mineralocorticoids.
Type I (Distal RTA)
Type II (Proximal RTA)
1. Minimum urine pH > 5.5
< 5.5
2. % filtered HCO3 excreted < 10
> 15
3. Serum potassiumlow
Low
4. Fanconis s syndromeno
Yes
5. Daily acid excretionlow
Normal
6. Stones/nephrocalcinosisseen
Absent
7. Daily HCO3 replacement < 4 mmol/kg
> 4 mmol/kg

28. Fanconis syndrome associated with:


(a) Generalised aminoaciduria
(b) Hypophosphataemia

(c) Fructose intolerance

(OPG5th/465) (UP 96)


(d) All of the above

Fanconis syndrome: Phosphates, amino acids, protein and glucose not reabsorbed through renal
tubules. Syndrome includes cystinosis, tyrosinosis and Lowes syndrome; Systemic metabolic acidosis,
hyperchloremia hypokalemia and hypophosphatemia seen.
Tumour induced osteomalacia
Renal tubular acidosis
Chronic use of antacids.
29. In proximal renal tubular acidosis is the most important investigation is:
(OPG5th/465) (AIIMS 89)
(a) Vitamin D resistant rickets
(b) Dehydration and fever
(c) Nephrocalcinosis
(d) Bicarbonate loss
30. A 10-month-old boy, weighing 3 kg has polyuria, polydipsia and delayed motor milestones. Investigations show
blood levels of creatinine 0.5 mg/dl, potassium 3 mEq/L, sodium 125 mEq/L, chloride 88 mEq/L, calcium 8.8 mg/
dl, pH 7.46 and bicarbonate 26 mEq/L. Ultrasonography shows medullary nephrocalcinosis. The most likely
diagnosis is:
(OPG5th/466) (AIIMS/03)
(a) Renal tubular acidosis
(b) Diabetes insipidus
(c) Bartters syndrome
(d) Pseudohypoaldosteronism
Bartters syndrome: consist of
Hypokalemia (urinary K+ excretion)
Increased plasma renin
Hyperaldosteronism; metabolic alkalosis and normal blood pressure
Hypomagnesemia and elevated PGF2 is present
Weakness or periodic paralysis and polyuria occur
Main defect is defective NaCl reabsorption from thick ascending limb of Henles loop (main site for
magnesium absorption).
It may be mimicked by Mg++ deficiency; diuretic use or vomiting.
Treatment is by aldosterone antagonists to prevent potassium loss and beta blockers may lower
renin production use of PG synthetase inhibitors as indomethacin is beneficial in it.
31. In a child with diabetes insipidus; the osmolalities are:
(a) Serum-300; Urine-50
(b) Serum-30; Urine-290
(c) Serum-50; Urine-500

(OPG5th/466) (AI/2000)
(d) Serum-260; Urine-30

Nephrogenic Diabetes Insipidus : (Long arm chromosome)


Unresponsive to action of ADH
Urine is always hypotonic to plasma in presence of hypernatremia
During infancy failure to thrive, polyuria, unexplained fever and dehydration may be prominent.
Vasopressin is elevated.
32. Nephrolithiasis in infants is a complication of all except:
(a) Hyperparathyroidism
(b) Idiopathic hypercalciuria
(c) Sarcoidosis

28 D

29 D

30 C

31 A

32 D

(OPG5th/467) (AIIMS 97)


(d) Proximal renal tubular acidosis

80 / Pediatric Buster
33. Most common cause of urine obstruction in a boy:
(OPG5th/467) (AI/2001)
(a) Anterior urethral valves
(b) Posterior urethral valves
(c) Stone in urethra
(d) Duplication of renal
pelvis
34. Which one of the following is the most common cause of abdominal mass in neonates? (OPG5th/467) (AI/03)
(a) Neuroblastoma
(b) Wilms tumor
(c) Distended bladder
(d) Multicystic dysplastic kidneys
35. In a male fetus,ultrasound identification of bilateral hydronephrosis and bladder dilatation is diagnostic of:
(OPG5th/467)(UPSC/02)
(a) Polycystic kidney disease
(b) Multicystic dysplastic kidney
(c) Uteropelvic junction obstruction
(d) Posterior urethral valve syndrome
36. One-year-old male child presented with poor urinary stream since birth. The investigation of choice for evaluation
is:
(OPG5th/467) (AIIMS/03)
(a) Voiding cystourethrography (VCUG)
(b) USG bladder
(c) Intravenous urography
(d) Uroflowmetry
37. A 13-year-old boy is referred for evaluation of nocturnal enuresis and short stature. His blood pressure is normal.
The hemoglobin level is 8 g/dl, urea 112 mg/dl, creatinine 6 mg/dl, sodium 7 mg/ dl, phosphate 6 mg/dl and
alkaline phosphates 300 U/l. Urinalysis shows trace proteinurina with hyaline casts. Ultrasound shows bilateral
small kidneys and the micturating cystourethrogram is normal. The most likely diagnosis is
(AIIMS/03)
(a) Alports syndrome
(b) Medullary sponge kidney
(c) Chronic glomerulonephtritis
(d) Nephronophthisis.
38. Which one of the following statements is false with regard to pyuria in children?
(AI/03)
(a) Presence of more than 5WBC/hpf for girls and more than 3 WBC/hpf for boys. (b) Infection can occur without
pyuria
(c) Pyuria may be present without Urinary tract infection
(d) Isolated pyuria is neither confirmatory
nor diagnostic for UTI.
39. Which one of the following statements is false with regard to Xanthogranulomatous pyelonephritis in children:
(AI/03)
(a) Often affects those younger than 8 yrs of age
(b) It affects the kidney focally more frequently than diffusely
(c) Boys are affected more frequently
(d) Clinical presentation in children is same as in adults.
40. Read the passage carefully and answer the following three questions. Passage: One and a half-year-old girl child,
fed mostly on dilute cows milk,develops diarrhea which persists for 10 days. This was followed by swelling on
feet. The likely diagnosis is
(OPG5th/460)(UPSC/02)
(a) Kwashiokor
(b) Acute renal failure
(c) Congestive cardiac failure
(d) Indian childhood cirrhosis
41. One life-threatening complication of this condition is:
(OPG5th/460)(UPSC/02)
(a) Hyperkalemia
(b) Hypernatremia
(c) Metabolic acidosis
(d) Hypoglycaemia
42. The most important aspect of management is:
(OPG5th/460)(UPSC/02)
(a) Diuretics
(b) Fluid restriction
(c) Diet with adequate calories and protein
(d) Potassium restriction

33 B

34 D

35 D

36 A

37 D

38 D

39 B

40 B

41 A

42 D

12

Endocrine and Metabolic


Disorders

1. In India the commonest cause of juvenile onset of DM is:


(OPG5th/471) (AIIMS/MAY /01)
(a) MODY
(b) Fibrocalcific pancreatopathy
(c) Gallstones
(d) IDDM
2. Obesity is associated with:
(OPG5th/481) (PGI 80)
(a) Frhlichs syndrome
(b) Prader-Willi syndrome
(c) Lawrence-Moon-Biedl syndrome
(d) All
OBESITY:
Constitutional
Psychogenic
Genetic

Hormonal

Hypothalamic

Prader Willi syndrome


Lawrence-Moon-Biedl syndrome
Pseudohypoparathyroidism
Hypothyroidism
Hypogonadism
Cushing syndrome
Growth hormone deficiency
Frhlichs syndrome.

3. Cretinism is characterised by all except:


(OPG5th/483) (TN 89)
(a) Mental retardation
(b) Macroglossia
(c) Hyperpyrexia
(d) Dwarfism
4. Dwarfism with disproportionate body proportions is seen in:
(OPG5th/483) (AI 93)
(a) IUGR
(b) Hypopituitarism low growth hormone
(c) Hypothyroidism
(d) Congenital
5. The most common cause of congenital hypothyroidism:
(OPG5th/483)
(a) Thyroid dysgenesis
(b) Pendred syndrome
(c) Defective realease
(d) Deficiency of deiodinase
6. Delayed dentition is most characteristic of:
(OPG5th/483) (AIIMS 83)
(a) Mongolism
(b) Acromegaly
(c) Cretinism
(d) Malnutrition
Congenital Hypothyroidism:
Earliest sign is patent posterior fontanelle and wide open cranial sutures.
Umblical hernia
Edematous
Sleep most of the time
Physiological jaundice prolonged.
By 8-10 wk - Coarse facial features
- Thick protuberant tongue
- Hoarse voice
1 B

2 D

3 C

4 C

5 A

6 C

Endocrine and Metabolic Disorders / 82

- Rough dry pale skin


- Hypotonia; large abdomen
- Physical and mental retardation
Social smile is delayed; Dental and skeletal maturation are also significantly delayed. The child
has short stature, head size is normal but the extremities are short.
Cardiomegaly and refractory anemia is also common.
X-ray shows punctate epithelial dysgenesis.

Acquired Hypothyroidism:
Growth retardation
Ratio of upper and lower segment is disproportionate
Delayed dental development
Delayed puberty with galactorhoea and sexual precocity and high FSH and LH
Enlarged sella
Pseudotumour cerebri
Gross mental retardation is not usually present.
Goiter is freqently encountered.
7. Which of the following X-ray findings in a newborn infant would be most suggestive of hypothyroidism?
(a) Osteoporosis
(b) Epiphyseal dysgenesis
(c) Prominent thymic shadow (OPG5th/483)(AIIMS 78, 80, 81)
(d) Absence of ossification of hamate bone
8. An 11-year-old child, who is mentally and physically retarded with delayed bone age had calcifications in the
epiphyses on X-ray. The probable diagnosis:
(OPG5th/483) (AIIMS/NOV/01)
(a) Hypopituitarism
(b) Hypothyroidism
(c) Malnutrition
(d) Hypoadrenalism
9. Which of the following is a feature of hypothyroid cretinism:
(OPG5th/483) (AIIMS 92)
(a) Goitre with focal seizures
(b) Blindness with mental retardation
(c) Deafness with facial palsy
(d) Goitre with facial palsy
10. Infant with no social smile, no eyebrows, protruded tongue. Diagnosis is:
(OPG5th/483) (TN/99)
(a) Cretinism
(b) Downs syndrome
(c) Mucopolysaccharidosis
(d) Rickets
11. Which of the following is true regarding cretinism:
(OPG5th/483) (AI/2001)
(a) Short limbs compared to trunk
(b) Proportionate shortening
(c) Short limb and short stature
(d)
Short limb long stature
12. Congenital hypothyroidism is diagnosed most early by:
(OPG5th/484) (AIIMS 90)
(d) PBI
(a) RAIU
(b) TSH
(c) T3 leveis
13. A 6-month-old infant is brought with a history of constipation and excessive sleepiness. On examination, he is
lethargic, has periorbital puffiness, large tongue and umbilical hernia . The investigation which will help to
diagnose this condition is:
(OPG5th/484) (UPSC/01)
(a) T4 TSH assay
(b) Karyotyping
(c) Rectal mucosal biopsy
(d) Knee X-ray
14. In neonatal screening programme for detection of congenital hypothyroidism, the ideal place and time to collect
the blood sample for TSH estimation is:
(OPG5th/484) (AIIMS/03)
(a) Cord blood at time of birth
(b) Heal pad blood at time of birth
(c) Heal pad blood on 4th day of birth
(d) Peripheal venous blood on 28th day
15. Ambiguous genitalia is not seen in:
(OPG5th/488) (KERALA 94)
(a) Gonadal agenesis
(b) Gonadal dysgenesis
(c) Hermaphroditism
(d) Super female
Ambiguous Genitalia : Discrepancy between external and internal genitalia.
Chromosormal Anamolies - 45 X 0/46 XY
XXY (Klinefelter syndrome)
Pseudohermaphroditism
- Female
- Exposure to androgen as maternal mediation or congenital
adrenal hyperplasia
7 B

8 B

9 C

10 A

11 C

12 B

13 A

14 A

15 D

83 / Pediatric Buster
Male

- Gonadal dysgenesis; Endorgan senstivity defects;


Gonadotropin abnormalities
Buccal smear is examined for sex chromatin

16. The most common cause of female pseudohermaphroditism is:


(OPG5th/488) (AI 90)
(a) Congenital adrenal hyperplasia (b) Turners syndrome (c) Testicular atrophy (d) Mullerian duct failure
17. The most common cause of ambiguous genitalia in children is:
(OPG5th/488) (AI 92)
(a) Testicular feminisation
(b) Klinefelters syndrome
(c) 21-hydroxylase deficiency
(d) Mixed gonadal dysgenesis
18. Pseudohermaphroditism in a female child is most commonly due to:
(OPG5th/488) (AI 94)
(a) 21-hydroxylase deficiency
(b) 17-hydroxylase deficiency
(c) 11-hydroxylase deficiency
(d) 3-hydroxylase deficiency
19. A two-week old neonate is diagnosed as having ambiguous external genitalia at birth. the karyotype is 46 XX
and examination under anaesthesia does not reveal any abnormality, but the 17-alpha hydroxy progesterone is
raised. The most likely diagnosis is:
(OPG5th/490) (UPSC/01)
(a) Congenital adrenal hyperplasia
(b) Turners syndrome
(c) Rokitansky-Kuster-Hausser syndrom
(d) Klinefelters syndrome
Congenital adrenal hyperplasia:
ATCH stimulates excess adrenal steroid.
Two common varieties are21 hydroxylase deficiency and 11 beta hydroxylase deficiency.
Females have pseudohermaphroditism.
Boys have normal genitalia at birth; early closure of epiphysis (short stature) and early onset of
puberty.
21- hydroxylase Deficiency:
- Associated with aldosterone deficiency
- May present with life threatening emergencies.
- Hyponatremia and hyperkalemia
11-hydroxylase deficiency:
- Accumulation of deoxycortisol
- Hypernatremia and hypokalemia, hypertension
- Androgens are increased
17-alpha hydroxylase:
- Salt retention due to increased secretion of mineralicorticoids
- Sex steroids and glucocorticoids reduced
- Hypernatremia and hypokalamia
-

Girls: Sexual infantilism


Boys: Male pseudohermaphroditism.

20. An 8-day old breastfed baby presents with vomiting, poor feeding and loose stools. On examination the heart rate
is 190 minute, blood pressure 50/30 mmHg, respiratory rate 72 breaths/minute and capillary refill time of 4
seconds. Investigations show hemoglobin level of 15 g/dl, Na 120 mEq/L, K 6.8 mEq/L, Cl 81 mEq/L bicarbonate
15 mEq/L, urea 30 mg/dl and creatinine 0.6 mg/dl. The most likey diagnosis is:
(OPG5th/490) (AIIMS/03)
(a) Congenital adrenal hyperplasia
(b) Acute tubular necrosis.
(c) Congenital hypertrophic pyloric stenosis
(d) Galactosemia.
21. The most common enzymatic defect in congenital adrenal hyperplasia (adrenogenital syndrome) is:
(OPG5th/489)(UPSC/03)
(a) 11-hydroxylase deficiency
(b) 17-hydroxylase deficiency
(c) 21-hydroxylase deficiency
(d) 3-beta
dehydrogenase deficiency
22. In the female, congenital adrenal hyperplasia causes:
(OPG5th/490) (AIIMS 83)
(a) Infant hercules
(b) Macrogenitosomia praecox
(c) Female pseudohermaphrodites
(d) None
16 A

17 C

18 A

19 A

20 A

21 C

22 C

Endocrine and Metabolic Disorders / 84


23. Congenital adrenal hyperplasia is an inborn defect of metabolism due to inability to synthesise:
(a) Hydrocortisone
(b) Glucagon
(c) 17-ketosteroids
(d) 17-hydroxy corticoids
(OPG5th/490) (AI 89)
24. 21-hydroxylase deficiency is due to all except:
(OPG5th/490) (UP 97)
(a) Androstandione
(b) Cortisole
(c) Corticosterone
(d) Aldosterone
25. Clinical features associated with congenital adrenal hyperplasia include all except: (OPG5th/490) (AIIMS 85)
(a) Hypertension
(b) Episodes of hyperglycaemia
(c) Somatic and sexual
(d) Small hercules appearance
in boys
26. Hypertension with androgenisation of a female child is a feature of congenital adrenal hyperplasia due to
deficiency of:
(OPG5th/490) (UPSC 95)
(a) C-21 hydroxylase
(b) C-11 hydroxylase
(c) C-17 hydroxylase
(d) Desmolase
27. A newborn of 7 days old with vomiting and dehydration clinical examination was normal except for hyperpigmentation of nipple. Electrolytes Na: 120 mEq, K:9 mEq. Most likely diagnosis: (OPG5th/490) (MANIPAL/98)
(a) Primary hypothyroidism (b) Congenital adrenal hyperplasia (c) Panhypopituitarism (d) Pyloric stenosis
28. A child of 46 XY karyotype presents with BP 110/80 mmHg and ambiguous genitalia. Most likely enzyme
deficiency is:
(OPG5th/490) (AI/01)
(a) 21-hydroxylase
(b) 17-hydroxylase
(c) 11-hydroxylase
(d) 3-beta hydroxylase
29. Antenatal treatment with steroids is effective in:
(OPG5th/490)(UPSC/03)
(a) Congenital nephrotic syndrome
(b) Congenital adrenal hyperplasia
(c) Spinal muscular atrophy
(d) Congenital varicella syndrome
30. A 1 month old baby presents with frequent vomiting and failure to thrive. There are features of moderate
dehydration. Blood sodium is 122 mEq/L and potassium is 6.1 mEq/L. The most likely diagnosis is:
(OPG5th/490) (H15/2102) (AI/03)
(a) Gitelman syndrome
(b) Barter syndrome
(c) 21-hydroxylase deficiency
(d) 11-beta-hydroxylase
deficiency
31. All of following may be causes of precocious puberty in girls except:
(OPG5th/491) (AIIMS 95)
(a) Hypothalamic hamartoma
(b) McCune-Albright syndrome
(c) Granulosa cell of human ovary
(d) Congenital 21-hydroxylase deficiency
32. True (central) precocious puberty may occur due to:
(OPG5th/491) (AI 91)
(a) Prolactinoma
(b) Hypothalamic hamartoma
(c) McCune Albright syndrome
(d) Adrenal adenoma
33. Which of the following is true regarding precocious puberty is:
(OPG5th/491) (AI 94)
(a) Sexual activity attained early
(b) Mental function increased
(c) No reproductive function
(d) Body proportion enlarged
Precocious Puberty
Puberty before the age of 7 years in girls or 9 years in boys.
Central precocious puberty true - Idiopathic (Most common)
(Start before age of 3 years)
- Hypothalamic hamartoma also common cause
- Congenital anomalies; Prader-Willi; Tay-Sachs
- Other CNS abnormalities as space occupying lesion.
Pseudoprecocious puberty (peripheral):
- CAH
- Granulosa tumour of ovaries
- Leydig cell tumor
- Gonadotroplin producing tumour
Unusual forms:
M.C. Cause Albright syndrome (Polycystic fibrous dysplasia; cafe-au-lait spots and endocrinopathy of thyroid).
Signs in girls follow the sequence of thelarche; growth spurt and pubarche; menarche is early.
Elevated LH ndicates true precocious puberty

23 A

24 A

25 B

26 B

27 B

28 B

29 B

30 C

31 D

32 B

33 A

Central Nervous System / 85

If urinary 17-ketosteroids are elevated and tests are small; congenital adrenal hyperplasia is most
likely diagnosis.
Highly potent long acting analogues of LHRH are being used for management of true precocious
puberty.
Medroxyprogesterone acetate for peripheral precocious puberty.

34. The diagnosis is when familial polyostosis, precocious puberty and pigmentation is:
(OPG5th/491) (AI 93)
(a) Tuberous sclerosis
(b) McCune-Albright syndrome
(c) Klinefelters syndrome
(d) SLE
35. Delayed puberty in children are associated with:
(OPG5th/492) (PGI 2K)
(a) Poliomyelitis
(b) Hypopituitarism
(c) Hypothyroidism
(d) Anorexia nervosa
Delayed Puberty:
Sexual development (increase in the size of testes) are absent by age of 14 years in boys and by
13 years (breast budding) in girls.

34 B

35 D

13

Central Nervous System

Causes of consvulsion in newborn


1st day Birth asphyxia (MC cause neonates)
Inborn errorsphenylketonuria
Hypoglycemia; hypocalcemia
Accidental injection of local anesthetic into fetal scalp during the paracervical block
given to mother 2nd-3rd day
Cerebral convulsionhypoglycemia.
4th-7th days
Tetany
Kernicterus
Development defects
Meningitis
CMV and Toxoplasma
Tetanus
1. Seizures are often associated with:
(OPG5th/503) (AIIMS 81, DNB 90)
(a) Homocystinuria
(b) Hypoglycinaema
(c) Phenylketonuria
(d) All of the above
2. Convulsions in infants can be caused by:
(OPG5th/503) (PGI 87)
(a) Hypoglycaemia
(b) Hypocalcaemia
(c) Anoxia
(d) Fever
(e) All
3. The most common cause of convulsions in a child with fever is:
(OPG5th/508) (JIPMER 85)
(a) Febrile convulsions
(b) Meningitis
(c) Epilepsy
(d) Hypothyroidism
4. True about febrile convulsion:
(OPG5th/508) (UP 96)
(a) Focal neurological deficit (b) Phenobarbitone for two years (c) If not treated first episode then recurrence
occur
(d) Benign condition
Febrile convulsions:
Commonest cause of seizures during early childhood (6 mon-5 yr); peak age 14-18 mon
Not related to height of temperature but frequent if temperature rises abruptly > 35C.
Fits occur within 24 hr. of onset of fever.
More than one episode may occur on the same day.
Convulsions are always generalized.
No postictal residual neurological deficit and EEG a few days after the seizure is always normal.
Family history in siblings is not always forthcoming but present in 1/3rd cases
L.P. is always done and results are atypical.
Pertussis vaccine should not be given to a child with progessive neurological illness or previous
convulsion history but can be given in nonprogessive disorders of neuromuscular system.
1 D

2 E

3 A

4 D

87 / Pediatric Buster

Chance of developing epilepsy in near future is high if


1. Sezure lasts more than 15 min.
2. Focal in distribution
3. EEG is abnormal even after a few days.
4. More than 2 seizures in a day or 4 febrile seizure in a year.
5. Complex partial seizures may menifest after several years of prolonged febrile seizures.
6. Prevention of recurrent febrile seizures may not reduce the risk of developing epilepsy in the
future.
7. Managed by prompt reduction of temperature with paracetamol or hydrotherapy.
8. Aspirin is avoided because of fear of Reyes syndrome.
Diazepam may be used for typical seizures while valproate or phenobarbitone for atypical ones.
Phenobarbitone has no role in preventing recurrent febrile seizure while diazepam is safe for reducing
risk of recurrence.
5. Which is true regarding febrile seizures:
(OPG5th/508) (AI 93)
(a) 50% recurrence
(b) Long-term phenytoin required
(c) Interictal EEG normal
(d) Status epilepsy is
common
6. Untrue about acute febrile convulsions is:
(OPG5th/508) (ARNU 95)
(a) Focal in nature
(b) EEG normal after 2 weeks
(c) Usually occur below 6 years age
(d) All
7. Features of typical febrile seizures in children include:
(OPG5th/508) (AI 90)
(a) Mental retardation
(b) Positive family history in one-third of cases
(c) Diagnostic EEG changes
(d) Focal neurological deficits.
8. Regarding febrile seizures which of the following is/are true:
(OPG5th/508) (PGI 2000)
(a) Initially anticonvulsants are not needed
(b) Can lead on to permanent neurovascular deficits
(c) There
is associated family history
(d) Carbamazepine is used in the prophylaxis
9. Which is false of febrile convulsions:
(OPG5th/509) (JIPMER 92)
(a) Aspirin must be given during spike of fever
(b) Seizures occur early in disease
(c) Temperature more
than 38C
(d) Occurs from 6 months to 5 years
10. A 6-year-old child with acute onset of fever of 104F developed febrile seizures and was treated. To avoid future
recurrence of seizure attacks what should be given:
(OPG5th/509) (AIIMS /MAY/01)
(a) IV diazepam infusion over 12 hr
(b) Paracetamol 400 mg + phenobarbitone daily
(c) Paracetamol
400 mg 6th hrly
(d) Oral diazepam 6th hrly
11. Treatment used in acute febrile convulsion in children is:
(OPG5th/509) (AIIMS 96, AI 98)
(a) Diphenyl hydantoin
(b) Phenobarbitone
(c) Diazepam
(d) Ethosuximide
12. A 4-year-old male child has febrile seizures. Best prophylaxis is:
(OPG5th/509) (PGI 2K)
(a) Paracetamol q 6 hourly
(b) Paracetamol and diazepam
(c) Diazepam
(d) Phenobarbitone
13. Absence attacks in children are suggestive of:
(OPG5th/510) (AIIMS 98)
(a) Grand mal epilepsy
(b) Petit mal epilepsy
(c) Myoclonic epilepsy
(d) None of the above
14. All are the features of absence seizures except:
(OPG5th/510) (AI/04)
(a) Usually seen in childhood
(b) 3-Hz spike wave in EEG.
(c) Postictal confusion
(d) Precipitation by
hyperventilation.
Absence Attacks
Petit Mal Epilepsy - Usually start b/w 4-5 yr.
Peak prevalence is in children b/w 6-8 yr.
Disappears before puberty
Not preceded by an aura
Patient may show sudden staring spell and eye fluttering.
Postictal drowsiness and confusion do not occur.
Hyperventillation often precipitates the attack
There may be learning disabilities and behaviour disorders.
EEG show characteristic 3 spike/second and slow wave pattern.
DOCEthosuximide and valproate.
5 C

6 A

7 B

8 A

9 A

10 B

11 C

12 B

13 B

14 C

Central Nervous System / 88


Myoclonic Epilepsy of Infancy (Hypsarrhythmia) or Infantile Spasm
Generalized seizures b/w birth and 12 months of age.
Usually brief and disappears at 3-5 yr and appears by other forms of generalised seizures.
Also called Salam seizuresdue to sudden massive contraction of the abdominal muscles which
force the patients to twist like a jackknife at the waist.
Mental retardation is associated; common cause iscutaneous syndrome (Tuberous sclerosis).
DOC- Corticosteroid or ACTH in high doses; valproate may also be helpful.
15. The feature of Petit mal epilepsy of EEC is:
(OPG5th/510) (AIIMS 87)
(a) Spike and dome
(b) Continuous situated spikes
(c) Waves at the rate of 10/sec.
(d) Continuous tall
spikes
16. A 3-year-old child develops asynchronous clonic movements of one side of face and right upper limb. He does not
lose consciousness and the episode subsides after 5 minutes. This is the first episode and the child is a febrile.
The most likely diagnosis is:
(OPG5th/511) (UPSC/2001)
(a) Complex partial seizures
(b) Infantile spasms
(c) Simple partial seizures
(d) Febrile seizures
17. Hypsarrhythmia in a child is due to:
(OPG5th/511) (AIIMS 91, DELHI 93)
(a) Grand mal epilepsy
(b) Petit mal epilepsy
(c) Myoclonic epilepsy
(d) Reflex epilepsy
18. Sudden flexion of body at the waist is called:
(OPG5th/511) (PGI 81, ROHTAK 87)
(a) Focal motor seizure
(b) Salam attack
(c) Myoclonic seizure
(d) Jacksonian seizure
19. Carbamazepine is the drug of choice in the treatment of:
(OPG5th/513t) (AI 90)
(a) Absence attacks
(b) Partial complex seizures
(c) Infantile spasm
(d) Postanozic myoclonus
20. The drug of choice for status epilepticus in children is:
(OPG5th/513t) (AIIMS 83)
(a) Hydantoin
(b) Phenobarbitone
(c) Diazepam
(d) Carbamazepine
21. Drug of choice in petit mal epilepsy is:
(OPG5th/513t) (UPSC 85, 87, 88)
(a) Phenobarbitone
(b) Dilantin
(c) Ethosuximide
(d) Valproic acid
22. Most common cause of meningitis in neonates is:
(OPG5th/517) (JIPMER 86)
(a) Meningococcus
(b) Haemophilus
(c) E. coli
(d) Streptococcus
23. Bacterial meningitis in a 9-month-old child is commonly caused by:
(OPG5th/517) (AIIMS 83)
(a) H. influenzae
(b) M. tuberculosis
(c) Pneumococcus
(d) Streptococcus
24. Bacterial meningitis in children (2 months -12 years of age) is usually due to the following organisms except:
(OPG5th/517) (AI/04)
(a) Streptococcus pneumoniae
(b) Neisseria meningitidis
(c) Haemophilus influenzae type B
(d) Listeria
monocytogenes.
Bacterial Meningitis
Age group

Common bacterial
pathogens

Initial antibiotic
regimens

Newborn

Escherichia coli
Klebsiella pneumoniae
Listeria monocytogenes
Enterococcus sp.
Salmonella sp.
H.influenzae
S.pneumoniae
Group B Streptococcus
Listeria monocytogenes
H.influenzae
S. pneumoniae
N. meningitidis

Ampicillin plus
aminoglyccoside
or cefotaxime

4-12 weeks

12 weeks and older

15 A

16 C

17 C

18 B

19 B

20 C

21 C

22 C

Ampicillin plus either


cefotaxime
or ceftriaxone
Ceftriaxone or
cefotaxime or
ampicillin plus
chloramphenicol

23 A

24 A

89 / Pediatric Buster

S/S in neonates; vacant stare; persistent vomiting with fever, refusal to suck after a normal feeding
pattern has been established, poor tone poor cry; circulatory collapse, Tremors convulsion and
neurological deficit.
S/S in older children; irritability; photophobia, high fever cheyne stokes; seizures; generalized flexed
posture, hypertonia, marked neck rigidity; Kernigs sign positive; and Brudzinski sign positive.

25. Meningitis in preschool children is most often due to:


(a) Pneumococcus
(b) Streptococcus
(c) Staphylococcus

(OPG5th/517) (AI 93)


(d) Haemophilus

Pneumococcal meningitis:
Common cause of septic meningitis at all ages except for first few weeks.
Follows otitis media, sinusitis, pneumonia or head inhury
Subdural effusion is a usual complications.
26. Common organism causing meningitis in 6 months to 2 years of age:
(a) Streptococcus
(b) H. influenzae
(c) Neisseriae
(d) Staphylococcus

(OPG5th/517) (AI 93, 96, 98)

Haemophilus meningitis:
Usually type B organisms
Subdural effusions follow the initial illness.
Convulsions are common.
Residual auditory deficit is a common complications.
Staphylococcal meningitis:
Occurs usually in the newborn baby with associated umbilical infection; pyoderma or septicemia.
27. Aseptic meningitis is mostly caused by:
(OPG5th/517) (UPSC 85, JIPMER 87)
(a) Adenovirus
(b) Enterovirus
(c) Arbovirus
(d) Herpes virus
28. The most common presentation of neonatal meningitis is:
(OPG5th/517) (ORISSA 98)
(a) Bulging fontanels
(b) Nuchal rigidity
(c) Poor feeding
(d) Convulsion
29. The most common complication following meningococcal meningitis is:
(OPG5th/518) (AIIMS 85)
(a) Subdural effusion
(b) Hydrocephalus
(c) Arthritis
(d) Waterhouse-Friderichsen syndrome
Meningococcal meningitis:
Epidemics are caused by sero type A organism
Carrier state is common in children.
Children show petechial hemorrhages.
Meningococcemia may be associated with acute fulminating illness with adrenal insufficiently;
hypotension and shock. This is MC complication called Waterhouse-Friderichsen syndrome.
30. The most common complication of Haemophilus influenzae meningitis is:
(OPG5th/518)
(a) Arthritis
(b) Bronchopneumonia
(c) Otitis media
(d) Subdural effusion
31. Subdural effusion is most commonly associated with meningitis caused by:
(OPG5th/518) (AI 89)
(a) Haemophilus influenzae
(b) Mycobacterium tuberculosis
(c) Neisseria meningitis
(d) E. coli.
32. A patient with meningitis has the CSF finding of reduction in glucose, increase in protein and moderate to low
chlorides and rise in polymorphs; the diagnosis is:
(OPG5th/518) (AIIMS/99)
(a) Meningococcal meningitis
(b) TBM
(c) Viral meningitis
(d) Fungal meningitis
33. In meningitis in children, most common complication is:
(OPG5th/518) (AIIMS 94)
(a) MR
(b) Seizures
(c) Hearing loss
(d) Hydrocephalus
Complications of meningitis:
1. Neurological deficit
2. Mental deterioration
25 A

26 B

27 B

28 C

29 D

30 D

31 A

32 A

33 B

Central Nervous System / 90


3.
4.
5.
6.
7.
8.
9.

Cerebritis
Brain abscess
Focal fits
Deafness
Subdural empyema (H. influenzae or Strepto pneumoniae)
Internal hydrocephalus
Spinal cord compression

34. CSF glucose is markedly reduced in:


(OPG5th/518)
(BHU/92)
(a) Encephalitis
(b) Pyogenic meningitis
(c) TB meningitis
(d) Viral meningitis
35. Indication for use of corticosteroids in a child with meningococcemia would be presence of:
(OPG5th/518) (AIIMS 81, AP 91)
(a) Shock
(b) Petechiae
(c) Meningitis
(d) Temperature greater than 104F
36. A young girl with mild fever, convulsions, vomiting having in CSF GLU-40 mg%, Cl~ 560 ml/L with predominent
lymphocytosis, most likely to have:
(OPG5th/521) (AI/93)
(a) Pyogenic meningitis
(b) TBM
(c) Viral meningitis
(d) Brain abscess
Tubercular meningitis:
Most common b/w 6-24 months.
Pathological hallmark is vasculitis and arachnoiditis.
Usually the white matter is affected and there is endarteritis obliterans.
Exudates and tubercles are numerous in the region of temporal lobe and along the course of middle
meningeal artery.
Patient present with prodromal stage followed by meningeal signs and coma (Bitots respiration).
Usual sequelae are
- Hemiplegia
- Optic atrophy
- Hydrocephalus
- Seizures and cranial nerve palsies
37. Complication of TB meningitis includes A/E:
(OPG5th/521) (AI 95)
(a) Optic atrophy
(b) Parkinsonism
(c) Hydrocephalus
(d) Seizures
38. A 10-month-old child presents with two weeks history of fever, vomiting and alteration of sensorium. Cranial CT
scan reveals basal exudates and hydrocephlus. The most likely etiological agent is:
(OPG5th/521) (AI/04)
(a) Mycobacterium tuberculosis
(b) Cryptococcus neoformans
(c) Listeria monocytogenes
(d) Streptococcus
pneumoniae
39. Which is not a feature of tuberculous meningitis?
(OPG5th/521) (PGI 80, AIIMS 92)
(a) Vasculitis
(b) Subarachnoiditis
(c) Inflamed dura
(d) Decreased CSF glucose
40. Reyes syndrome is caused by all except:
(OPG5th/524) (KERALA 95) (AIIMS 92)
(a) Adenovirus
(b) RSV
(c) Herpes
(d) Influenza
41. All of the following are true of Reyes syndrome except:
(OPG5th/524) (UP 97)
(a) It frequently complicates viral infections
(b) Prothrombin time is prolonged
(c) Disease may be
precipitated by salicylates
(d) Deep jaundice is present
42. All are seen in Reyes syndrome except:
(OPG5th/524) (AIIMS 87)
(a) Aminoaciduria
(b) Metabolic acidosis
(c) Increased serum transaminases
(d) Respiratory alkalosis
43. Which is incorrect about Reyes syndrome:
(OPG5th/524) (JIPMER 91)
(a) Bilirubin of more than 3 mg
(b) Normal prothrombin time
(c) Cerebral oedema
(d) Microfatty changes
in liver without inflammatory changes
Reyes syndrome: (Jamshedpur fever)
Charaterized by encephalopathy (cerebral edema), microvesicular fatty change of the liver and
mitochondrial injury.
34 B

35 A

36 B

37 B

38 A

39 C

40 C

41 D

42 A

43 A

91 / Pediatric Buster

Most commonly associated viral illness is type B influenza virus.


Other associated factors are hyperammonemia, elevated serum, free fatty acids, salicylate aspirin
toxicity.
Mostly affects children < 15 yr.
Levels of bilirubin and aminotransferases is also elevated.
Child present with vomiting disturbance of sensorium irregular breathing and tachypnea.
No focal neurological or meningeal signs.
Hepatomegaly is present in 50% cases.
Jaundice is infrequent.
There is some degrees of hypoglycemia with low levels of glucose in CSF.
Prothrombin time is prolonged.
EEG shows generalised slow waves.

44. Earliest change in raised ICT in a child is:


(a) Silver beaten appearance
(b) Posterior clinoid erosion

(OPG5th/525) (KERALA 91)


(c) Erosion of sella
(d) Sutural separation

Classification of brain tumuor


Infratentorial
Supratentorial
(i) Cerebellar astrocytoma
(i) Astrocytoma and related glial tumors.
(ii) Medulloblastoma
(ii) Ependymomas
(iii) Brainstem gliomas
(iii) Craniopharyngioma
(iv) Ependymomas
(iv) Optic nerve glioma
(v) Pineal region tumors
(vi) Choroid plexus papilloma
(vii) Leukemia, metastatic tumors
45. True about brain tumour in children is:
(OPG5th/526) (AI/2000)
(a) Infractentorial are commonest one
(b) Most common are supratentorial ones
(c) MC is brainstem
glioma
(d) Brain tumour are commonest form of malignancy
46. Suprasellar calcification in a child is indicative of:
(OPG5th/527) (AI 92)
(a) Neuroblastoma
(b) Glioma
(c) Meningioma
(d) Craniopharyngioma
47. The most common type of intracranial tumour in childhood is:
(OPG5th/527) (AI 88)
(a) Dermoid
(b) Glioma
(c) Meningioma
(d) Craniopharyngioma
48. Most common posterior fossa tumour in children:
(NEL17th/1703) (AIIMS 95)
(a) Medulloblastoma
(b) GI multiforme
(c) Astrocytoma
(D) Meningloma
49. A 10-year-old child presented with headache, vomiting, gait instability and diplopia, on examination he had
papilloedema and gait ataxia The most probable diagnosis is:
(NEL17th/1706) (AIIMS/02)
(a) Hydrocephalus
(b) Brainstem tumour
(c) Suprasellar tumour
(d) Midline posterior fossa tumour.
50. Most common posterior fossa tumour in children:
(NEL17th/1732) (AI 97)
(a) Medulloblastoma
(b) G. multiforme
(c) Astrocytoma
(d) Meningioma
Brainstem glioma:
The most common cranial nerve symptom include diplopia and facial weakness.
C/F of increased intracranial pressure including papilloedema occur late.
Makes the diagnosis ususally apparent and biopsy is unnecessary.
Surgical treatment of brainstem glioma is controversial.
The primary treatment is irradiation and although some brainstem gliomas are radiosensitive; the
5 yr survival is approximately 20%.
Chemotherapy is of no value.
Low grade focal tumours of midbrain or medulla have an excellent prognosis following radical
excision.

44 D

45 A

46 D

47 B

48 C

49 D

50 C

Central Nervous System / 92


Brain tumor:
Second most common form of malignancy.
M.C. solid tumor seen in children.
Approximatley 70% are infratentorial
Medulloblastomas:
They are the most common tumors of first decade of life and also in children < 7 yr.
M.C. primitive neuroectodermal tumor.
M.C. site in childrenvermis of cerebellum (middling) and present with truncal ataxia.
Disseminated via CSF and site of recurrence is posterior fossa.
It is an expansile tumour but prognosis is favourable in childhood.
Treatmentsurgical with irradiation or chemotherapy
Astrocytomas:
M.C. infratentorial tumor and most of them arise in cerebellum.
Brainstem gliomas are subgroup of astrocytoma, occur mostly in 1st two decade of life.
Pilocytic astrocytomas are characterized by rosenthal fibres and microcysts.
Brain stem gliomaIIIrd common infratentorial tumour.
Diagnostic features are ependymal canal and rossette.
GFAP (Glial fibrilliary acid protein) is also rossette.
Present with hydrocephalus.
Metastasise along the neuraxis.
Surgical excision is rarely possible.
51. Tumour with best prognosis is:
44(OPG5th/527) (AI 94, 97)
(a) Medulloblastoma
(b) Ependymoma
(c) Cerebellar astrocytoma
(d) Glioblastoma-multiformi
52. Most common tumour in children:
(OPG5th/527) (AI-96)
(a) Glioma
(b) Ependymoma
(c) Meningioma
(d) Lymphangioma
53. The commonest intracranial tumour in childhood is:
(OPG5th/527) (AIIMS 83, 87) (AI 89)
(a) Dermoid
(b) Glioma
(c) Meningioma
(d) Medulloblastoma
54. In children the suprasellar tumour and calcified the diagnosis is:
(OPG5th/527) (AIIMS 96)
(a) Craniopharyngioma
(b) Rathkes pouch anomaly
(c) Meningioma
(d) Neuroblastoma
Craniopharyngiomas:
M.C. supratentorial brain tumor in children.
Arises from squamous epithelial cell rests of embryonic Rathke pouch.
Histologic pattern resembles that of enamel of tooth, They are cystic with oily fluid and glittering
cholesterol crystals.
X-ray skull shows calcification; enlarged sella turcica and erosion of clinoid process.
C/F include
- Growth failure
- Bitemporal hemianopsia
- Visual field defects
- Signs of raised ICT
- Endocrine abnormalities.
Incidence of Glioma is higher in children with neurocutaneous syndrome.
Foster Kennedy syndromeatrophy of fundus of the same side and papilloedema in the opposite
eye in frontal lobe tumor.
55. Following are false of tuberculoma in childhood except:
(a) Always associated with meningitis
(b) Commonest in brainstem
extracerebral TB
(d) No macrophages are seen
51 C

52 D

53 B

54 A

55 B

(OPG5th/528) (AI 93)


(c) Always associated with

93 / Pediatric Buster
Hydrocephalus:
Characterized by increased CSF pressure
M.C. cause of obstructive hydrocephalus in a child is aqueduct stenosis.
Other causes may be Dandy Walker syndrome (posterior fossa cyst continuous with fourth ventricle)
and Arnold-Chiari syndrome; intrauterine infections as rubela, CMV and toxoplasmosis.
M.C. cause of acquired hydrocephalus is post-inflammatory stricture after bacterial meningitis.
Other acquired causes areposterior fossa tumors and intracranial hemorrages.
C/F
Headache
Papilloedema
Nausea
Ophthalmoplegia
Vomiting
Irritability
Impaired upward gaze.
Normal pressure hydrocephalus:
D - Dementia
I - Incontinence
A - Ataxia
L - Low IQ
E - Extensor plantar
D - Dysphagia
Diagnosis:
Increase in the head circumference in the first 3 months of life by more than 1 cm every fortnight.
Excessive molding of skull bones during labor.
Widening os squamoparietal sutures.
Treatment is repeated lumbar puncture.
or
Shunt operation with or without acetazolamide
56. Common cause of hydrocephalus in children is:
(OPG5th/529) (AI 98, 96)
(a) Congenital anomaly
(b) Perinatal injury
(c) Postinflammatory stricture
(d) Brain tumours
57. Most common cause of obstructive hydrocephalus in a child is:
(OPG5th/529) (JIPMER 93)
(a) Dandy-Walker syndrome
(b) Arnold-Chiari malformations
(c) Aqueductal stenosis
(d) Meningitis
58. Neonatal hydrocephalus is seen in A/E:
(OPG5th/529) (UP 94)
(a) Intraventricular haemorrhage
(b) Aqueductal stenosis
(c) Brain tumour
(d) AIDS
59. All of the following cause congenital hydrocephalus except:
(OPG5th/529)
(DELHI/94)
(a) Toxoplasma
(b) TBM
(c) Rubella
(d) Arnold-Chiari syndrome
60. The most common cause of hydrocephalus in children is:
(OPG5th/529) (AI 96)
(a) Posterior fossa tumours
(b) Congenital malformation
(c) Complication of meningitis
(d) Rupture of intracranial aneurysm
61. Dandy-Walker syndrome is due to obstruction of:
(OPG5th/530) (AIIMS 80, JIPMER 91, 92)
(a) Foramen of Monro
(b) Foramen of Morgagni
(c) Foramen of Magendie and Luschka
(d) Arachnoid granulations
62. All of the following are neural tube defects except:
(OPG5th/531) (AI/04)
(a) Myelomeningocele
(b) Anencephaly
(c) Encephalocele
(d) Holoprosencephaly
NEURAL TUBE DEFECTS include
1. Spina bifida (meningocele, meningomyelocele, spina bifida occulta)
2. Anencephaly
3. Encephalocele
4. Craniorhachischisis
5. Iniencephaly
63. In unexplained ataxia in a child ...poisoning is a possibility:
(a) Arsenic
(b) Lead
(c) Salicylate
(d) Mercury
56 C

57 C

58 D

59 B

60 C

61 C

62 D

63 B

(OPG5th/538) (PGI 89)

Central Nervous System / 94


64. The most common type of cerebral palsy is:
(a) Spastic
(b) Extrapyramidal
(c) Atonic

(OPG5th/540) (PGI 92)


(d) None

Cerebral Palsy
Nonprogressives neuromuscular disorders of cerebral origin.
Not a familial disease
Incidence 2/100 live birth may be due to birth trauma, anoxia, hypoglycemia or infections.
Child posture and movement are permanently impaired.
Intellectual function may or may not be affected.
Pyramidal (Spastic type) is M.C.; characteristics are
- Abnormal neonatal reflexes (prolonged)
- Hyperexcitable and firm grasp reflex
- Inability to flex knees or limbs
- Opisthotonus
- Stretch tendon reflex brisk
- Diplegia
In atonic type
- Despite hypotonia
- Tendon reflexes are brisk and Babinski response is +ve
Kernicterus is associated with extrapyramidal athetoid type.
These children may associate dental defects as enamel hypoplasia.
65. Cerebral palsy which manifests after the age of one year is most often:
(OPG5th/540) (JIPMER 93)
(a) Ataxic
(b) Spastic
(c) Diplegia
(d) Mixed
66. Cerebral palsy is defined as a:
(OPG5th/540) (JIPMER 80, UPSC 87)
(a) Nonprogressive neuromuscular disorder of cerebral origin (b) Progressive neuromuscular disorder of cerebral
origin (c) Nonprogressive neuromuscular disorder of peripheral origin (d) Progressive neuromuscular disorder
of peripheral origin
67. Preventable cause of mental retardation is:
(OPG5th/543) (AI 95)
(a) Down syndrome
(b) Phenylketonuria
(c) Cretinism
(d) Cerebral palsy

Mental retradation is associated with


- Down syndrome
- Anoxia neonatorum
- Hypoxia
- Hypoglycemia
- Kernicterus
- Postvaccinal encephalopathies.
- Intracerebral haemorrhage.
- Aminoaciduria and
- Tuberous sclerosis.
Grades of mental retardation
IQ
Mild
- 51-70
Moderate
- 36-50
Severe
- 21-35
Profound
- 0-20

68. IQ (intelligence quotient) of a child means:


(OPG5th/543)
(DELHI 96)
(a) The creative efficiency of child
(b) The capability of the child to perform intellectual tasks in relation to
other children of same age (c) The efficiency of memory of child (d) Determination of the quantificial learning
ability of child
69. Child with epilepsy, mental retardation, hypopigmented patch on trunk and abdomen, is suffering from:
(OPG5th/546) (AIIMS/2K)
(a) Sturge Weber syndrome
(b) Tuberous sclerosis
(c) Neurofibromatosis
(d) Incontenentia pigmenti
64 A

65 B

66 A

67 C

68 B

69 B

95 / Pediatric Buster
Tuberous Sclerosis complex: Autosomal dominant
Cardinal features are skin lesions; convulsions and mental retardation; may affect heart; kidney,
eye, bone and lung.
Tubers may be (-nt upto 3-4 yr) in subependymal region; calcify and project in ventricles as candle
dripping.
Hydrocephalus is seen
MRI is diagnostic
Patients develop infantile spasms and hypsarrhthmic EEG and later develop myoclonic epilepsy.
Brain tumours are less but chances of astrocytoma is there
Skin lesions areAsh
Leaf macules (early cutaneous sign)
Adenoma sebaceum
Shagreen patch (lumbosacral)
50% have rhabdomyoma of heart which resolve spontaneously.
70. Sphenoid wing dysplasia is seen in:
(OPG5th/546) (JIPMER/2K)
(a) Von Hippel Lindaus disease (b) Neurofibromatosis (c) Sturge Weber syndrome (d) Bournvilles disease
71. True about Duchennes muscular dystrophy are A/E:
(OPG5th/547) (AI 96)
(a) X-linked recessive
(b) Cardiomyopathy
(c) Respiratory failure in thud
(d) Congenital hypotonia
Duchennes type pseudohypertrophic muscular dystrophy
X-linked recessive inheritance with a high mutation frequency.
One-third of new cases have no previous family history.
The muscle fibres vary in size; clusters of basopholic fibres with vesicular nuclei; generalized fibrosis
focal areas of necrosis, and phagocytosis are observed.
Symmetric weakness; initially pelvifemoral later weakness in leg shoulder girdle and then trunk
muscles, pseudohypertrophy of calves, reduced intelligence and cardiac involvement.
Gowers sign is observed. Waddling gait with a compensatory lumbar lordosis.
Most patients die in the second decade due to intercurrent or heart failure.
Serum levels of creatine phosphokinase are elevated in most of the carrier females.
Other features are:
Pseudohypertrophy of deltoid and infraspinatus, fatty infitration of heart, respiratory infections.
Macroglossia, bone thinning, scoliosis, pathological fracture and loss of all reflexes except ankle jerk.
ECG shows tall R waves and deep Q waves in V1 lead.
Steroid may improve strength but no effect on prognosis
72. Which does not present as floppy baby:
(a) Littles disease
(b) Werdnig-Hoffman syndrome

(c) Down syndrome

(OPG5th/555) (JIPMER 91)


(d) Ehler-Danlos syndrome

Floppy Infant- Infant with marked hypotonia of all the muscles.


Causes - CNS lesions
- Birth trauma; asphyxia, ARDS.
- Kernicterus; Down syndrome
- Inborn errors of metabolism
Spinal cord lesions
- Transverse myelitis, Werdning-Hoffmann spinal muscular atrophy GB syndrome.
Peripheral nerves
- Polyneuritis
Diptheric paralysis
Arsenic neuropathy
Myoneural junction
- Myasthemia gravis
Muscles
- Muscular dystrophies
(central core disease)
Glycogen storage disease
Polymyositis
Mitochondrial myopathy (raggedred fibres)
70 A

71 D

72 A

96 / Pediatric Buster
Misc

PEM
Rickets
Ehler Danlos syndrome
CHDS
Prader-Willi syndrome
Cretinism
Infant botulism

73. Floppy baby syndrome is seen in:


(OPG5th/555) (AIIMS/2000)
(a) Clostridia botulinum infection
(b) Down syndrome
(c) Hypothyroidism
(d) Muscle dystrophy
74. All are seen in myotonic dystrophy except:
(NEL 17th/2065) (JIPMER 91)
(a) Cataract
(b) Respiratory failure
(c) Cardiac defect
(d) Enlarged testes
75. A patient with frog position, areflexia, hypotonia, normal mentation, tongue fasciculations and delayed
milestones is suffering from:
(NEL17th/2075)s(AIIMS/2K)
(a) Limb girdle dystrophy
(b) Spinal muscle dystrophy
(c) Guillain Barr syndrome
(d) Downs syndrome

73 A

74 D

75 B

14

Malignancies in Childhood

Leukemia:
Most common malignancy of childhood; second is brain tumor.
The frequency of leukemia increases with
Fanconis anemia
Down syndrome
Blooms syndrome
Klinefelters syndrome
Wiskott Aldrich syndrome and ataxia telangiectasia
Radiation exposure increases the risk of all leukemias except CLL and hairy cell.
Incidence of leukemia is 13/lac population
1. The most common type of leukaemia in children is:
(OPG5th/561) (AI
(a) ALL
(b) AML
(c) CLL
(d) CML
2. The most common haematological malignancy in children is:
(OPG5th/561) (KERALA
(a) CLL
(b) CML
(c) AML
(d) ALL
3. Blast cells of acute lymphocytic leukemia in childhood contain:
(OPG5th/562) (UP
(a) Surface antigen
(b) CALLA +ve
(c) Antibodies on WBC
(d) Excessive mitochondria
4. Poor prognosis in acute lymphatic leucaemia is indicated by all, except: (OPG5th/562) (JIPMER 80, UPSC
(a) Intrathoracic mass
(b) Children between 2 and 10 years
(c) Cranial secondaries
(d) WBC around 10,000 mm
5. Best prognosis in ALL in:
(OPG5th/562)(UP
(a) Pre B cell
(b) Precursor B cell
(c) B cells
(d) T cells
ALL:
Accounts for 70-80% of all leukemias
Incidence is 1:2000 live birth.
Peak age of onset is 3-7 yr; more common in males.
It has three subtypes L1, L2, L3.
L1 carries the best prognosis.
L3 is associated with EBV
Cytochemically all takes PAS stain and coarse granules are seen in positive cells, Tdt is present
in 90% of leukemic lymphoblasts.
Tdt is not found in L3 subtype
The cells may be CALLA +VE or CALLA-VA, 90% or 10%
Patient may demonstrate hyperdiploidy or aneuploidy. Hyperdiploidy is associated with good prognosis.
1 A

2 D

3 B

4 D

5 A

89)
94)
96)
89)

96)

Malignancies in Childhood / 98

Patients with 9: 22; 8: 14 or 4 : 11 translocation carry poor prognosis


Children less < 2 yr or > 10 yr carry poor prognosis
Other poor prognostic markers are :
- WBC > 25,000/cumm
- Mediastinal widening
- Hypogammaglobulinemia
- Platelet count < 40,000/cmm
- Presence of T or B cell marker
- Massive organomegaly
- Extramedullary disease at diagnosis
- Best prognosis is seen with CALLA +VE subtype.
C/F in order of frequency
- Fever (60%)
- Fetechiae
(M.C. site oral and GIT mucosa)
- Bleeding (48%) when platelet count <20,000
- Anorexia
- Malaise and decreased activity
- Bone pain,
- Arthralgia, hepatosplenomegaly and rarely lymphadenopathy
Anterior mediatinal mass is common with T cell subtype of ALL
Metabolic complication are commonest with ALL-L3 commonest metabolic complications are
hyponatremia and hypokalemia.
Testicular involvement may also be seen more with T-cell type.
Neurological complications is less common at time of diagnosis but meninges are common site of
relapse.
Presenting foremost symptom of leukemic meningitis is headache, malaise or third to 7th cranial
nerve palsies.
CNS meningitis shows leukemic blast cells, decreased glucose and increased protein.
Complete remission is achieved in 90% patient of ALL in 4 weeks by combination of vincristine and
prednisolone with either L-asparaginase or daunorubicin.
CNS prophylaxis includes whole cranium radiation to 2000 rads and five intrathecal treatments
using methotrexate.
In maintenance phase of 2-3 yr; 6-mercaptopurine a methotrexate is commonly used.
Testicular relapse is treated by irradiation to affected testis.
Complications of treatment of ALL: myelosuppression and immunosuppression
- Pneumocystic pneumonia
- Viral infections as herpes simplex; zoster measles and CMV
- Sterility
- Secondary malignancy and myeloid leukemia (late)
AML
M3 is associated with disseminated coagulopathy
Seven subtypes
M4 and M5 with infiltration of gums and CNS leukemia
M7 with pancytopenia
Symptomatology is same as ALL but microvascular occlusions are seen most commonly.
Chloromas may occur at presentation (perioccular) more common with (8;21) M2 type.
Hepatosplenomegaly are present in 50% cases.
Auerrods are diagnostic of AML.
Cytochemical staining with myeloperoxidase; -naphthyl-ASD chloroacetate esterase and
Sudanblack helps in diagnosis.
Complete remission induction rate with cytarabine plus daunorubicin is 60 - 80%.
Trans-retinoic acid therapy can induce complete remission is M3 subtype.
Bone marrow transplantation in therapeutic approach is first remission.
Monoclonal antibodies have some roles.

99 / Pediatric Buster
6. Mediastinal widening is found in:
(OPG5th/563) (AIIMS 89)
(a) Histiocytic lymphoma
(b) Prolymphatic leukaemias
(c) CML
(d) ALL
7. Which type of leukaemia given prophylactic methotrexate for extracranial irradiation:
(OPG5th/564) (AI 97)
(a) ALL
(b) AML
(c) CLL
(d) CML
8. A 5-year-old child presents with history of fever off-and-on for past 2 weeks and petechial spots all over the body
and increasing pallor for past 1 month. Examination reveals splenomegaly of 2 cm below costal margin. The most
likely diagnosis is:
(OPG5th/563) (AI/04)
(a) Acute leukemia
(b) Idiopathic thrombocytopenic purpura
(c) Hypersplenism
(d) Aplastic anemia.
9. Wilms tumor is associated with all of the following except:
(OPG5th/574) (AI 90)
(a) Aniridia
(b) Beckwith syndrome
(c) Hemihypertrophy
(d) Polycystic kidney
10. True about juvenile CML:
(OPG5th/569) (UP 96)
(a) Philadelphia chromosome
(b) High HbF content
(c) Low Alkaline PO4
(d) Massive splenomegaly
Juvenile CML:
Children < yr
Philadephia chromosome is negative
HbF is 15-50%
Facial rash; Bleeding manifestation and lymphadenopathy is frequent
Frequent normoblasts on peripheral smear
Poor response to busulphan.
Median survival is 9 months
11. Which of the following childhood malignancies has highest cure rater:
(OPG5th/575) (PGI 81, AMU 86)
(a) Wilms tumour
(b) Neuroblastoma
(c) Retinoblastoma
(d) Rhabdomyosarcoma
Wilms Tumor (Nephroblastoma) Most common abdominal malignancy in children > 1 yr (< 1 yr neuroblastoma).
Peak age incidence 3-4 yr.
Arise from embryonic nephrogenic tissue from abnormal proliferation of metanephric blastema.
Consists of mixture of epithelial and stromal elements. (multicentric)
Associated with deletion of short arm chromosome 11.
More prevalent with associated anomalies as WAGR syndrome (Wilms tumor, anidridia,
genitourinary anomalies (Horseshoe kidney and mental retardation) and hemihypertrophy,
neurofibromatosis and Beckwith syndrome
M.C. presentation is painless smooth abdominal lump.
Triad of symptoms will be
Lump
Fever
Hematuria
Hypertension is also accompanying.
Usually U/L but in 5-10% may be B/L
Metastasize hematogenously to lung.
Prognostic factor is its histology.
Staging
I - Tumour limited to kidney and completely resected.
II - Tumour extends beyond the kidney but still resected completely.
III - Residual nonhematogenous tumour confirmed to abdomen after resection
positive lymph node and massive tumour spillage
IV - Hematogenous metastases.
V - B/L renal involment
Patient having a predominantly epithelial type of tumour has favoured history.
Egg shell pattern calcification on X-ray
USG and IVP helps in diagnosis.
6 D

7 A

8 A

9 D

10 B

11 A

Malignancies in Childhood / 100

Preoperative biopsy not done


Surgery is choice of therapy (Nephrectomy + lymph node sampling) chemo- and radiotherapy in endstage of disease).
Actinomycin D is drug of choice.
Bilateral Wilms tumour (10-14%) is treated by partial resection and chemotheraphy.
Botryoid Wilms tumour arises from metanephric blastemal remnants of renal pelvis and
treatement of choice is nephroureterectomy.

12. A 4-year-old male child has painless haematuria persisting for several months. On examination, a nonballotable
mass was detected in the right loin. The most likely diagnosis is:
(OPG5th/574) (UPSC/01)
(a) Renal tuberculosis
(b) Wilms tumuor
(c) Hydronephrosis
(d) Renal oxalate stone
13. Neuroblastoma true are A/E:
(OPG5th/573) (PGI 87)
(a) Associated with mild hypertension
(b) Rare tumour to undergo regression
(c) Commonest tumour of
adrenal cortex
(d) Treatment is excision and postoperative radiotherapy
14. A-1-year-old child presenting with abdominal mass and calcifications on X-ray is suggestive of:
(a) Wilms tumour
(b) Neuroblastoma
(c) Teratoma
(d) Rhabdomyosarcoma (OPG5th/573) (PGI 86, 93)
15. Regarding childhood malignancy the true statement is:
(OPG5th/573) (AIIMS 94)
(a) AML is the most common
(b) 10% of CML have spleen involvement
(c) Neuroblastoma is common in
the 1-4 years age group
(d) CNS involvement is the most common presentation of lymphoma

Neuroblastoma:
Most common malignant tumour of infancy (50% case occur in first 2 yrs.)
At birth, patient may be born with metastases to placenta.
Autosomal dominant inheritance in familial incidence.
Arise from neuroblast and sites are adrenal, sympathetic chain, retroperitoneal area, posterior
mediastinum or cervical area.
Have a high frequency of spontaneous regression.
Neuroblastoma is seen in patients with
Neurofibromatosis
Nesidioblastosis
Hirschsprungs disease
Pathologically has characteristic of Homer Wright pseudorossette.
Most common presentation is asymptomatic, irregular, firm, abdominal mass.
May also present with Horner syndrome; mediastinal mass or orbital metastasis with proptosis,
periorbital ecchymosis or bilateral black eyes (Panda eyes).
Excess catecholamine (principally norepinephrine) causes flusing, sweating and hypocalcemic,
watery diarrhoea (due to VIP).
Dancing eye syndrome (opsomyoclonus and nystagmus) paraplegia or myasthenia gravis may also
present.
Spinal involvement best detected by MRI.
Patient may have widespread subcutaneous nodule blue in colour
(Bluebeary muffin baby).
X-ray abdomen show stippled calcification
USG is helpful in diagnosis; CT helps in staging but standard mode of imaging is MRI.
MIBG isotope scan is sensitive method to know primary and secondary.
Site of metastasis
In Infant
- Liver
In older children
- Bone (M.C. skull and diaphysis of distal femur
and humerus).
Most accurate method of biochemical diagnosis is urinary excretion of VMA and HVA (metabolites of
catecholamine).
Evans stagingStage
I - Tumor confined to single organ and can be completely resected.
II - Tumor extending beyond the organ but not crossing midline; regional lymph
nodes on hemilateral side may be involved.
III - Tumour crossing beyond the midline may have B/L lymph nodes.
12 B

13 C

14 B

15 C

101 / Pediatric Buster


IV
IVs

- Distant metastasis.
- Patients who otherwise be of stage I or II but have metastasis confined to
liver; skin or bone marrow without cortical bone involvement (Prognosis is good).
TOC for stage I, II and IVs is surgery.
Neurone specific enolase and serum ferritin levels are important tumor and prognostic marker.
In advanced diseaseserum ferritin appears to be more critical prognostic factor.
Thoracic and head neck tumors do better than abdominal tumour.
Presence of opsoclonus or nystagmus, VIP, mature history, good nutrition, normal serum ferritin and
neuron specific enolase have good prognosis.
In stage III and IV, chemotherapy is used with cyclophosphamide and adriamycin.

16. Most common abdominal mass in neonates is:


(OPG5th/573) (AI 94)
(a) Wilms tumour
(b) Polycystic kidney
(c) Neuroblastoma
(d) Rhabdomyosarcoma
17. True in neuroblastoma are A/E:
(OPG5th/573) (UP 96)
(a) Proptosis
(b) Cafe-au-lait spot
(c) Hypertension
(d) Hepatomegaly
18. Which of the following is not true about neuroblastoma:
(OPG5th/573)
(DELHI/97)
(a) Intracranial metastasis causes skull hyperostosis
(b) IVP shows the renal pelvicalyceal displacement
(c)
Calcification occurs in 50% cases
(d) Usually arise in adrenal gland
19. An abdominal mass palpable in an asymptomatic neonate is not likely to be:
(OPG5th/573) (AIIMS 83)
(a) Polycystic kidney
(b) Multicystic renal dysplasia
(c) Neuroblastoma
(d) Distended urinary bladder
20. A 1-year-old child presenting with abdominal mass and calcification on X-rays is suggestive of:
(a) Teratoma
(b) Neuroblastoma
(c) Wilms tumour
(d) Rhabdomyosarcoma (OPG5th/573) (PAL 92)
21. Mother bathing infant finds a mass in left loin. On X-ray child has calcification in renal fossa, diagnosis is:
(OPG5th/573) (AIIMS/2K)
(a) Hepatoblastoma
(b) Renal cell carcinoma
(c) Neuroblastoma
(d) Glioblastoma and abdominoforme
22. Most common presentation of neuroblastoma:
(OPG5th/573) (AI 98)
(a) Lytic lesion in skull with suture diasthesis
(b) Lung metastasis
(c) Renal invasion
(d) Secondaries
in brain
23. Retinoblastoma is bilateral in:
(NEL 17th/1722) (PGI 81, AMU 86)
(a) 1% of cases
(b) 50% of cases
(c) 100% of cases
(d) 30% of cases
Retinoblastoma:
Most common ocular tumor in children
90% occur before the age of 5 yr; average age at diagnosis is 18 months.
Mostly unilateral with autosomal dominant inheritance.
Usual chromosomal anomaly is 13q 14 (Band 14 on long arm of chromosome 13).
M.C. presentation is leukocoria followed by strabismus
Specific histological findings are
- Flexner Wintersteiner rosette
- Fleurettes
M.C. presenting pattern of growth is endophytic characterize by calcification; exophytic pattern
causes retinal detachment.
Most widely used staging system is Reese-Ellisworth classification.
Sensitive and diagnostic test is CT scan.
M.C. cause glaucoma in patient with retinoblastoma is neovascularization.
TOC for retinoblastoma filling most of globe is enucleation with a long stump of optic nerve.
Other modalities of treatment arecryotherapy, photocoagulation; localized radioactive plaques
and systemic chemotherapy.
Cryotherapyant to equator; confined to sensory retina
Xenon are photocoagulationposterior pole
Systemic chemotherapycyclophosphamide
Enzyme markers of retinoblastoma are increased LDH; increased phosphoglucose isomerase.
16 C

17 D

18 A

19 C

20 B

21 C

22 A

23 D

102 / Pediatric Buster

M.C. secondary malignancy in retinoblastoma is osteogenic sarcoma (lung and breast cancer)
High prevalence of retinoblstoma is reported with trisomy 21
CEA may be elevated in retinoblastoma

24. Child with permeative bone lesions involving all body bones is likely to be suffering from:
(OPG/5th578) (AIIMS/2K)
(a) Neuroblastoma
(b) Histiocytosis X
(c) Metastases of Wilms tumor
(d) Osteomalacia
25. A child presents with seborrheic dermatitis, lytic skull lesions, ear discharge and hepatosplenomegaly; likely
diagnosis is:
(OPG5th/578) (AI/2001)
(a) Leukemia
(b) Lymphoma
(c) Histiocytosis X
(d) Septicemia
26. An eight-year-old boy presents with back pain and mild fever. His plain X-ray of the dorsolumbar spine reveals
a solitary collapsed dorsal vertebra with preserved disc spaces. There was no associated soft tissue shadow. The
most likely diagnosis is:
(OPG5th/578) (AI/03)
(a) Ewings sarcoma
(b) Tuberculosis
(c) Histiocytosis
(d) Metastasis

24 B

25 C

26 C

15

Collagen Vascular Disorders


and Genetics

1. True about systemic juvenile arthritis are A/E:


(OPG5th/581) (AI-98)
(a) Fever
(b) Hepatosplenomegaly
(c) Leucocytosis
(d) Rhumatoid factor positive
2. Chonic iridocyclitis is seen in:
(OPG5th/582) (AIIMS 97)
(a) Pauciarticular Rh factor ve
(b) Polyarticular Rh factor +ve
(c) Juvenile systemic Rh arthritis
(d) Pauciarticular Rh factor +ve
3. In juvenile RA not seen is:
(OPG5th/582) (AIIMS 98)
(a) Scleritis
(b) Uveitis
(c) Catract
(d) Keratopathy
4. All of the following are features of systemic juvenile rheumatoid arthritis except:
(a) Rash
(b) Fever
(c) Hepatosplenomegaly
(d) Uveitis
(OPG/5th/582)AIIMS/MAY /01)
5. Uveitis is associated with the following type of juvenile rheumatoid arthritis:
(OPG5th/582) (AI 90)
(a) Polyarticular
(b) Pauciarticular
(c) Systemic
(d) Rheumatoid factor positive
6. The initial drug of choice in the treatment of juvenile chronic arthritis is:
(OPG5th/583) (AI 88)
(a) Prednisolone
(b) Indomethacin
(c) Salicylates
(d) Penicillamine
7. Transient myeloproliferative disorder of the newborn is seen in association with:
(OPG5th/591) (AI/03)
(a) Turner syndrome
(b) Downs syndrome
(c) Neurofibromatosis
(d) Ataxia telangiectasia
8. Infant with Down syndrome can have:
(OPG5th/592) (JIPMER 86)
(a) Pyloric stenosis
(b) Annular pancreas
(c) Duodenal atresia
(d) Hirschsprungs disease
9. Infants born to mothers of advanced age have a greater risk of:
(OPG5th/592) (AIIMS 83)
(a) Down syndrome
(b) Trisomy 18
(c) Glycogenesis type II
(d) Marfans syndrome
10. Which is not a feature of Down syndrome:
(OPG5th/592) (AIIMS 91)
(a) Clinodactyly
(b) Pigmented birth marks
(c) Hypotonia
(d) Respiratory tract infections
DOWN SYNDROME
Commonest chromosomal disorder; trisomy 21
Extrachromosome is from mother (14/21 translocation)
Mental and physical retardation; flat facies, oblique palpebral fissures
Flat nose; short palate, furrowed protruding tongue, hypertelorism.
Hypotonia; brachycephalic with flat occiput.
Clinodactyly (hypoplasia of middle phalanx with a single flexion crease of 5th finger), 11 pairs of
ribs and positive metacarpal sign.
Simian crease; iliac index below 60; positive metacarpal sign.
Congenital heart diseaseM.C. being AV communicans, VSD and PDA
Brushfield spots; Distal triradius
CHDs are major cause of early mortality.
Lower respiratory, tract infection pose a threat to life. They are more prone to juvenile CML and
ALL.
1 D

2 A

3 A

4 D

5 B

6 C

7 B

8 B

9 A

10 B

104 / Pediatric Buster


11. All are features of Down syndrome except:
(OPG5th/592) (AI 94)
(a) Brushfields spots in iris
(b) Simian crease
(c) Mental retardation
(d) Hypertonicity
12. Diagnosis of Down syndrome is favoured by A/E:
(OPG5th/592) (AIIMS/99)
(a) Short femur
(b) Short humerus
(c) Decrease nuchal thickness
(d) 11 pairs of ribs
13. All the following are features of mongolism except:
(OPG5th/592) (KARNAT 96)
(a) Iliac index below 60
(b) 11 pairs of ribs
(c) Positive metacarpal sign
(d) orbital hypotelorism
14. Routine screening for Down syndrome should be done when the mother is above:
(OPG5th/592) (AIIMS 94)
(a) 30 years of age
(b) 35 years of age
(c) 40 years of age
(d) 45 years of age
15. A Down syndrome girl has 21/21 translocation of chromosomes. Her father is carrier of balanced translocation.
Risk of Down syndrome in next pregnancy is:
(OPG5th/592)(ALL INDIA/02)
(a) 100%
(b) 50%
(c) 25%
(d) 0%
16. True in Klinefelters syndrome is:
(OPG5th/594) (AI 98)
(a) Short stature
(b) Pituitary adenoma
(c) Subnormal intelligence
(d) Breast adenoma
17. A mentally retarded man with gynaecomastia and hypogonadism is most likely to be a case of:
(OPG5th/594) (AI 89)
(a) Male Turner syndrome
(b) Klinefelters syndrome
(c) XYY male
(d) Trisomy 21
Turners syndrome:
XO chromosomal constitution
Spontaneous abortions
X chromosome is most often maternal
Lymphedema of dorsum of hand and feet
Webbed neck, short stature, high arched palate
Medial tibial exostosis
Short 4th metacarpal and metatarsal
Congenital defects
- Horseshoe kidney, double renal pelvis
- Coarctation of aorta
- Perceptive hearing defects.
Streak ovariesincreased changes of dysgerminoma and gonadoblastoma
18. Lacunar skull is most frequently associated with:
(a) Diabetes mellitus
(b) Posterior fossa tumour
syndrome

(OPG5th/599) (AIIMS 79, 81)


(c) Encephalocele or meningocele
(d) Arnold-Chiari

Lacunar skull:
Bony defect in frontal and parietal region
Associated with meningocoele
Presence of craniosynostosis
19. Scapocephaly is due to premature closure of the:
(NEL17th/1992)(ORISSA 99)
(a) Coronal suture
(b) Metopic suture
(c) Sagittal suture
(d) Lambdoid suture
20. Porencephaly is due to:
(NEL17th/1987)(MP/2K)
(a) Dandy-Walker syndrome
(b) Cerebral infarction
(c) Fetal alcohol syndrome
(d) Trisomy 13
21. Phocomelia is:
(OPG5th/600) (AI 96)
(a) Reduplication of bones
(b) Defect in long bones
(c) Absence of brain
(d) Defect in short bones

11 D

12 C

13 B

14 B

15 A

16 C

17 B

18 C

19 C

20 A

21 B

16

Inborn Errors of
Metabolism

1. Phenylketonuria is diagnosed by:


(OPG5th/609) (AI 88)
(a) Guthrie test
(b) Ferric chloride test
(c) Gerhards test
(d) VMA in urine
2. Inborn error of metabolism associated with mental retardations is:
(OPG5th/609) (AI 92)
(a) Down syndrome
(b) Phenylketonuria
(c) Duchennes syndrome
(d) Mc Ardles syndrome
Phenylketonuria (PKU) (Autosomal recessive)
Low phenylalanine hydroxylase; synthesis impaired
Child normal in the first month of life. After some weeks manifestations are irritability, tremor
and convulsions.
Retarded development; blond hair, blue iris and fair skin musty body odor; Guthries test
(Phenylalanine is needed for growth of Bacillus subtilis)
Treatment: Restriction of phenylalanine in diet for life long
Mental retardation may be preventable
Children born to mothers with uncontrolled phenylketonuria have microcephaly and congenital
defects.
3. A children present with change in colour of hair + blue eye and mental retardation along with change of urine
in:
(a) Malonylurea
(b) Phenylketonuria
(c) Haemocystinuria
(d) Cystinuria
(OPG5th/609) (UP 97)
5th
4. Mother with uncontrolled phenylketonuria the child may have:
(OPG /609) (H/2198) (AIIMS 99)
(a) Mental retardation and cataract
(b) Microcephaly, cardiac defect and cataract
(c) Phenylketonuia
(d) Blond hair with mental retardation
5. Cystinuria is characterised by:
(OPG5th/611) (AMC 83)
(a) Generalised aminoaciduria
(b) Systemic acidosis
(c) Deposition of cystine crystals in renal tubular cells
(d) Recurrent urinary calculi
6. Which of the following inborn errors of metabolism is associated with mental retardation?
(a) Alkaptonuria
(b) Homocystinuria
(c) Pentosuria
(d) Galactosaemia
(OPG5th/611) (AIIMS 83)
Albinism:
Deficiency of Tyrosinase
Oculocutaneous albinism (autosomal recessive)
Oculoalbinism X-linked recessive.
Depigmented skin, white silky hair and pink of blue
Visual acuity decreased and nystagmus often present
Intelligence is normal.
1 A

2 B

3 B

4 D

5 C

6 B

Inborn Errors of Metabolism / 106


7. Large doses of pyridoxine are of value in some cases of:
(a) Phenylketonuria
(b) Homocystinuria
(c) Nonketotic hyperglycaemia

(OPG5th/611) (PGI 79, 81, 87)


(d) Ketotic hyperglycaemia

Alkaptonuria:
Deficiency of homogentisic acid oxidasealkaline urine
Ochronotic arthritis commonly involves shoulders and hiprenal stone and nephrosis.
Ascorbic acid is sometimes recommended.
Homocystinemia (Autosomal recessive)
Deficiency of cystathionine beta synthetase, methionine and homocysteine level in blood elevated
and in 50% cases in brain also.
Subluxation of lens (downwards and medially); recurrent thromboembolic episodes convulsions,
mental retardation and osteoporosis and lesions of lens, glaucoma and marfanoid habitus.
Activation of Hageman factor is observed.
Plasma level of folate is observed.
Treatment: large doses of pyridoxine for life long.
Presence of homocysteine in urine is detected by cyanide nitroprusside test.
Risk of coronary, peripheral and cerebral vascular disease
8. Characteristic clinical picture of Maple syrup urine disease:
(a) Choreoathetosis
(b) Flaccid paralysis
(c) Petit mal disorder

(OPG5th/612) (AIIMS 81), (AMC 83)


(d) Decerebrate rigidity

Maple syrup urine disease:


Deficiency of branched chain ketoacid dehydrogenase
Elevated levels of leucine, isoleucine and valine
Poor feeding, lethargy and vomiting within first week.
Ataxia, decerebate rigidity and convulsions.
Progressive nervous deterioration
Hypoglycemic attacks and mental retardation
Thiamine use may stabilize the enzyme
Cystinosis: Crystals deposited in cornea, bone marrow and leukocytes. Kidney is involed in severe
casesglomerular as well as tubular deterioration; polyuria, polydypsia and dehydration.
Cystinuria: (AR) Increase in renal clearance of urinary excretion cystine, lysine, arginine and ornithine
(COLA)
- Radio-opaque renal calculi
- Positive cyanide nitroprusside reaction in urine
- D-penicillamine and captopril useful
9. In an infant with galactosemia .....should be avoided:
(a) Gluten
(b) Egg
(c) Milk
(d) Rice
10. Dietary treatment of galactosemia:
(a) Must be continued life-long
(b) Can be relaxed after puberty
normal intellectual development started in the first week of life

(OPG5th/613)
(OPG5th/613) (JIPMER 81, DELHI 86)
(c) Involves delayed weaning
(d) Ensures
(e) Is best monitored by liver function tests

Galactosemia:
An autosomal recessive disorder manifests within few days after birth (after milk infestion)
Deficiency or galactose 1 phosphate uridyl transferase.
Persistence of jaundice, enlarged liver and oildrop cataract.
Child becomes mentally retarded.
Galactose free diet for whole life. Mental retardation is irreversible.
11. All of the following cause splenomegaly except:
(a) Gauchers disease
(b) von-Gierkes disease
7 B

8 D

9 C

10 A

11 B

(c) Tay-Sachs disease

(OPG5th/614) (AIIMS 83)


(d) Niemann-Picks disease

Miscellaneous / 107
12. Child with recurrent hypoglycemic attacks and hepatomegaly is likely to have:
(OPG5th/614) (JIPMER 93)
(a) von-Gierkes disease
(b) Neonatal diabetes
(c) Neonatal hepatitis
(d) Galactosemia
13. Which of the following symptoms are associated with gargoylism (Hurlers syndrome): (OPG5th/615) (ORISSA 99)
(a) Hepatosplenomegaly
(b) Corneal opacities
(c) Profuse nasal discharge
(d) All of the above
14. A 2-year-old with coarse facial features and hepatosplenomegaly showed tall QRS waves in the ECG.The
diagnosis is most likely to be:
(OPG5th/615) (AIIMS/NOV/01)
(a) Glycogen storage disease type II
(b) Hurlers disease
(c) Hunters disease
(d) Hemochromatosis
15. In Gauchers disease there is accumulation of....inside the cells:
(OPG5th/616) (TN 95)
(a) Galactosidases
(b) Sphingomyelins
(c) Glucosidases
(d) Cerebrosides
16. Which of the following is associated with chronic liver disease?
(OPG5th/618) (AIIMS 83)
(a) Hepatitis A
(b) Alpha-1 antitrypsin deficiency
(c) Malaria
(d) Infections mononucleosis
17. Which is not a feature of Wilsons disease in a child:
(OPG5th/618) (AIIMS 91)
(a) Fanconi syndrome
(b) Sensory changes
(c) Haemolytic anemia
(d) Chronic active hepatitis
18. All of the following are recognised features of Wilsons disease except:
(OPG5th/618) (UP 97)
(a) Psychological disturbances
(b) Increased ceruloplasmin levels
(c) Increased copper content of liver
(d) Histopathological features of chronic active hepatitis
Wilsons diseaseautosomal recessive disorder with accumulation of copper in cytoplasm of liver cells.
- Hemolysis
- Affection of proximal renal tubulesFanconi syndrome
- Kayser-Fleischer ring.
- Chronic active hepatitis
- CNS manifestation with cerebellar ataxia and parkinsonism; rigidity in older patient.
- There is no sensory involvement.
- Low serum levels of ceruloplasmin and reduction of total serum copper level.
- D-penicillamine is used as chelating agent.

12 A

13 D

14 B

15 D

16 B

17 B

18 B

17

Miscellaneous

1. Interstitial keratitis in an 8-year-old girl can occur in all of the following conditions except:
(a) Tuberculosis
(b) Leprosy
(c) Filaria
(d) Syphilis
(OPG5th/621) (UPSC 96)
2. Consider the following statements : Cherry red spot of the macula is seen in A/E: (OPG5th/625) (UPSC 98)
(a) Tay-Sachs disease
(b) Sandhoff disease
(c) Niemann-Pick disease
(d) Generalised gangliosides
(e)
None
3. In infants, the cause of blindness arising out of oxygen toxicity is:
(OPG5th/626) (UPSC 97)
(a) Degeneration of crystalline lens
(b) Growth of blood vessels into vitreous followed by fibrosis
(c) Damage
to cornea
(d) Enzyme defect in lens
4. In children atopic dermatitis is associated with increase in:
(OPG5th/634) (UPSC 96)
(a) Immunoglobulin A
(b) Immunoglobulin E
(c) Immunoglobulin G
(d) Immunoglobulin M
5. A 3-year-old child is seen with coma, weakness, salivation and one sided constricted pupil. The most probable
diagnosis is:
(OPG5th/644)
(a) Tetanus
(b) Organophosphorus poisoning
(c) Phenobarbitone poisoning
(d) Electrolyte imbalance
6. The major toxicity of acetaminophen overdose involves:
(OPG5th/646) (PGI 78, AIIMS 84)
(a) The CNS
(b) The heart
(c) Liver
(d) Acid-base metabolism
7. Child with a history of consumption of iron tablets is symptomatic. Treatment will be: (OPG5th/646) (PGI 2000)
(a) Desferrioxamine 100 mg/IV
(b) Stomach wash
(c) Exchange transfusion
(d) X-ray to delineate number
of capsules swallowed
8. Water tolerance test is useful in differentiating the short stature as a result of:
(NEL17th/15) (AIIMS 81)
(a) Hypopituitarism
(b) Gonadal dysgenesis
(c) Primordial dwarfism
(d) Mild hypothyroidism
9. Preference of use one hand (handedness) is evedent by:
(NEL17th/49) (KERALA 97)
(a) 6 months
(b) 1 year
(c) 2 years
(d) 3 years
10. Waking up at night screaming with fear, at 2.5 years age is generally a manifestation of:
(NEL 17th/82)(PGI 78, 79, 80)
(a) Organic illness
(b) Normal development pattern
(c) Castration anxiety
(d) Separation anxiety
11. Which of the following childhood disorder improves with increase in age:
(NEL17th/88)(MP/2K)
(a) Conduct disorder
(b) Emotional problems
(c) Temper tantrum
(d) Sleep disorder
12. A two-year old girl child is brought to the OPD with features of hand wringing stereotype movements, impaired
language and communication development, breath-holding spells, poor social skills and deceleration of head
growth after 6 months of age. The most likely diagnosis is:
(NEL17th/94)(AIIMS/03)
(a) Aspergers syndrome
(b) Retts syndrome
(c) Fragile X syndrome
(d) Cotard syndrome
13. Abstract thinking occur at the age of:
(NEL17th/103) (JIPMER 93)
(a) 2 years
(b) 3 years
(c) 9 years
(d) 12 years
14. All are complications of formula fed baby over human milk fed baby except:
(NEL17th/158) (JIPMER 95)
(a) Necrotising enterocolitis
(b) Otitis media
(c) Hypocalcaemia
(d) Vitamin K deficiency
1 C
14 D

2 E

3 B

4 B

5 B

6 C

7 A

8 A

9 D

10 D

11 D

12 B

13 C

109 / Pediatric Buster


(NEL17th/160) (PGI/86)

15. 90% of the milk is obtained by the breastfeeding infant in:


(a) 20 minutes
(b) 10 minutes
(c) 7 minutes
(d) 4 minutes

BREAST MILK
- Presence of long chain polyun saturated fatty acids (LC PUFA) in breast milk is responsible for
better visual and cognitive development of child.
- CMV, human T-cell lymphotropic virus type I, rubella, hepatitis B and herpes simplex virus have
been demonstrated in breast milk.
- Most of the milk is obtained early in the feeding (50% in the first 2 minute and 80-90% in the first
4 minute)
16. Sellicks manoeuvre is done for:
(NEL17th/291) (AIIMS 97)
(a) Prevent gastric aspiration
(b) CPR
(c) Assisted respiration
(d) Removal of foreign body
17. In pediatric advanced life support, intraosseous access for drug/fluid administration is recommended for pediatric
age of:
(NEL17th/293)(AIIMS/02)]
(a) <1 year age
(b) <5 years age
(c) <6 years age
(d) Any age
INTRAOSSEOUS ACCESS
Intraosseous line: Most common sites for insertion are the upper tibia, sternum and anterior superior
iliac spine.
18. Following is seen in cold injury to newborn baby except:
(NEL17th/336) (AI 99)
(a) Metabolic acidosis
(b) Sclerema
(c) Bradycardia
(d) Extreme shivering
19. Serum cholesterol is usually high in all the following conditions except:
(NEL17th/379) (AI 89)
(a) Nephrotic syndrome
(b) Primary biliary cirrhosis
(c) Kwashiorkor syndrome
(d) Diabetes mellitus
20. A child come with abdoman pain, arthralgria, hematuria, hypertension diagnosis is: (NEL17th/416) (JIPMER 98)
(a) Haemolytic uremic syndrome
(b) Porphyria
(c) Rheumatic fever
(d) Dengue
21. Which is a high risk neonate:
(NEL17th/452) (PGI 89)
(a) Less than 1500 gm
(b) Previous neonatal death
(c) Less than 37 weeks
(d) All
22. Splenic rupture in children is best diagnosed by:
(NEL/469) (AIIMS 89)
99
(a) Paracentesis
(b) Ultrasound
(c) Plain X-ray
(d) Tc scan
23. All of the following groups newborns are at an increased risk of hypoglycemia except: (NEL17th/508t)(AIIMS/02)
(a) Birth asphyxia
(b) Respiratory distress syndrome
(c) Maternal diabetes.
(d) Post term infant.
24. CMV infection in mother the newborn is likely to develop:
(NEL17th/523) (AIIMS 99)
(a) Blood dyscrasia
(b) Myocarditis
(c) Encephalitis
(d) Deafness
CMV disease: Transmitted through oropharyngeal secretion, urine, cervical and vaginal excretions,
spermatic fluid; breast milk and blood. Congenital CMV infection develops in approximately 5% of
infected fetuses and risk to foetus is greatest when the pregnant woman has primary CMV infection.
Only 1% foetus are infected in women with recurrence of previous infection
90-95% of infected foetuses are asymptomatic
The asymptomatic ones develop psychomotor, senosineural hearing loss, ocular or dental
abnormalities over the next years.
CF: Anemia, thrombocytopenic purpura, hepato-splenomegaly, neonatal hepatitis, jaundice,
meningoencephalitis; microcephaly, intracraninal calcification and chorioretinitis, petechial rash.
Diagnosis is by serum lgM and SGOT; confirmed by isolation of CMV.
25. Enzyme replacement therapy is available for the treatment of the following disorder: (NEL17th/464)(AIIMS/03)
(a) Gaucher disease
(b) Niemann-Pick disease
(c) Hunter syndrome
(d) Phenylketonuria.

15 D

16 A

17 C

18 B

19 C

20 B

21 D

22 B

23 D

24 C

25 A

Miscellaneous / 110
26. History of dislike for sweet food items is typically present in:
(NEL17th/477)AIIMS/03)
(a) Diabetes mellitus
(b) Glycogen storage disease
(c) Hereditary fructose intolerance
(d) Galactosemia.
HEREDITARY FRUCTOSE INTOLERANCE
Due to deficiency of fructose 1-6 bisphosphate aldolase (Aldolase B)
Rapid accumulation of fructose 1-phosphate causing severe toxic symptoms on exposure to fructose.
Gene present on chromosome 9q 13-32.
Patients are perfectly healthy and asmptomatic until fructose or sucrose (table sugar) is ingested
(usually from fruit or sweetened cereal).
Jaundice, hepatomegaly, vomitting, lethargy, irritability convulsions.
Reducing substance in urine.
Treatment is complete elimination of all sources of sucrose, fructose and sorbitol from diet, which
often patient learns to do himself.
27. Which one is not unfavourable for fetal development:
(a) Herpes
(b) Rubella
(c) Alcohol
(d) Tetracycline
28. Closure of the anterior fontanelle is delayed except: in:
(a) Down syndrome
(b) Osteogenesis imperfecta
(c) Hypogonadism
(e) Meningomyelocele
Large anterior fontanalle
Achondroplasia
Apert syndrome
Athyrotic hypothyroidism
Cleidocranial dysostosis
Congenital rubella syndrome
Hallermann-Streiff syndrome
Hydrocephaly
Hypophosphatasia

(NEL

17th/524)

(AIIMS 89)

(NEL17th/525) (PGI 88)


(d) Hypothyroidism

Intrauterine growth retardation


Kenny syndrome
Osteogenesis imperfecta
Prematurity
Pyknodysostosis
Russell-Silver syndrome
13-, 18-, 21-trisomies
Vitamin D deficiency rickets

29. Breastfeeding is contraindicated if the mother is taking:


(a) Propranolol
(b) Broad spectrum antibiotics
(c) Sulfonyl ureas

(NEL
(d) Insulin

17th/530)

(PGI 89)

Contraindication of breastfeeding
Mothers with septicemia, active tuberculosis, typhoid, breast cancer or malaria should not breastfeed.
30. Asphyxial injury in a term baby is characterized by all except:
(a) Seizures
(b) Differential hypotonia (lower limbs>upper limbs)
in clearing oral secretions.

(NEL17th/566)(AIIMS/03)
(c) Altered sensorium
(d) Difficulty

31. In asymmetrical IUGR which organ is not affected?


(NEL17th/557) (UPSC 95)
(a) Subcutaneous fat
(b) Muscle
(c) Liver
(d) Brain
32. What is the venous hematocrit level at which you will diagnose polycythemia in a newborn?
(a) 55%
(b) 60%
(c) 65%
(d) 70%
(NEL17th\606)(UPSC/03)
Polycythemia
Polycythemia in newbornDefined as central hematocrit of 65% or more
- Incidence of neonatal polycythemia is increased
(i) At high altitude
(ii) In post mature foetus
(iii) In small for gestational age
(iv) After delayed clamping of umbilical cord.
26 C

27 A

28 C

29 B,C 30 B

31 D

32 C

111 / Pediatric Buster


(v)
(vi)
(vii)
(viii)
(ix)
(x)

Infants of diabetic mother


Trisomies 13, 18, 21
Adrenogenital syndrome
Neonatal Graves disease
Hypothyroidism and in
Beckwith-Wiedemann syndrome

33. A 2-day-old newborn baby presented with microcephaly, macroglosia, visceromegaly and a blood glucose level of
20 mg/dl. What is the most likely diagnosis?
(NEL17th/616)(ORISSA 98)
(a) Prader-Willi syndrome (b) Beckwith-Wiedemann syndrome (c) Werner syndrome (d) Cockayne syndrome
34. Lymphocyte transformation is a sensitive test most useful in identification of:
(NEL17th/650-51) (PGI 79, AIIMS 81)
(a) Drug reaction
(b) Contact dermatitis
(c) Pollen allergy
(d) food allergy
35. All true of Hair cartilage hypoplasia syndrome except:
(NEL17th/698) (AIMS 97)
(a) Short strature dwarf
(b) Sparce hair
(c) Neutropenia
(d) T cell dysfunction
36. Most common opportunistic infection in children with neutropenia is:
(NEL17th/722) (JIPMER 93)
(a) Gram-negative bacilli
(b) Streptococci
(c) Staphylococci
(d) Pneumococci
37. A 3-year-old boy is brought to the casualty by his mother with progressive shortness of breath for 1 day. The
child has history of bronchial asthma. On examination, the child is blue, grasping and unresponsive. What will
you like to do first?
(NEL17th/772)(AIIMS/02)
(a) Intubate
(b) Administer 100% oxygen by mask
(c) Ventilate with bag and mask
(d) Administer
nebulised salbutamol
38. A 5-yr-old boy a known patient of bronchial asthma, present with cough wheezing and breathlessness.
Examination show respiratory rate 48/minute, pulsus paradoxus and bilateral rhonchi. The most appropriate
immediate treatment is:
(NEL17th/772) (UPSC/02)
(a) Intravenous theophylline and corticosteroids
(b) Intravenous theophylline and nebulized salbutamol
(c) Nebulized salbutamol and ipratropium
(d) Nebulized salbutamol and intravenous corticosteroids
39. Most common cause of chronic arthritis in children:
(NEL17th/799) (PGI 2000)
(a) Rheumatic fever
(b) Rheumatoid arthritis
(c) JRA
(d) Rheumatic arthritis
40. Blood and mucus in stools are seen with all except:
(NEL17th/799) (AI 89)
(a) E. histolytic
(b) Shigella
(c) E. coli
(d) V. cholera
41. Common features of sarcoidosis in childhood include:
(NEL 17th/822) (AIIMS 78, PGI 79, 81)
(a) Parotitis
(b) Erythema nodosum
(c) A high incidence of spontaneous remission
(d) All are true
42. A 6-year old boy is brought to emergency with purpuric and petechial rash all over the body. He appears
semiconscious and febrile. Total leucocyte count is 20000 per mm3 with 88% polymorphs. Platelet count is
1.2 lac/mm3. The most likely diagnosis is:
(NEL17th/897)(UPSC/04)
(a) Idiopathic thrombocytopenic purpura
(b) Meningococcemia
(c) Staphylococcal
(d) Dengue hemorrhagic
fever
43. Bad prognosis in meningococcal infection is indicated by:
(NEL17th/898)PGI 88)
(a) Hypertension
(b) Rash more than 3 days
(c) Leucocytosis
(d) Leucopenia
44. Which is not true of congenital syphilis:
(NEL 17th/979) (JIPMER 92)
(a) Severe periostitis
(b) VDRL negative indicates absence of disease
(c) Osteochondritis occurs after 6
month
(d) Bone within bone appearance
45. Enamel hypoplasia is associated with all except:
(NEL17th/1038) (JIPMER 79, AIIMS 81)
(a) Cerebral palsy
(b) Nephrosis
(c) Rubella
(d) Gardners syndrome
46. Rota virus immunity in children occur above....years:
(NEL 17th/1081) (AMC 87 AIIMS 84)
(a) 2
(b) 5
(c) 7
(d) 10
47. A 6-month-old infant presents to the diarrhea clinic unit with some dehydration. The most likely organism
causing diarrhea is:
(NEL17th/1082)(AIIMS/03)
(a) Entamoeba histolytica
(b) Rotavirus
(c) Giardia lamblia
(d) Shigella
33 B
46 B

34 A
47 B

35 C

36 C

37 A

38 D

39 D

40 D

41 D

42 B

43 D

44 B

45 B

Miscellaneous / 112
48. Fatty change occurs in A/E:
(NEL17th/1129)(UP 96)
(a) Kwashiorkor
(b) Marasmus
(c) Miliary tuberculosis
(d) Alcoholic liver disease
49. Cirrhosis with portal hypertension is seen in all of the following except:
(NEL17th/1155) (AI 91)
(a) Cystic fibrosis
(b) Alpha-1 antitrypsin deficiency
(c) Congenital hepatic fibrosis
(d) Wilsons disease.
50. The following is not a feature of Pierre-Robin syndrome:
(NEL17th/1209) (JIPMER 91)
(a) Hearing defect
(b) Coloboma irides
(c) Respiratory distress
(d) Mandibular hypoplasia
51. Primary symptom of hiatus hernia during the newborn period is:
(NEL 17th/1221) (AIIMS 80, UPSC 86)
(a) Anorexia
(b) Anaemia
(c) Vomiting
(d) Distended abdomen
52. Lactase deficiency in children is characterised by:
(NEL 17th/1268) (UP 97)
(a) Dumping syndrome
(b) Abdominal bloating, cramps and diarrhoea
(c) High pH of the stool
(d) Low fatty acid levels in blood
53. Which of the following does not cause gastroenteritis in infants:
(NEL 17th/1278) (UP 95)
(a) Adenovirus
(b) Calcivirus
(c) Reovirus
(d) Rotavirus
54. What is true of canals of Lambert in the alveolar spaces of the lung:
(NEL 17th/1374) (PGI 78, 80)
(a) Bronchi alveolar connection
(b) Prevent atelectasis
(c) Delay in collapse
(d) All
55. Percentage of HbF in a 6-month-old infant is:
(NEL17th/1376) (KERALA 97)
(a) 10
(b) 30
(c) 50
(d) 60
56. Acanthocytosis is usually not associated with:
(NEL17th/1395) (PGI 80, AIIMS 81)
(a) Steatorrhoea
(b) Failure to thrive
(c) Abnormal stools
(d) Abnormally high plasma cholesterol
57. A 3-month-old infant presents with intermittent respiratory strider since 10 days of age the most likely
diagnosis is:
(NEL 17th/1409) (AIIMS 95)
(a) Laryngomalacia
(b) Tracheoesophageal fistula
(c) Laryngotracheobronchitis
(d) Neoplasm
58. Primary treatment of choice for hepatoma in childhood is:
(NEL17th/1473) (JIPMER 81, AMC 82)
(a) Vitamin B12
(b) Surgery
(c) Chemotherapy
(d) Radiation therapy
59. All cause nephrotic syndrome except:
(NEL17th/1500) (TN/89)
(a) Minimal lesion glomerulonephritis
(b) Membranous glomerulonephritis
(c) Post-streptococcal glomerulonephritis
(d) Focal glomerulosclerosis
Post-streptococcal acute glomerulonephritis :
Caused by strains 4 and 12 causing pharyngitis and type 49 causing pyoderma.
Occurs in school age children susually 1-2 wk after streptococcal infection.
Typical example of immune complex disease.
Maximum endocapillary proliferation and subepithelial humps of Ig G and complement is seen
pathologically.
Rapid edema; smoky urine (haematuria) and red casts are diagnostic.
Hypertension is usually present.
Pus cells are also present in urine.
ESR in increased and ASO titres are elevated.
Serum complement C3 is low and comes to normal within 5-6 wk.
Atypical presentation may be convulsions.
LVF or
ARF
95% recover spontaneously or with conservative therapy.
Excellent prognosis in childhood.
60. Primary antiphospholipid syndrome is characterised by all the following clinical features except:
(NEL 17th/1668)(UPSC 96)
(a) Recurrent foetal loss
(b) Arterial and venous thrombosis
(c) Foetal cardiac abnormalities
61. Ankle colonus of 5 contractions in a 2-months-old infant:
(NEL17th/1671) (AIIMS 79)
(a) Indicates neurological disease
(b) Often normal
(c) Abnormal at any age
(d) Normal in early years
of language development

48 B
61 B

49 A

50 B

51 C

52 B

53 C

54 D

55 A

56 B

57 A

58 B

59 C

60 C

113 / Pediatric Buster


62. Child with a gammaglobinaemia with respiratory tract infection and diarrhoea is most likely having infection:
(a) Coxsackie virus
(b) Rotavirus
(c) Shigella
(d) None
(NEL 17th/1682) (TN 97)
63. Which does not predispose to malignancy:
(NEL 17th/1683)(JIPMER 91)
(a) Turners syndrome
(b) Cri-du chat syndrome
(c) Down syndrome
(d) Klinefelter syndrome
64. Prenatal diagnosis is possible for all of following except:
(NEL17th/1748) (AI 96)
(a) Sickle cell trait
(b) -thalassaemia
(c) Ectodermal dysplasia
(d) Duchennes muscular dystrophy
65. Nephritic syndrome may occur secondary to all of the following situation except:
(NEL 17th/1757) (AI 88)
(a) Malaria
(b) Poststreptococcal glomerulonephritis
(c) Kanamycin administration
(d) Penicillamine administration
66. In chronic renal failure:
(NEL 17th/1771) (AIIMS 83)
(a) Urine output is more than 3 lit per day
(b) Urine concentration is decreased
(c) Sodium conservation
is good
(d) Polycythaemia is present
67. Most common aetiology of hepatic metastasis in children is:
(NEL 17th/1725) (PGI 78, 81)
(a) Astrocytoma
(b) Neuroblastoma
(c) Melanoma
(d) Retinoblastoma
68. Most common posterior fossa tumour in children:
(NEL17th/1732) (AI 97)
(a) Medulloblastoma
(b) G. multiforme
(c) Astrocytoma
(d) Meningioma
69. A 3-year-old boy is detected to have bilateral renal calculi. Metabolic evaluation confirms the presence of marked
hypercalciuria with normal blood levels of calcium, magnesium, phosphate, uric acid and creatinine. A diagnosis
of idiopathic hypercalciuria is made. The dietary managemnt includes all, except: (NEL17th/1825) (AIIMS/03)
(a) Increased water intake
(b) Low sodium diet
(c) Reduced calcium intake
(d) Multifactorial inheritance.
70. Management of inappropriate secretion of antidiuretic hormone consists of all of the following except:
(NEL 17th/1857) (AI 91)
(a) Restriction of fluids
(b) Desmopressin nasal spray
(c) Hypertonic saline
(d) Furosemide
71. All are causes of precocious puberty in a female except:
(NEL 17th/1863) (AI/99)
(a) Craniopharyngioma
(b) Head injury
(c) McCune Albright syndrome
(d) Hyperthyroidism
72. A newborn with a goitre large enough to cause dyspnoea is best treated with:
(a) Sulfonamides
(b) Tracheostomy
(c) T3
(d) Iodides
(NEL 17th/1882) (AIIMS 81, 84)
17th
73. Treatment of choice in childhood thyrotoxicosis: (NEL
/1886) (AIIMS 92)
(a) Radio iodine
(b) Lugols iodine
(c) Carbimazole
(d) Surgery
74. A child presents with _____, antimongloid slant, pulmonary stenosis, short stature and undecended testis. Most
likely diagnosis is:
(NEL17th/1925) (AIIMS 94)
(a) Klinefelters syndrome
(b) Noonans syndrome
(c) Hypoparathyroidism
(d) XYY
75. Which is a feature of dawn phenomenon:
(NEL17th/1963) (AIIMS)
(a) Early morning hypoglycaemia
(b) Early morning hyperglycaemia
(c) Breakfast hypoglycaemia
(d)
Post-prandial coma
76. The normal concentration of protein CSF at 4 weeks may be as high as:
(NEL17th/1980) PGI 80
(a) 500 mg%
(b) 125 mg%
(c) 50 mg%
(d) 25 mg%
77. One of the major problems associated with spina bifida with meningomyelocele is:(NEL 17th/1985) (AIIMS 83)
(a) Hydrocephalus
(b) Deafness
(c) Cataract
(d) Encephalitis
78. A 5-year-old child is admitted with headache, vomiting and difficulty in walking. Physical findings include truncal
ataxia, papilloedema and left lateral rectus palsy. No finger-to-nose ataxia could be detected on the left or right
side. The most likely diagnosis is:
(NEL17th/1990) (UPSC/99)
(a) Dandy-Walker syndrome
(b) Syringobulbia
(c) Arnold-Chiari malformation
(d) Medulloblastoma
79. Chronic lead encephalopathy in a child has following except:
(NEL17th/2010) (AIIMS 98)
(a) Thrombosis
(b) Reactive hyperglycaemia
(c) Hyperosmolarity
(d) Acidosis
80. Which one of the following statements about Guillain-Barre syndrome is correrct: (NEL17th/2080)(UPSC/01)
(a) Slow progressive musle weakness, more marked distally
(b) 80% of patients develop residual neurological
disability in 3 to 6 months
(c) Increase in CSF albumin is disproportionate to the rise in number of cells
(d) Facial or bulbar weakness is very rare
81. One of the following factors is not considered a high risk criterion for development of deafness:
(NEL 17th/2130)(KARN 94)
(a) Birth asphyxia (b) Bacterial meningitis (c) Congenital (TORCH) infections (d) Direct hyperbilirubinaemia
62 B
75 B

63 B
76 B

64 C
77 A

65 C
78 A

66 B
79 B

67 B
80 C

68 C
81 D

69 C

70 B

71 D

72 C

73 C

74 B

114 / Pediatric Buster


82. Harlequins skin change in the newborn is seen in:
(NEL17th/2201) (JIPMER 93)
(a) Autonomic dysfunction
(b) Icthyosis
(c) Septicaemia
(d) Polycythaemia
83. Gait changes are seen in:
(NEL17th/2253)(PGI 88)
(a) Polio
(b) Duchenne muscular dystrophy
(c) Proximal muscle atrophy
(d) Slipped epiphysis
(e) all
84. A child presents with a history of scorpion sting. He is having increased sweating. What is the best next step:
(NEL17th/2390)(JIPMER/2K)
(a) Lytic cocktail
(b) Atropine
(c) Antivenom
(d) Local xylocaine infiltration
85. In infants, extracardiac digitalis toxicity is manifested by:
(NEL17th/2450) (TN 89)
(a) Vomiting
(b) Blurring of vision
(c) Convulsions
(d) Rashes

82 B

83 E

84 A

85 A

18

Self-assessment

1. Lactase deficiency affects:


(UP 98)
(a) Milk ingestion
(b) Cereals
(c) Meat
(d) Soya bean
2. All of the following are associated with increased bone age except:
(KERALA/98)
(a) Congenital adrenal hyperplasia
(b) Gonadal tumours
(c) Turners syndrome
(d) Polyostotic dysplasia
3. Visceromegaly, microcephaly with blood sugar below 20 mg/dl with sinusitis is:
(ORISSA/98)
(a) Prader Willi syndrome (b) Beckwith Wiedemann syndrome (c) Alport syndrome (d) Cockyne syndrome
4. The characteristics of caput succedaneum include all of the following except:
(AIIMS 81)
(a) Crosses midline
(b) Crosses the suture line
(c) Does not disappear within 2-3 days
(d) Soft
5. The antenatal diagnosis of all of the following conditions is feasible except:
(UPSC 95)
(a) Huntingtons chorea
(b) Down syndrome
(c) Severe combined syndrome
(d) None
6. Cephalhematoma usually disappears within:
(JIPMER 80, UPSC 82)
(a) 3-5 months
(b) 2-5 weeks
(c) 3-5 weeks
(d) 5-7 weeks
Caput succedaneumDue to stagnation of fluids in layers of scalp below girdle of contact; not limited
by suture line; disappears in 24 hours of birth. Usually seen after rupture of membrane.
CephalohaematomaSub-periosteal haemorrage due to rupture of emissary vein developed on
parietal or temporal boes either due to forceps, vaccum extraction or prolonged labor. Soft, pulsatile
does not cross the suture line and have well defined margins. Never present at birth but develops in
12-24 hours and rabsorbed in 6-8 weeks.
7. Birth injury to spinal roots of brachial plexus usually results in:
(a) Apnoea
(b) shock
(c) Paralysis of arm
(d) None of the above
8. General incidence of congenital anomalies in newborns is:
(a) 1%
(b) 3%
(c) 11%
(d) 15%
9. Stills disease is characterised by:
(a) Rheumatoid factor positive
(b) Neutropenia
(c) Renal involvement

(DNB 87, PGI 81)


(JIPMER 93)
(AIIMS 96)
(d) Maculopapular rash

Juvenile rheumatoid arthritis (JRA) (Stills disease) : Defined as arthritis of one or more joints
with onset before age of 16 yr and persisting for at least 6 wk.
Types
(a) Pauciarticular
(b) Polyarticular
(c) Systemic (with fever and rash)
1 A

2 C

3 B

4 C

5 D

6 A

7 C

8 B

9 B

Self-assessment / 116
PauciarticularMost frequent type of JRA
Type Imore common in females
- Age of onset 5 yr.
- Knees are more commonly affected; also involved are ankles and elbows, small joints escape.
- Iridocyclitis; secondary glaucoma and cateract may occur.
- Antinuclear antibodies are positive and rheumatoid factor is negative (HLA DR -5 DR W-8)
Type IImore in boys older than 8 years
- Hip girdle usually involved.
- Sacroileitis may develop.
- ANA and rheumatoid factor -ve.
- HLA B27.
PolyarticularGirls affected. Loss of lumbar lordosis, diminished mobility of spine and low back pain.
Type I
Theumatoid factor positive; arthritis is more severe. HLA DR 4
Type II
Rheumatoid factor negative; PIP involved; flexion deformities. Hip development is
affected due to lack of mobility.
Systemic JRAMore in boys
Intermittent fever. Characterisitic evanscent maculopapular rash with central clearing
- Pericarditis and interstitial lung disease
- Lymphadenopathy and splenomegaly
- ANA +ve RA -ve
Amyloidosis develops in long-standing disease.
Prognosis is relatively better in sero-negative polyarticular JRA.
Naproxen or indomethacin is drug of choice.
10. True in artificial feeding are A/E:
(UP 96)
(a) Eczema
(b) Jaundice
(c) Tetany
(d) Enteropathic necrotising
11. Ovulation and ability to become pregnant usually follow menarche by about:
(AIIMS 81)
(a) 5-6 months
(b) 2-4 months
(c) 1 month
(d) 12-24 months
12. Sneezing in a neonate indicates:
(a) Nasal allergy
(b) Syphilis
(c) Nasopharyngitis
(d) A normal phenomenon
13. Neonatal seizure with cataract is seen in A/E:
(UP 97)
(a) Galactosaemia
(b) Toxoplasma
(c) Tay-Sachs disease
(d) Premature with birth asphyxia
14. A 6-month-infant exclusively on breastfeeding develops deficiency of:
(UP 96)
(a) Iron
(b) Vitamin C
(c) Vitamin A
(d) Calcium
15. Commonest cardiac rhythm disturbance encountered in children is:
(KARNATAKA/98)
(a) Supraventricular tachycardia
(b) Ventricular tachycardia
(c) Sinus tachycardia
(d) Junctional rhythm
16. Lowes syndrome is characterised by the following except:
(AIIMS 84)
(a) Hypophosphataemic rickets
(b) Undescended testes
(c) Defect in the CNS and eyes
(d) Aminoaciduria
17. Continuous pharyngeal suction in an asphyxiated infant is not recommended because it may lead to:
(a) Vomiting
(b) Seizures
(c) Bradycardia
(d) All of the above
(AIIMS 83)
18. Which is an abnormal finding in a neonate:
(PGI 89)
(a) Glycosuria
(b) Bacteriuria
(c) WBCs in urine
(d) Hyperbilirubinaemia
19. Cardiomyopathy is a recognised finding in all except:
(PGI 81, BHU 87)
(a) Friedreichs ataxia
(b) Transfusion haemosiderosis
(c) Cystic fibrosis
(d) Muscular dystrophy
20. The most common cause of convulsions in a newborn on the first day:
(AI 92)
(a) Hypocalcaemia
(b) Hypoglycaemia
(c) Anoxia
(d) Head injury
21. Which of the following drugs can cause dyspnoea in children:
(PGI 79, AIIMS 79, 82)
(a) Paracetamol
(b) Salicylates
(c) Ampicillin
(d) Propranolol
22. Increased learning disabilities are seen in:
(AIIMS/99)
(a) PKU
(b) Down syndrome
(c) Cretinism
(d) Attention deficit

10 B

11 D

12 D

13 C

14 A

15 A

16 B

17 C

18 B

19 C

20 C

21 B

22 D

117 / Pediatric Buster


23. Pneumatocele is caused by:
(AI 96, 98)
(a) Staphylococcus
(b) Streptococcus
(c) E. coli
(d) P. carinii
24. Parents of a 3 years child come with the complaint that their daughter is not eating, her weight is 12 kg and
height is 88 cm, child is active and mobile. The advice to parents will be:
(AIIMS/99)
(a) Force her to eat
(b) Multivitamin tonic
(c) Dont bother for food
(d) Urinary examination
25. Antidote for muscarine poisoning is:
(JIPMER 80, DNB 91)
(a) Sodium benzoate
(b) Thiocyanate
(c) Atropine
(d) Bal
26. In the case of CNS relapse in AML, chemotherapy would consist of intrathecal
(UPSC 97)
(a) Methotrexate
(b) Methotrexate + cytosine arabinoside
(c) Prednisolone
(d) Adriamycin
27. Boy comes with recurrent hematuria, URTIS had two similar episodes when he was 2 months and 5 months old;
true is:
(a) Complement level
(b) Decreased ASO titre
(c) Focal and segmental proliferative GN
(d) Decrease
in renal function with progressive attacks
28. Antibodies to one of the following infection is not transmitted to child:
(PGI 96)
(a) Measles
(b) Pertussis
(c) Diphtheria
(d) Polio
29. The typical feature of pericardial involvement on physical examination is:
(AIIMS 81, 83)
(a) Friction rub
(b) Cardiac temponade
(c) Paradoxical pulse
(d) Venous distention
30. Which is contraindicated in treating a child with familial hypercholesterolaemia?
(AIIMS 80, DELHI 80)
(a) Cholestyramine
(b) Oestrogen
(c) Nicotinic acid
(d) Thyroxine
31. Which of the following lesions is of least clinical significance?
(PGI 79)
(a) Single atrium
(b) Ostium premium defect
(c) Incompetent patent foramen
(d) Secundum atrial septal defect
32. Greater part of hilar shadows are the shadows of the:
(PGI 79 AIIMS 80)
(a) Bronchi
(b) Lymph nodes
(c) Lymphatic vessels
(d) Pulmonary arteries
33. Lethargy in a 2-day-old infant most strongly suggests:
(JIPMER 80, PGI 82)
(a) Infection
(b) Imperforate anus
(c) Congenital heart disease
(d) Hyaline membrane disease
34. Scratch tests are best read within:
(JIPMER 81 PGI 82)
(a) 1 hour
(b) 4 hours
(c) 5 minutes
(d) 15-20 minutes
35. A school age child is most apt to have fears about:
(PGI 80, DNB 88)
(a) Snakes
(b) High places
(c) Loud noises
(d) Death or injury
36. Bilirubin is not present in urine in neonatal jaundice because:
(AIIMS/99)
(a) It is lipophilic
(b) Bind to glucuronic acid
(c) Renal tubules are not developed
(d) Deposited in brain
37. Cerebral oedema is associated with all, except:
(NIMHANS 81, 84)
(a) Craniostenosis
(b) Lead intoxication
(c) Vitamin A overdosage
(d) Adrenal corticosteroid therapy
38. Maximum development of brain occurs during:
(AIIMS 79,81,83)
(a) Infancy
(b) Adolescence
(c) First trimester of pregnancy
(d) School going year
39. Most frequently occurring emotional disorder of children:
(PGI 81)
(a) Schizophrenia
(b) Neurosis
(c) Mental retardation
(d) Autism
40. Ammoniacal urine may result in:
(PGI 80, 82)
(a) Urethritis
(b) Pyelonephritis
(c) Hydronephrosis
(d) Metal ulcers
41. Drug of choice for an attack of familial periodic paralysis is:
(JIPMER 81, AMU 88)
(a) ACTH
(b) Potassium chloride
(c) Calcium chloride
(d) Adrenaline
42. Splenectomy is least useful in:
(AI 89)
(a) Congenital elliptocytosis
(b) Thalassaemia major
(c) Congenital spherocyte hemolytic anemia
(d) Hereditary nonspherocytic hemolytic anemia
43. The APT test is:
(PGI 81)
(a) Semiquantitative test for foetal haemoglobin
(b) Semiqunatitative test for lead poisoning
(c) Screening
test for haemoglobin
(d) None
44. The major advantage of alternate day prednisolone therapy for conditions such as asthma:
(PGI 81 AP 89)
(a) There is less adrenal suppression
(b) More effective
(c) More convenient
(d) Less expensive
23 A
36 A

24 C
37 A

25 C
38 A

26 A
39 B

27 D
40 D

28 B
41 D

29 A
42 D

30 B
43 A

31 C
44 A

32 B

33 A

34 B

35 D

Self-assessment / 118
45. Following a scratch test a reaction of a moderate wheal and flare, without pseudopods. The test is interpreted
as:
(a) 4 plus
(b) 3 plus
(c) 2 plus
(d) 1 plus
(PGI 80, 81)
46. A child presented with effortless vomiting, we will come across all of the following biochemical findings except:
(a) Decrease urine potassium
(b) Decrease serum potassium
(c) Increase serum
(AIIMS/99)
bicarbonate
(d) Increase pH of blood
47. Chromatin positive seminiferous tubule dysgenesis had been seen in association with:
(AIIMS 78, 85, 87)
(a) Hypothyroidism
(b) Hypopituitarism
(c) Adrenal hyperplasia
(d) Down syndrome
48. Pericardial knock is characteristically heard in which of the following heart lesion? (PGI 81, KERALA 86)
(a) Pericardial effusion
(b) Rheumatic carditis
(c) Cardiac myxoma only
(d) Toxic myocarditis
49. What is the normal average stool sodium content in a newborn who is the fed on breast milk:
(PGI/90)
(a) 19 mEq/L
(b) 26 mEq/L
(c) 22 mEq/L
(d) 27 mEq/L
50. Which of the following would be considered normal in a 4-year child?
(PGI 80)
(a) Tics
(b) Hyperexcitability
(c) Anxiety
(d) Hyperkinesis
51. Which adjustment mechanism is probably most commonly used by children?
(PGI 80, NIMHANS 83)
(a) Aggression
(b) Fantasy
(c) Projection
(d) Somatisation
52. Ectopic ureter may be frequently associated with:
(JIPMER 81, AMU 89)
(a) Oliguria
(b) Dysuria
(c) Bilateral hydroureter
(d) Paradoxical incontinence
53. Bat wing appearance in X-ray is seen in:
(KERALA 94)
(a) Bronchial asthma
(b) Cardiogenic pulmonary oedema
(c) Mitral stenosis
(d) Pulmonary embolism
54. Which of the following is not a complication of smoking in pregnancy:
(JIPMER 95)
(a) Congenital malformation
(b) Intrauterine growth retardation
(c) Poor weight gain
(d) Foetal death
55. If present of definitive pathological significance in sputum is:
(AI 95)
(a) Mycobacterium tuberculosis
(b) Klebsiella
(c) Streptococcus
(d) Staphylococcus
56. In the labratory, for amplification of a single DNA chain polymerase chain reaction was allowed to continue for
3 cycles. At the end of this the number of DNA chains would be:
(AIIMS/NOV/01)
(a) 4
(b) 8
(c) 16
(d) 3
57. Micronodular cirrhosis is caused by:
(TN 95)
(a) Alcoholic cirrhosis
(b) Indian childhood cirrhosis
(c) Biliary atresia
(d) Wilsons disease
58. An infant is crying excessively even after being given good feed. He is passing a large quantity of urine and repeatedly
getting dehydrated. Urine examination shows no proteinuria and a specific gravity of 1004. This is a case of:
(UPSC 96)
(a) Diabetes mellitus (b) Diabetes insipidus (c) Congenital nephritic syndrome (d) Protein losing enteropathy
59. In children with cerebral oedema, which one of the following corticosteroids will be effective?
(UPSC 96)
(a) Hydrocortisone
(b) Prednisolone
(c) Dexamethasone
(d) Betamethasone
60. All of following statements are true except:
(DELHI 96)
(a) At 2-3 years of age, lymphocytes from 6-70% of total leucocytes
(b) After puberty, neutrophils replace
lymphocytes as the predominant leucocyte
(c) Lymphocytes are an important protective against influence
childhood infection
(d) There is no change in proportion of lymphocytes with age
61. The maximum number of deaths in children occur in following age group:
(DELHI 96)
(a) 2-5 years
(b) 1-2 years
(c) first 7 days
(d) 6-12 months
62. Shakir tape is used for:
(DELHI 96)
(a) Measurement of height
(b) Measurement of length of infant
(c) Measurement of midarm circumference
(d) Measurement of skin pad thickness

Shakirs tape to measure nutritional status by measuring mid arm circumferances, having 3 zones
1. Green (> 13.5 cm) Normal
2. Yellow (12.5-13.5 cm) Border line malnutrition
3. Red (< 12.5 cm) Severe malnutrition.

63. Sitting height is equal to:


(a) Head circumference
(b) Chest circumference
45 C
58 B

46 A
59 C

47 D
60 D

48 A
61 C

49 A
62 C

50 D
63 D

(c) Upper segment


51 A

52 D

53 B

(ASSAM 95)
(d) Crown heel length
54 A

55 A

56 B

57 A

119 / Pediatric Buster


64. Vocabulary of 1-year-old child is:
(AMU 95)
(a) 1-10 words
(b) 10-20 words
(c) 20-30 words
(d) 30-40 words
65. The upper segment: lower segment ratio at 2 years of age in a normal child is:
(KARNAT 96)
(a) 1.8:1
(b) 1.5:1
(c) 1.25:1
(d) 1.21:1
66. Exclusive milk ingestion can manifest as:
(AIIMS 97)
(a) Scurvy
(b) Beriberi
(c) Phrynoderma
(d) Night blindness
67. Secondary lactose intolerance not seen with:
(PGI 96)
(a) Rotavirus diarrhoea
(b) Shigella infection
(c) Entamoeba
(d) Yersinia
68. Anthropometric assessment which does not show much change in 1-4 years:
(PGI 96)
(a) Midarm circumference
(b) Skin fold thickness
(c) Chest circumference: head circumference ratio
(d)
Height
69. Symptoms of minor brain damage:
(KERALA 96)
(a) Fits
(b) Spasticity
(c) Hyperactivity
(d) Hoarse cry
70. Respiratory alkalosis is seen in:
(H/285) (AI 88)
(a) Over dosage with morphine
(b) Salicylate poisoning
(c) Severe asthamatic attack
(d) Advanced
interstitial lung disease
71. Acrodynia results from toxicity due to:
(H/2567) (AI 88)
(a) Lead
(b) Mercury
(c) Arsenic
(d) Zinc
72. Normal arterial blood pH is:
(AI 89)
(a) 7.2 to 7.25
(b) 7.25 to 7.35
(c) 7.35 to 7.40
(d) 7.45 to 7.50
73. Oxygen therapy is likely to improve cyanosis in all the following except:
(AI 89)
(a) Isolated pulmonary stenosis
(b) Interstitial pulmonary fibrosis
(c) Left ventricular failure
(d) Intracardiac right to left shunt
74. Gastric aspirate in excess of 20 ml at birth is suggestive of:
(AI 89)
(a) Swallowed amniotic fluid (b) Cardiac achalasia (c) Duodenal atresia (d) Respiratory distress syndrome
75. Fetal lung maturity is enhanced by maternal intake of all of the following drugs except:
(AI 90)
(a) Phenobarbitone
(b) Dexamethasone
(c) Heroin
(d) Thyroxine
76. The most common complication of amoebic liver abscess in children is:
(AI 90)
(a) Rupture into peritoneal cavity
(b) Rupture into the right lung
(c) Secondary infection
(d) Recurrence
77. Unilateral blood stained nasal discharge in a 6-year-old boy is commonly due to:
(AI 91)
(a) Enlarged adenoids
(b) Chronic maxillary sinusitis
(c) Foreign body in nostril
(d) Choanal atresia
78. Decreased platelets are seen in all of the following conditions except:
(AI 91)
(a) Disseminated intravascular coagulation
(b) Henoch-Schonlein purpura
(c) Myelofibrosis
(d) Acute lymphocytic leukaemia.
79. Decreased serum iron levels and low serum iron binding capacity in a 7-year-old child with anaemia indicate:
(AI 91)
(a) Iron deficiency anaemia
(b) Beta thalassaemia
(c) Chronic infection
(d) Sideroblastic anaemia
80. The most common tumour of bladder in children is:
(AI 92)
(a) Papilloma
(b) Haemangioma
(c) Adenoma
(d) Rhabdomyosarcoma
81. A child presented with defective Moros reflex; preserved grasp reflex, the position of limb is in supination with
limitation of abduction and external rotation, the site of lesion is:
(AIIMS/99)
(a) C3, C4
(b) C5, C6
(c) C7, C8
(d) T1, T2
CMV disease: Transmitted through oropharyngeal secretion, urine, cervical and vaginal excretions,
spermatic fluid; breast milk and blood. Congenital CMV infection develops in approximately 5% of
infected fetuses and risk to foetus is greatest when the pregnant woman has primary CMV infection.
Only 1% foetus are infected in women with recurrence of previous infection
90-95% of infected foetuses are asymptomatic
The asymptomatic ones develop psychomotor, senosineural hearing loss, ocular or dental
abnormalities over the next years.
CF: Anemia, thrombocytopenic purpura, hepatosplenomegaly, neonatal hepatitis, jaundice, meningoencephalitis; microcephaly, intracraninal calcification and chorioretinitis, petechial rash. Diagnosis is
by serum lgM and SGOT; confirmed by isolation of CMV.
64 B
77 C

65 D
78 B

66 A
79 C

67 A
80 D

68 A
81 B

69 C

70 B

71 B

72 C

73 D

74 A

75 B

76 B

Self-assessment / 120
82. The use of steroids in neonates is required in:
(a) Bronchial aplasia
(b) Meconium aspiration syndrome
(c) Congenital polycystic disease
(d) Craniopharyngioma
83. Prednisolone is the treatment of choice in a haemophiliac with:
(a) Spontaneous haematuria (b) Gingival bleeding (c) Traumatic hemarthrosis (d) None

(AI 94)

(AIIMS 81)

84. Vomiting in first day of baby; cause is:


(AI 97)
(a) Pyloric stenosis
(b) Oesophageal atresia
(c) Aerophagy
(d) Amniotic gasteritis
85. The following has antibody transfer of mother to children except:
(AIIMS 96)
(a) Rh-incompatibility
(b) Iso-thrombocytopenia
(c) Toxic erythema
(d) Myasthaenia gravis
86. Recurrent respiratory infection is seen in A/E:
(AIIMS 96)
(a) Reflux oesophagitis
(b) Tetralogy of Fallot
(c) Ataxia telangietasia
(d) Ventricular septal defect
87. Exclusively milk fed diet has deficiency of:
(AIIMS 96)
(a) Vitamin A
(b) Vit D
(c) Vit C
(d) Vit B
88. An infant on breast milk has golden yellow stool and evening colic the problem diagnosis is: (AIIMS 96, AI 98)
(a) Vitamin K deficiency
(b) Iron deficiency
(c) Iodine deficiency
(d) Vitamin C deficiency
89. Rectal polyp usually present with:
(UP 93)
(a) Obstruction
(b) Perforation
(c) Bleeding
(d) Malignant change
90. Alveolar arterial tension gradient increases in A/E:
(UP 96)
(a) Diffusion defects
(b) Hypoventilation
(c) R >L shunt
(d) Ventilation perfusion abnormality
91. Most common cause of bleeding per rectum in children is:
(UP 97)
(a) Rectal polyp
(b) Intussusception
(c) Foreign body
(d) Hirchsprungs disease
92. Calculate the percentage of burns on the head, neck and face in a child of one year:
(PGI/86)
(a) 10%
(b) 16%
(c) 13%
(d) 15%
93. Which of the following is not transmitted transplacentally?
(AI 88)
(a) Toxoplasma
(b) Cytomegalovirus
(c) Herpes
(d) Hepatitis
Maternal Infections transmitted to foetus are:
Cord blood lgM is usually found to be elevated in cases of intrauterine infection
CMV
Herpes simplex
Hepatitis
Coxsackies
Rubella
Mumps
Toxoplasma, Malaria
Treponema
Listeria, Grp B
Steptococci and E. coli
Enterovirus
94. A 5-year-old child has anemia of long duration. The investigation to be done is:
(a) Estimation of Hb%
(b) RBC count
(c) Peripheral smear
(d) PCV
95. A child comes with migratory polyarthritis, the investigation of choice to confirm
(a) Pharyngeal swab for culture and sensitivity
(b) ASLO titre
(c) ESR
96. A 2-year-old child has a weight of 6.4 kg and has vitamin A deficiency. What is the
child?
(a) First degree
(b) Second degree
(c) Third degree
(d) Fourth degree
97. The percentage of calories to be supplied by proteins is:
(a) 5-10%
(b) 10-15%
(c) 15-20%
(d) More than 20%
98. Leucaemoid reaction is seen in:
(a) Acute infection
(b) Myelomatosis
(c) Haemorrhage
(d) Erythroleukemia
99. The malignancies of childhood include all except:
(a) ALL
(b) Wilms tumour
(c) Retinoblastoma
(d) AML
82 A
95 B

83 A
96 D

84 B
97 C

85 C
98 A

86 B
99 D

87 C

88 B

89 C

90 C

91 A

(AI 88)
the diagnosis would be:
(d) ECG
(AI 87)
grade of malnutrition in this
(AIIMS 87)
(PGI 87)
(PGI 87)
(e) Bronchogenic carcinoma
(PGI 87)
92 B

93 C

94 C

121 / Pediatric Buster


100. Most virulent variety of shigellosis is caused by:
(AIIMS 86)
(a) S. shigella
(b) S. sonnei
(c) S. flexneri
(d) S. boydii
101. Oxygen therapy is likely to improve cyanosis in all the following except:
(AI 89)
(a) Isolated pulmonary stenosis
(b) Interstitial pulmonary fibrosis
(c) Left ventricular failure
(d) Intracardiac right to left shunt
102. Generalised pruritus can be associated with all of the following except:
(AI 89)
(a) Hodgkins disease
(b) Rheumatoid arthritis
(c) Old age
(d) Lichen planus
103. Most virulent form of pneumococcus is:
(AIIMS 86)
(a) Type A
(b) Type B
(c) Type C
(d) All are equally virulent
104. Sitting height is equal to:
(ASSAM 95)
(a) Head circumference
(b) Chest circumference
(c) Upper segment
(d) Crown heel length
105. A 2-year-old child comes with discharge, seborrheic dermatitis, polyuria and hepatosplenomagaly. Which of the
following is the most likely diagnosis?
(NEL/1729) (AIIMS / 04)
(a) Leukemia
(b) Lymphoma
(c) Langerhans cell histiocytosis
(d) Germ cell tumor.
106. All of the following drugs are contraindicated in patients with G-6-PD deficiency, except: (Nel/1637) (AIIMS /04)
(a) Co-trimoxazole
(b) Furazolidone
(c) Nalidixic acid
(d) Ceftriaxone.
Agents known to cause hemolysis in G-6-PD deficiency subjects
Antimalarials: Primaquine, quinine, mepacrine (quinacrine)
Nitrofurans: Nitrofurantoin, furazolidin, nitrofurazone
Sulfas: Sulfacetamide, sulfametghoxypyridazine, sulfisoxazole, acetyl sulfanilamide
Sulfones: Diaminodiphenylsulfone (DDS), sulfoxones, thiazolsulfone
Antipyretics: Phenacetin, antipyrine (phenazone), aspirin, amidopyrine
Antibiotics: Nalidixic acid, gentamicin, kanamycin, cloxacillin, PAS, novobiocin, chloramphenicol
Vitamins: Vitamin K1 (water-soluble analogues), large doses of vitamin C
Miscellaneous: Quinidine, BAL, methylene blue, probencid, naphthalene (moth balls), fava beans (broad
beans).
107. Regarding primary peritonitis which of the following is correct?
(KERALA 87)
(a) Caused by pneumococcus
(b) Often seen in female
(c) Due to ascending vaginal infection
(d) Associated with steroid therapy
(e) All
Acute Primary Peritonitis:
Most cases occur in children with ascites resulting from nephrotic syndrome or cirrhosis.
M.C. bacteria is pneumococci others being group A streptococci, enterococci; Staph; E. coli and
Klebsiellae
Equal predisposition for male and female and mostly occur before 6 yr.
Hypotension and tachycardia are common with a toxic appearance.
Prior use of corticosteroids may diminish the clinical expression.
108. Streptococcal vaginitis in a child is treated with:
(PGI/86)
(a) Systemic penicillin
(b) Local gentian violet
(c) Oestrogens
(d) Penicillin and oestrogens
109. Phototherapy reduces unconjugated bilirubin to conjugated bilirubin by:
(AIIMS 86)
(a) E.isomerisation
(b) H. isomerisation
(c) B.isomerisation
(d) Increasing hepatic enzyme levels
110. Vaccine is available against which type of Pneumococcus:
(AIIMS/86)
(a) Type A
(b) Type B
(c) Type C
(d) Types A and C
(e) Types B and C
111. Acute osteomyelitis in infants is best diagnosed by:
(JIPMER 87)
(a) History and high degree of suspicion
(b) X-ray
(c) Blood culture
(d) Elevated WBC count
112. Syphilis attack the foetus after the month:
(JIPMER/87)
(a) 2nd
(b) 3rd
(c) 4th
(d) 7th
113. Dose of chloroquine at 4.0 years:
(AI 89)
(a) 150 mg
(b) 300 mg
(c) 450 mg
(d) 600 mg
100 A
113 B

101 D

102 C

103 C

104 D

105 C

106 D

107 E

108 D

109 A

110 D

111 A

112 A

Self-assessment / 122
114. Infantile diarrhoea can be caused by all except:
(AI 89)
(a) Rotavirus
(b) Retrovirus
(c) Adenovirus
(d) Calci virus
115. Suppression of lactation is indicated in all except:
(AI 89)
(a) Neonatal death
(b) Mid trimester abortion
(c) IDDM
(d) Lithium administration
116. The most common feature of hypothyroidism in children is:
(PGI/87)
(a) Cataract
(b) Recurrent seizure
(c) Cold extremities
(d) Laryngospasms
117. Haematoma of the sternomastoid muscle detected in a 16-days-old infant requires:
(BHU 87)
(a) Immediate surgical evacuation
(b) Prophylactic antibiotic therapy
(c) No immediate therapy
(d) Surgical intervention within 2 weeks
118. A 4 kg infant with severe dehydration (10% dehydration) needs the following amount of intravenous fluid in the
first 24 hours:
(JIPMER 80, PGI 83)
(a) 500 ml
(b) 800 ml
(c) 100 ml
(d) 1200 ml
119. Neonatal thrombocytopenia is most commonly due to:
(AIIMS/99)
(a) Auto immune thrombocytopenia
(b) Isommune thrombocytopenia
(c) SLE
(d) None of the above
120. Causes of neonatal seizure which carry the best prognosis:
(JIPMER/84)
(a) Birth asphyxia
(b) Hypoglycaemia
(c) Bacterial meningitis
(d) Late onset hypocalcemia
121. The most common complication following smallpox vaccination:
(AIIMS 83)
(a) Allergic rashes
(b) Eczema vaccination
(c) Encephalitis
(d) Vaccina gangrenosa
122. The most common cause of aplastic anemia is:
(AIIMS 83)
(a) Idiopathic
(b) Chloramphenicol
(c) Phenylbutazone
(d) Petroleum products
123. Posterior cervical lymph node enlargement in children is most often due to:
(AIIMS 83)
(a) TB
(b) Hodgkins disease
(c) Acute tonsillitis
(d) Pediculosis capitis
124. Recurrent abdominal pain in children most often due to:
(AIIMS 83)
(a) Roundworms
(b) Emotional/behavioural problems
(c) Amoebiasis
(d) Giardiasis
125. In the management of a child unconscious from poisoning; the first thing to do is to:
(AIIMS 83)
(a) Establish airway
(b) Take detailed history
(c) Blood for chemical analysis
(d) Gastric wash out
126. From which of the following the highest percentage of iron is absorbed?
(AIIMS 83)
(a) Spinach
(b) Soya bean
(c) Mutton
(d) Egg
127. The most important factor to overcome protein-energy malnutrition in children less than 3 years is: (AIIMS 83)
(a) Supply of subsidised food from ration shop
(b) Early supplementation of solids in infants
(c) Immunization to the child
(d) Treatment of anemia and pneumonia in infant and toddlers
128. The most common cause of neonatal death in India is:
(AIIMS 83)
(a) Prematurity
(b) Congenital malformations
(c) Metabolic disease
(d) Birth injury
129. Breastfeeding is best for the baby:
(AIIMS 83)
(a) But has no effect on the mother
(b) But is likely to affect the mothers appearance significantly
(c) But
may cause breast cancer in the mother
(d) And is also good for the mother
130. The average faecal sodium excretion in rotavirus diarrhoea is:
(AIIMS 84)
(a) 120 mEq/L
(b) 90 mEq/L
(c) 30 mEq/L
(d) 75 mEq/L
131. Breastfed babies as compared to artificially fed babies have all the following benefits except:
(AIIMS 84)
(a) Lower incidence of diarrhoea episodes
(b) Lower incidence of respiratory infection
(c) Lower incidence
of late onset tetany
(d) Lower incidence of future obesity
ADVANTAGES OF BREASTFEEDING
Counters risk of allergic disorders like asthma and eczema
Cancer (lymphoma) risk is reduced
Configuration of jaw is better because of the mechanism of sucking at the breast.
Caries in teeth are less common.
Coronary artery disease risk in later life is reduced.
Clever children: Premature babies who were breastfed, turned out to be more intelligent than the
artificially fed.
Crib death or sudden infant death syndrome risk is probably reduced in breastfed babies.

114 B
127 B

115 C
128 A

116 B
129 D

117 C
130 A

118 B
131 D

119 B

120 D

121 C

122 A

123 D

124 C

125 A

126 C

123 / Pediatric Buster


132. Delayed speech in a 5-year-old child with normal motor and adaptive development is most likely due to:
(a) Mental retardation
(b) Cerebral palsy
(c) Kernicterus
(d) Deafness
(AIIMS 84)
133. A 5-year-old child is assessed to have developmental age of one year. His developmental quotient would be:
(a) 100
(b) 80
(c) 60
(d) 20
(AIIMS 84)
134. A 3-year-old child presented with gradually developing swelling over 7 days, his BP was 104/76 mmHg; urine
examination shows albumin +++ hyaline cast +. The most likely diagnosis will be:
(AIIMS/99)
(a) Selective proteinuria
(b) Increased blood urea
(c) Decreased blood volume
(d) Decreased complement
135. The most common cause of blindness in children is:
(AIIMS 85)
(a) Congenital rubella
(b) Meningitis
(c) Vitamin A deficiency
(d) Thiamine deficiency
136. Costochondral junction swelling is seen in:
(AMU/84)
(a) Scurvy
(b) Rickets
(c) Chondrodystrophy
(d) All of the above
137. Most common cause of severe haematemesis in a child is:
(TN/92)
(a) Portal hypertension
(b) Peptic ulcer
(c) Mallory-Weiss syndrome
(d) None
138. An infant presented with a compressible swelling at the root of nose, the most probable diagnosis will be:
(AIIMS/99)
(a) Meningoencephalocoele
(b) Lacrimal cyst
(c) Epibulbar dermoid
(d) Ethmoid mucococle
139. True about normal thymus in neonate in X-ray:
(AIIMS/99)
(a) Causes mediastinal enlargement (widening)
(b) Displacement of trachea
(c) Lateral sharp border with
snail like feature
(d) Reduces with steroid
140. Absolute lymphocytosis occurs in all except:
(AIIMS 89)
(a) Mumps
(b) Whooping cough
(c) Enteric fever
(d) Infections
141. The following is of serious pathological significance in infants:
(KARNAT 89)
(a) Loss of weight
(b) Palpable left kidney
(c) Palpable spleen
(d) Deviation of trachea from midline
142. The most common cause of enlarged cardiac shadow in X-ray of a child is:
(TN 89)
(a) PDA
(b) Coarctation of aorta
(c) Pericarditis
(d) Rheumatic carditis
143. The number of fontanelles present in a newborn child is:
(KERALA 90)
(a) 1
(b) 2
(c) 4
(d) 6
144. All requires hospitalisation except:
(AI 91)
(a) 5% burns in children
(b) 10% scalds in children
(c) Electrocution
(d) 15% deep burns in adults
145. True about bones in children is A/E:
(AI/2000)
(a) Thickened periosteum
(b) Spongy bone
(c) Commonly comminuted bone
(d) Dislocation rare
146. A comatose child has a garlic like odour to the breath. The most probable diagnosis is:
(JIPMER 81)
(a) Flavism
(b) Atropine
(c) Lead poisoning
(d) Arsenic poisoning
147. A child takes self decisions at the age of.... years:
(KERALA 91)
(a) 3
(b) 5
(c) 7
(d) 9
148. Which of the following is incorrectly matched:
(JIPMER 91)
(a) StutteringToddler
(b) Sleep pattern of infancy2 to 3 years
(c) Fearspreschool
(d) Oedipus complexinfancy
149. Fetal respiratory movements occur earliest at:
(JIPMER 91)
(a) 12 wk
(b) 16 wk
(c) 20 wk
(d) 11 wk
150. Which does not cause pneumonia:
(AIIMS 91)
(a) Measles
(b) Varicella
(c) Mumps
(d) Cytomegalovirus
151. Normal pressure in hydrocephalus is characterised by all except:
(AI 92)
(a) Aphasia
(b) Ataxia
(c) Dementia
(d) Urinary disturbances
152. Fatty acid necessary during 0 to 6 months of age is:
(JIPMER 92)
(a) Linolec acid
(b) Linolenic acid
(c) Arachidonic acid
(d) palmitic acid
153. To evaluate jaundice in a neonate, what is measured:
(AI/2000)
(a) Total bilirubin
(b) Total and direct bilirubin
(c) Indirect bilirubin
(d) Direct bilirubin
154. Which is not a tumour of the first decade of life:
(AIIMS 92)
(a) Rhabdomyosarcoma
(b) Neuroblastoma
(c) Amyloblastoma
(d) Retinoblastoma
155. How many digits can a five-year-old child remember:
(PGI 93)
(a) 4
(b) 5
(c) 8
(d) 10
132 D
145 C

133 D
146 D

134 A
147 A

135 C
148 D

136 D
149 D

137 A
150 C

138 A
151 A

139 C
152 A

140 C
153 B

141 A
154 C

142 D
155 B

143 B

144 A

Self-assessment / 124
156. Which enzymatic reaction is depressed in malnutrition:
(PAL 93)
(a) Conjugation
(b) Carboxylation
(c) Oxidation
(d) Hydroxylation
157. Most common cause of intracerebral calcification in a child in India is:
(JIPMER 93)
(a) Tuberculoma
(b) A-V malformation
(c) Hyperparathyroidism
(d) Toxoplasmosis
158. Lutembachers syndrome refers to:
(JIPMER 79, AIIMS 81)
(a) Ostium primum defect with mitral stenosis
(b) Ostium secundum defect with mitral stenosis
(c) Ostium secundum defect with mitral incompetence
(d) None
159. Spasticity is associated with all except:
(PGI 80, AMU 86)
(a) Clonus
(b) Quadriplegia
(c) Clasp-knife rigidity
(d) LMN type paralysis
160. The behaviour of an adolescent is probably best described as being:
(JIPMER 80, AIIMS 83)
(a) Domineering
(b) Belligerent
(c) Aggressive
(d) Paradoxical
161. A child with increased intracranial tension (BP-160/120) and papilledema comes with convulsion, he was given
diazepam to control seizures, next step of treatment will be:
(AI/2000)
(a) I/V frusemide
(b) Sublingual nifedipine
(c) Sublingual nitrate
(d) Nitropruside infusion
162. Child with which of the following throat infection needs systemic antibiotic therapy: (JIPMER 81, DELHI 92)
(a) Pneumococci
(b) B. haemolytic streptococci
(c) Staph aureus
(d) All
163. The major site of active haemopoiesis in a foetus of 5 months is:
(AIIMS 81, PGI 81)
(a) Spleen
(b) Liver
(c) Lymph nodes
(d) Bone marrow
164. Under pulse polio programme, target age group is:
(JIPMER 98)
(a) Under 1 year
(b) Under 3 years
(c) Under 5 years
(d) Under 10 years
165. All the following hormones can affect growth of a child except:
(JIPMER 98)
(a) GH
(b) ACTH
(c) Insulin
(d) Somatostatin
166. What is correct in child:
(AIIMS 89)
(a) Bacterial sore throats most common
(b) Antibiotics required in all sore throats
(c) Antibiotics required
in cervical lymphadenitis with fever
(d) Tonsillectomy is indicated in recurrent sore throat
167. Diagnosis should be made only after abduction and external rotation, the site of lesion is:
(a) C3, C4
(b) C5, C6
(c) C7, C8
(d) T1, T2
(AIIMS/99)
168. A 6-day-old newborn infant develops ketonuria, seizures and hypoglycemia. The likely diagnosis is: (AIIMS/99)
(a) Tyrosinemia
(b) Aromatic aminoaciduria
(c) Phenylketonuria
(d) Intrauterine infections
169. The urine of newborn infants may show:
(AIIMS 91)
(a) Glycosuria
(b) Transient proteinuria
(c) Symptomatic bacteriuria
(d) Microscopic hematuria
170. In neonate with respiratory distress with multiple cavitary lesions on X-ray chest is:
(AIIMS 92)
(a) Diaphragmatic hernia
(b) Multiple congenital cysts
(c) Pneumatocoeles
(d) Obstructive emphysema
171. An infant with dyspnoea and multiple translucencies of whole left hemithorax is suffering from: (AIIMS 92)
(a) Pneumatocoele
(b) Multiple congenital lung cyst
(c) Diaphragmatic hernia
(d) Cystic fibrosis
172. In an infant exclusively fed on breast milk with pallor, hepatomegaly but no splenomegaly, investigation of
choice is:
(AIIMS 92)
(a) Serum iron levels
(b) HbF levels
(c) Serum B12 levels
(d) Serum folic acid levels
173. Taste perception of baby develops at:
(TN/99)
(a) Birth
(b) 4 months
(c) 6 months
(d) 8 months
174. Deficiency of which vitamin is not known in newborn:
(AIIMS 92)
(a) C
(b) D
(c) E
(d) K
175. Artificial feeding causes following except:
(AIIMS 91)
(a) Eczema
(b) Late onset tetany
(c) Haemorrhagic disease
(d) Necrotising enterocolitis
176. Which is true in growth of children:
(AIIMS 92)
(a) Should not be < 5 cm/year in prepubertal age (b) Growth is by obesity
(c) Prepubertal growth depends
on basal GH level
(d) Gonadal growth parallels somatic growth
177. Newborn child not passed urine for 36 hours, investigation method of choice:
(AIIMS 95)
(a) Ultrasound of kidney and bladder
(b) X-ray pelvis
(c) Cystoscopy
(d) CT scan
178. Following are true about Lawrence-Moon-Biedl syndrome except:
(AIIMS 95)
(a) Mental retardation
(b) Digital anomalies
(c) Hypogonadism
(d) Asthenic build

156 A
169 A

157 A
170 B

158 B
171 B

159 D
172 A

160 D
173 A

161 D
174 C

162 D
175 B

163 D
176 C

164 C
177 A

165 D
178 D

166 C

167 B

168 B

125 / Pediatric Buster


179. During CPR, calcium is not used if:
(AIIMS 97)
(a) Hypokalaemia
(b) Hyperkalaemia
(c) Hypocalcaemia
(d) Calcium antagonist therapy
180. Gonads have to be removed in:
(AIIMS 97)
(a) Pure gonadal dysgenesis
(b) Testicular dysgenesis
(c) Kllmanns syndrome
(d) Turners syndrome
181. Blood stained diaper in children is due to:
(AIIMS 98)
(a) Urethral haemangioma
(b) Meatal stenosis
(c) Bilharzhiasis (schistosomiasis)
(d) Sickle cell trait
182. Cystometric studies are useful in:
(AIIMS 98)
(a) Fistula
(b) Urge incontinence
(c) Stress incontinence
(d) Neurogenic bladder
183. Drug that contraindicated in infants:
(CUPGEE/99)
(a) Ciprofloxacin
(b) Barbiturate
(c) Theophilline
(d) Chloroquine
184. MC cause of pseudopancreatic cyst in children is:
(AI 99)
(a) Choledochal cyst
(b) Traumatic pancreatitis
(c) Annular pancreatitis
(d) None of the above
185. Incubators for neonates transfer energy by:
(PGI/98)
(a) Radiation
(b) Convection
(c) Conduction
(d) None
186. Capillary filling in neonates observed in:
(PGI/98)
(a) Palm
(b) Sternum
(c) Ear lobes
(d) Finger nails
187. A four-day-old baby, not being breastfed, given cane sugar feeds, presents with convulsions, lethargy, hepatomegaly, hypoglycemia, reducing sugar present in urine will indicate:
(AIIMS/2K)
(a) Glucose-6-phosphatase deficiency
(b) Aldolase B deficiency
(c) Fructokinase deficiency
(d) Betagalactosidase deficiency
188. Which of the following is a characteristic radiological finding in neonatal necrotizing enterocolitis: (JIPMER/2K)
(a) Gas in the portal system
(b) Gas in the intestinal wall
(c) Pneumoperitoneum
(d) Air fluid levels
189. Ramu, 8 years old boy presents with upper GI bleeding. On examination, there is splenomegaly, no ascites, no
hepatomegaly; esophageal varices present on UGIE. Diagnosis is:
(AI/2001)
(a) Budd-Chiari syndrome
(b) Non-cirrhotic portal fibrosis
(c) Cirrhosis
(d) Veno-occlusive disease
190. A neonate delivered at 32 weeks, on ventilator, X-ray shows white out. ABG reveals SpO2 of 75, ventilator settings
are as, FiO2, 70 rate 50/minute. Next step to be taken?
(AI/2001)
(a) Increase rate to 60
(b) Increase FiO2 to 80
(c) Continue with the same
(d) Weaning ventilator
191. A neonate presents with respiratory distress, contralateral mediastinal shift and multiple cystic air filled lesions
in chest. Most likely diagnosis is:
(AI/2001)
(a) Congenital diaphragmatic hernia
(b) Congenital lung cysts
(c) Pneumonia
(d) Emphysema
192. A 5 years child presents with calculus of size 2 cm in the upper ureter. He has haematuria on USG no obstruction
is seen. TT of choice is:
(AI/2001)
(a) Ureterolithotomy
(b) Endoscopy
(c) ESWL
(d) Observation with masterful inactivity
193. Ground glass appearance is not seen in:
(AI/2001)
(a) Hyaline membrane disease
(b) Pneumonia
(c) Left to right shunt
(d) Obstructive TAPVC
194. The cell chemistry in a 2 years old child presenting with history of diarrhoea and inadequate water intake will
be:
(OPG/195) (AIIMS/2000)
(a) Raised ECF with rise in sodium concentration
(b) Raised ECF with fall in sodium concentration
(c) Decreased ECF with rise in sodium concentration
(d) Decreased ECF with fall in sodium concentration
195. A 7-day-old child presents with seizure, skin rash, lactic acidosis, raised NH3 and acidic urine. The diagnosis
will be:
(AIIMS 2000)
(a) Phenylketonuria (b) Organic aciduria (c) Mitochondrial disease which affect the urea cycle (d) Tetanus
neonatorum
196. A child presented in emergency department with severe respiratory distress and diminished breath sounds on
right side. The X-ray shows opacity on right side. The diagnosis goes in favour of:
(AIIMS 2000)
(a) Pleural effusion
(b) Emphysema
(c) Foreign body
(d) Brochiolitis
197. The most common soft tissue tumor in children:
(PGI/2000)
(a) Liposarcoma
(b) Rhabdomyosarcoma
(c) Histiofibrocytoma
(d) Wilms tumour
179 A
192 C

180 A
193 C

181 A
194 D

182 B
195 B

183 A
196 A

184 C
197 B

185 C

186 C

187 B

188 B

189 B

190 D

191 B

Self-assessment / 126
Soft tissue sarcomas: (Rhabdomysarcoma)arises from
Unsegmented mesoderm.
M.C. age group affected is 2-6 yr abd adolescent.
Most common type is of embryonal type and have best prognosis.
M.C. 40% arise in head and neck and 30% in urogenital region.
M.C. site in males is bladder or prostate.
Lymph node metatases are extremely common.
Racquet shaped cells are found.
Most valuable for certain identificationcross striations; desmin and myoglobin.
Treatment is surgical resection of tumor mass plus chemotherapy and radiotherapy.
Commonly used drugs in chemotherapy is vincristine, actinomycin D and cyclosphosphamide.
198. Neonatal herpes is caused by:
(PGI 2000)
(a) HSV-2
(b) HSV-1
(c) HPV
(d) HHV-8
199. If a 2-year-old child develops leukemia, which one of the following radiological findings is likely to occur:
(PGI 2000)
(a) Osteoporosis of long bones
(b) Periosteal new bone formation
(c) A radiolucent rim near the epiphysis
(d) Osteolytic lesions
200. Production of adult hemoglobin in the fetus occurs from:
(PGI 2000)
(a) 5th week
(b) 10th week
(c) 11th week
(d) 20th week
(e) 25th week
201. Which of the following is true regarding larynx of neonates:
(PGI 2000)
(a) Larynx is present at the level of C4,5,6
(b) Epiglottis is large and leafy
(c) Glottis is the narrowest
part
(d) Tongue is small compared to the oral cavity
202. As per the WHO, for infant less than 6 months, the Hb cut off level is:
(AIIMS/NOV/01)
(a) 100 g/L
(b) 115 g/L
(c) 105 g/L
(d) 110 g/l
203. Breathing movements and swallowing movement seen in the foetus by:
(AIIMS/NOV/01)
(a) 13-14 weeks
(b) 18-24 weeks
(c) 15-18 weeks
(d) > 30 weeks
204. A child with recent onset of URTI after 2 days presents with acute onset of breathlessness, cough and fever. All
of the following can be given except:
(AIIMS/MAY /01)
(a) Antibiotics
(b) Antipyretics
(c) O2 inhalation
(d) Morphine
205. If in a community a high proportion of newborn babies suffers from methaemoglobinaemia, then which of the
following inferences would be most relevant:
(UPSC/01)
(a) The community is exposed to industrial smoke
(b) The local water supply is very rich in nitrites
(c) The babies are suffering from the effects of mercury poisoning
(d) The hemoglobin of the babie's blood
is in a very reduced state
206. Most common cause of death in children< 5 year age is:
(UPSC/01)
(a) Respiratory infection
(b) Diarrhoea
(c) Prematurity
(d) Accidents
207. Water content in infant:
(Orissa 01)
(a) 60-70%
(b) 75-80%
(c) 80-90%
(d) > 90%
208. Gender of external genitalia of foetus becomes clearly distinguished by:
(Orissa 01)
(a) 10 weeks
(b) 16 weeks
(c) 12 weeks
(d) 20 weeks
209. which one of the following pairs of diseases and features is not correctly matched?
(UPSC/01)
(a) Acute glomerulonephritishypertension
(b) Nephrotic syndromepneumococcal peritonitis
(c) Kwashiorkor pulmonary oedema
(d) Hepatic encephalopthyhypoglycemia
210. Consider the following intracranial haemorrhages:
(UPSC/01)
1. Intraventricular
2. Subdural
3. Subarachnoid
Which of these types of haemorrhages in a newborn are due to anoxia?
(a) 1, 2 and 3
(b) 1 and 2
(c) 2 and 3
(d) 1 and 3

198 A

199 B

200 E

201 C

202 B

203 C

204 D

205 B

206 A

207 B

208 A

209 C

210 D

127 / Pediatric Buster


211. A couple with a family history of beta thalassemia major in a distant relative, has come for counseling. The
husband has HbA2 of 4.8% and the wife has HbA2 of 2.3%. The risk of having a child with beta thalassemia
major is:
(OPG5th/310-)
15
(a) 50%
(b) 25%
(c) 5%
(d) 0%
(Hb /673) (AI/03)
212. An albino girl gets married to a normal boy. What are the chances of their having an affected child and what
are the chances of their children being carriers?
(AI/03)
(a) None affected, all carriers
(b) All normal
(c) 50% carriers
(d) 50% affected, 50% carriers
213. A 45-day-old infant developed icterus and two days later symptoms and signs of acute liver failure appeared.
Child was found to be postitive for HBsAg. The mother was also HBsAg carrier. The mothers hepatitis B
serological profile is likely to be:
(H15/1729) (AI/03)
(a) HBsAg positive only
(b) HBsAg and HbeAg positivity
(c) HBsAg and anti-HBe antibody positivity
(d)
Mother infected with mutant HBV.
214. In which one of the following conditions dactylitis cannot be seen?
(UPSC/02)
(a) Sickle cell anemia
(b) Beta thalassemia
(c) Congenital syphillis
(d) Tuberculosis
215. The commonest cause of bleed per rectum in children is:
(UPSC/02)
(a) Rectal polyp
(b) intussusception
(c) Meckels diverticulum
(d) Haemorrhoids
216. A 3-year-old child develop severe dehydration following acute gastroentertis, investigations show blood urea 60
mg/dl, pH 7.27, base excess 10 mEq/L, sodium 134 mEq/L and potassium 3.8 mEq/L. The most appropriate fluid
for initial treatment of this child is:
(UPSC/02)
(a) N/3 saline in 5% dextrose
(b) N/3 saline in 10% dextrose
(c) Normal saline with 5% dextrose
(d) 3% saline
217. Advanced paternal age is associated with
(UPSC/03)
(a) New mutations of autosomal dominant disorders
(b) Autosomal recessive disorder (c) Cornelia de Langes
syndrome
(d) Downs syndrome
218. The ossification centre of cuboid bone appears at the gestational age of:
(UPSC/03)
(a) 16 weeks
(b) 28 weeks
(c) 32 weeks
(d) 40 weeks

POINTS TO PONDER
In born Errors of Amino Acid Metabolism Associated with Abnormal Odor
Inborn Error of Metabolism

Urine Odor

Glutaric acidemia (Type II)


Hawkinsinuria
Isovaleric acidemia
Maple syrup urine disease
Hypermethioninemia
Multiple carboxylase deficiency
Oasthouse urine disease
Phenylketonuria
Trimethylaminuria
Tyrosinemia

Sweaty feet, acrid


Swimming pool
Sweaty feet, acrid
Maple syrup
Boiled cabbage
Tomcat urine
Hops-like
Mousy or musty
Rotting fish
Boiled cabbage, rancid butter

Grading of Nutritional Marasmus


Grades

Characteristics

1.
2.
3.
4.

Loss
Loss
Loss
Loss

211 D

212 C

of
of
of
of

fat from axilla


fat from abdominal wall and gluteal region
fat from chest and back
buccal pad of fat (which consists of fatty acids); it takes longer time to disappear

213 B

214 D

215 A

216 D

217 B

218 D

Self-assessment / 128

Indications for Hospitalization for Burns


Burns greater than 15% body surface area
High-tension wire electrical burns
Inhalation injury regardless of the size of body surface area burn
Inadequate home situation
Suspected child abuse or neglect
Burns to hands, feet, genitals
Congenital/hereditary Conditions Associated with Increased Risk of Leukemias
Ataxia telangiectasia
Congenital X-linked immunodeficiency
Down syndrome
Fanconi anemia
Bloom syndrome
Kostman syndrome
Klinefelter syndrome
Neurofibromatosis
Immunodeficiency
Ataxia telangiectasia

Frequency of Muscle Involvement in Poliomyelitis


Lower limbs
Quadriceps > Tibialis anterior > Peroneal muscles
Upper limbs
Deltoid > Biceps > Triceps
Trunk
Abdominal muscles > Back muscles > Intercostals > Diaphragm
Immunization of HIV Positive Children
Routine pediatric immunization must be given to the HIV infected infants and children with the
following modifications:
In case of asymptomatic cases, OPV, Pneumovax and Haemophilus influenzae vaccines be
omitted.
In case of symptomatic cases, OPV and BCG be omitted.

Poor Prognostic Signs in Meningococcal Disease


Hypotension
Absence of meningitis
Coma
Rapidly progressive purpura, especially of less than 24 hours duration
Hyperpyrexia
Cardiac or renal failure
DIC
Leukopenia, thrombocytopenia, low ESR and high serum antigen concentration
Low CSF polymorphonuclear leukocyte count
Long-term sequelae include sensorineural deafness and psychomotor disability.
Various Complications Occurring in Diphtheria
Bronchopneumonia
Nephritis (mere albuminuria in early stages)
Myocarditis: It may occur any time during the course of the disease. The early development is a bad
prognostic sign.

129 / Pediatric Buster

Paralysis About 12 to 21% of the cases suffer from the post-diphtheretic paralysis somewhere
between first and sixth weeks of illness. The following are the various types seen:
a. Pharyngeal and palatal paralysis are manifested by nasal voice, dysphagia, nasal regurgitation,
and failure to life the palate.
b. Ocular paralysis occurs late and is second in frequency. It is manifested in the form of diplopia,
squint, ptosis, ophthalmoplegias, etc.
c. General paralysis occurs quite late. There may be quadriplegia, paralysis of the neck muscles
and respiratory embarrassment, as a result of involvement of the diaphragm from phrenic nerve
paralysis. Even CSF protein may be elevated. This complication closely simulates GuillainBarr syndrome.
Vasomotor disturbances in the form of hypotension and cardiac failure 2 to 3 weeks after onset of
the disease occur rarely.
Gastritis
Hepatitis
World Health Organization (WHO) Duidelines for Identifying Severe Malaria
More than 2% erythrocytes infected with malarial parasites
Asexual parasite count 100,000/mm3 of blood
High fever with body temperature 105F (40.5C) and above
Severe anemia with hemoglobin under 5 g%
Hematocrit (PCV) 30%
Blood urea 55 mg
Acute hepatopathy and clinically detectable jaundice
Cerebral malaria (headache, mental disturbance, neurologic signs, convulsions, delirium, coma)
Renal complications like acute renal failure, acute tubular necrosis, dark urine, etc.
Hypoglycemia
Noncardiogenic pulmonary edema, shock lung syndrome
Hypovolemia, hypotension, feeble and rapid pulses, pale and clammy skin
Cardiac dysrhythmias
Secretory diarrhea and dysentery
Splanchnic capillary blockade (melana)
Other complications: Rupture of spleen, pyogenic pneumonia, miliary tuberculosis, septicemia,
symptoms resembling hemolytic anemias

Clinical Indicators of Poor Prognosis in Malaria as per WHO


Clinical indicators
Age under 3 years
Deep coma
Witnessed or reported convulsions
Absent corneal reflex
Decerebrate/decorticate rigidity
Clinical signs of organ dysfunction (renal failure, pulmonary edema)
Respiratory distress (acidosis)
Circulatory collapse
Papilledema and/or retinal edema
Laboratory indicators
Hyperparasitemia (>250000/microliter or >5%)
Peripheral schizotaemia
Peripheral blood polymorphonuclear leukocytosis (12000/cu mm)
Mature pigmented parasites (>20% of parasites)
Peripheral blood polymorphonuclear leukocytes with visible malarial pigment (5%)

Self-assessment / 130

Packed cell volume less than 15% or Hb: <5%


Blood glucose less than 2.2 m mol/l (40 mg/dl)
Blood urea more than 60 mg/dL
Serum creatinine more than 3.0 mg/dl
High lactic acid (>6 m mol/L) low CSF glucose
Raised venous lactic acid (>5 m mol/L)
More than three fold elevation of serum enzymes (ALT, AST)
Increased plasma 5-nucleotidase
Low antithrombin III levels
Very high plasma concentrations of tumor necrosis factor (TNF)

Situations for which Prenatal Diagnosis is at Present Available


Chromosomal disordersDown syndrome.
Rh incompatibility.
Genetic/metabolic disorders uchenne muscular dystrophy, glycogen storage disease, Gaucher disease,
familial idiocy, hemophilia, thalassemia, etc.
Anatomic defectsAnencephaly, meningomyeloccle, etc.
Mothers Needing Prenatal Diagnosis
Category of mothers

Risk of fetal defect

1. 35 years and older


2. With history of a previous Mongol baby
3. With history of one or more babies
affected by a neural tube defect
4. With history of a previous baby affected
by a diagnosable autosomal recessive
metabolic disorder
5. Recognised carriers of a severe
X-linked recessive disorder
6. With abnormally high serum AFP
levels found during routine antenatal screening
7. Liable to have chromosomally imbalanced
baby because of partial chromosome
translocation or mosaicism

2.6%
1.0%
5 to 10%
25%

25%
Upto 75%
20%

Clinical Clues to Etiologic Diagnosis


Clues

Organism

Such superficial infections as


Pyoderma, abscess, conjunctivitis,
umbilical sepsis, osteomyelitis;
onset of manifestations after 72 hours of birth

Staphylococcus

Grayish-black gangrenous lesions over skin

Pseudomonas

Peripartum flu-like maternal illness,


gastroenteritis, meconium-stained liquor
amnii, baby is unwell right at birth with
limpness and develops respiratory difficulty,
apneic spells, rash and hepatosplenomegaly on
the first day

Listeria

131 / Pediatric Buster


Maternal fever during labor, prolonged
rupture of membranes (PRM); respiratory
distress within 3 hours of birth, apneic spells, shock

Streptococcus group B

Common Life Threatening Congenital Anomalies


1. Choanal atresia
Respiratory distress in labor room, inability to pass nasal tube.
Suspect CHARGE syndrome (coloboma, heart defect, atresia choane,
retarded growth, genital abnormality, ear anomaly).
2. Tracheo-esophageal fistula
Polyhydramnios, aspiration pneumonia, excessive salivation,
inability to pass NG tube, suspect VATER syndrome
3. Diaphragmatic hernia
Scaphoid abdomen, bowels in chest, respiratory distress, cyanosis,
apparent dextrocardia.
4. Pierre Robin syndrome
Micrognathia, cleft palate, airway obstruction
5. Intestinal obstruction
Polyhydromnios
6. Duodenal atresia
Bile stained vomiting, suspect Down syndrome
7. Ileal atresia
Abdominal distension
8. Omphalocele
Polyhydramnios, intestinal obstruction
9. Neural tube defects
Polyhydramnios
Classification of Mucopolysaccharidoses
Disorders

Defects/Deficiencies

Inheritance

Manifestations

MPS IH (Hurlers
syndrome)

-iduronidase

Recessive

Corneal clouding, pigmentary


retinopathy, mental retardation,
cardiovascular lesions, bone lesion,
stiff joints, coarse facies, optic
atrophy, deafness, dwarfism,
subnormal ERG

MPS IS
(Scheie syndrome)

-iduronidase
(partial)

Recessive

Same as MPS IH, although may


have normal intellect

MPS II (Hunters
syndrome)

Iduronosulfate
sulfatase

X-linked
recessive

Pigmentary retinopathy, corneal


stromal opaticy, mental retardation
bone lesions, dwarfism, retinal
degeneration, optic atrophy, subnormal electroretinograph (ERG)

MPS IIIA (San


Filippo syndrome A)

Heparin
sulfate sulfatase

Recessive

Pigmentary retinopathy, mental


retardation, optic atrophy, retina
vascular narrowing

MPS IIIB (San


Filippo syndrome B)

N-acetyl-D-glucosaminidase

Recessive

Pigmentary retinopathy, mental


retardation, optic atrophy

MPS IV
(Marquio syndrome)

Excess production
of keratosulfate

Recessive

Corneal clouding, skeletal


dysplasia, optic atrophy

MPS VI
(Maroteaux-Lamy
syndrome)

Maroteaux-Lamy
corrective factor

Recessive

Corneal clouding, skeletal


dysplasia

MPS VII
(Sly syndrome)

-glucuronidase

Recessive

None

Self-assessment / 132
Neonatal screening (Infant heel puncture blood sample) includes
Tests for
1. Hypothyroidism
2. Phenylketonuria
3. Galactosemia
4. Maple syrup urine disease
5. Homocystinuria
6. Biotinidase deficiency
7. Adrenal hyperplasia
8. Hemoglobinopathy
9. Cystic fibrosis
10. Tyrosinemia
11. Aminoacidopathies
Conditions associated with antinuclear antibodies
Systemic lupus erythematosus
Drug induced lupus
Juvenile arthritis
Juvenile dermatomyositis
Vasculitis syndrome
Scleroderma
Infectious mononucleosis
Chronic active hepatitis
Hyperextensibility
Autoantibodies found in systemic
Antibody
1. Coombs antibodies
2. Antiphospholipid antibodies
3. Lupus anticoagulant
4. Antithyroid antibodies
5. Antiribosomal P antibody

lupus erythematosus
Manifestation
Hemolytic anemia
Antiphospholipid antibody syndrome
Coagulopathy
Hypothyroidism
Lupus cerebritis

219. Child with fever, convulsion CSF-protein-150 mg, glucose-40 mg%, chloride-520, lymphocytosis; diagnosis is:
(AIIMS 93)
(a) Tubercular meningitis
(b) Viral enchephalitis
(c) Pyogenic meningitis
(d) Viral meningitis
220. All of the following are neural tube defects; except:
(a) Myelomeningocele
(b) Anencephaly
(c) Encephalocele

(AI/04) (NEL/1983)
(d) Holoprosencephaly

Neural tube defects are most common congenital anomalies of CNS


Result from failure of neural tube to close
Major neural tube defects are:
Spina bifida occulta
Meningocele
Myelomeningoele
Encephalocoele
Anencephaly
Dermal sinus
Tethered cord
Syringomyelia
Diastematomyelia
Lipoma involving conus medullaris

219 A

220 D

133 / Pediatric Buster


Disorders of neuraonal migration
1. Lissencephaly or agria
2. Schizencephaly
3. Porencephaly
4. Holoprosencephaly
221. Retardation of skeletal maturity can be caused by all except:
(a) Chronic renal failure (b) Hypothyroidism (c) Protein-energy malnutrition (PEM)
hyperplasia

(AI/04)
(d) Congenital adrenal

Causes of delayed skeletal maturation


1. Malnutrition
a. PEM (most common)
b. Zn deficiency
c. Fe deficiency
2. Stimulant drugs
a. Dextroamphetamine
b. Methyl phenidate
3. IUGR
4. Hornomal abnormalities
a. GH deficiency
b. Thyroid hormone deficiency
c. Diabetes
d. Glucocorticoid excess
5. Chronic disease
a. Renal disease
b. Cardiac disease
c. Pulmonary disease (e.g. cystic fibrosis)
d. GI disorders (e.g. IBD, celiac disease)
e. Hematologic disorders (SCA, thalassemia)
f. Bone disease (e.g. achondroplasic)
6. Chromosomal and genetic disorders
222. Bone marrow transplantation can be used as a treatment for all except:
(a) Osteopetrosis
(b) Adrenoleukodystrophy
(c) Hurlers syndrome
(d) Hemochromatosis
Indications for Stem Cell Transplantation
Malignant
1. Leukemias
1. Acute myeloblastic leukemia
2. Acute lymphoblastic leukemia
3. Chronic myeloid leukemia
4. Chronic lymphocytic leukemia
2. Lymphoproliferative disorders
1. Hodgkins lymphoma
2. Non-Hodgkins lymphoma
3. Multiple myeloma
3. Solid tumors
1. Neuroblastoma
2. Wilms tumor
221 D

222 D

Non-malignant
1.

2.

3.

Bone marrow failure syndrome


1. Aplastic anemia
2. Fanconis aplasia
3. Blackfan-diamond anemia
4. Pure red cell aplasia
Immunodeficdiency states
1. Severe combined immunodeficiency disease
2. Wiscott-Aldrichs syndrome
3. Chronic granulomatous disease
4. Chediak Higashi syndrome
Hematological disorders
1. Thalassemia
2. Sickle cell anemia

(AI/04)

Self-assessment / 134
3. Ewings sarcoma
4. Rhabdomyosarcoma
5. Teratomas
4. Others
1. Breast carcinoma
2. Bronchial carcinoma

4.

3. Congenital neutropenia
4. Glanzmanns thromboasthenia
5. Osteopetrosis
Non-hematological genetic disorders
1. Mucopolysaccharaidosis
2. Leukodystrophy
3. Lysosomal disorders
4. Other metabolic disorders

223. Ataxia telangiectasia is characterized by all of the following except:


(a) Chronic sinopulmonary disease
(b) Decreased levels of -fetoprotein
(d) IgA deficiency

(AI/04)
(c) Chromosomal breakage

Ataxia Telangiectasia
It is an AR inherited disease
Clinical manifestations:
Progressive cerebellar ataxia
Oculocutaneous telangiectasia
Choreoathetosis
Pulmonary and sinus infections
Thymic hypoplasia with cellular and humoral (Ig A and IgG2) immunodeficiencies
Endocrine disorders
Lymphoreticular malignancies
Begins beween 12-15 months
Progresses relentlessly; child wheel chair bound by 10-12 years
Neuroimagingcerebellar atrophy

224. A 12-year-old girl with tremors and emotional lability has a golden brown discoloration of Descemets membrane.
The most likely diagnosis is:
(AI/04)
(a) Fabrys disease
(b) Wilson disease
(c) Glycogen storage disease
(d) Acute rheumatic fever
225. An infant develops cough and fever. The X-ray examination is suggestive of broncho-pneumonia. All of the
following viruses can be the causative agent except:
(AI/04)
(a) Parainfluenza viruses
(b) Influenza virus A
(c) Respiratory syncytial virus
(d) Mumps virus

Mumps virus never causes bronchopneumonia


Other viruses which causes pneumonia in children
1. Coronavirus
2. Coxsackievirus
3. Cytomegalovirus
4. Echoviruses
5. Enterovirus
6. Measles virus
7. Reovirus
8. Varicella-zoster virus

226. A two-year-old boy has vitamin D resistant rickets. His investigations revealed serum calcium 9 mg/dl,
phosphate 2.4 mg/dl, alkaline phosphatase 1041-IU, normal intact parathyroid hormone and bicarbonate 22
mEq/L. Which of the following is the most probable diagnosis?
(AIIMS/04)
(a) Distal renal tubular acidosis
(b) Hypophosphatemic rickets
(c) Vitamin D dependent rickets
(d) Hypoparathyroidism

223 B

224 B

225 D

226 B

135 / Pediatric Buster


227. The renal biopsy of a 6-year-old boy with recurrent gross hematuria shows IgA nephropathy. The urinary protein
excretion is 130 mg/day. Which of the following is the most appropriate next step in the management?
(AIIMS/04) (NEL/1738)
(a) Administer corticosteroids
(b) Give azathioprine
(c) Start cyslosporine
(d) Urinary bag sample
228. Which of the following is the most appropriate method for obtaining a urine specimen for culture in an 8-monthold girl?
(AIIMS/04)
(a) Suprapubic aspiration
(b) Indwelling catheter sample
(c) Clean-catch void
(d) Urinary bag sample
229. An 8-year-old boy during a routine check-up is found to have E. coli 1,00,000 cc/ml on a urine culture. The urine
specimen was obtained by mid-stream clean-catch void. The child is asymptomatic. Which is the most
appropriate next step in the management?
(AIIMS/04)
(a) Treat as an acute episode of urinary tract infection
(b) No therapy
(c) Prophylactic antibiotics for 6
months
(d) Administer long-term urine alkalinizer
230. In which of the following conditions, aniridia and hemihypertrophy are most likely present?
(NEL/1712) (AIIMS/04)
(a) Neuroblastoma
(b) Wilms tumour
(c) Non-Hodgkins lymphoma
(d) Germ-cell tumor
231. Injection glucagon is effective for management of persistent hypoglycemia in all, except:
(AIIMS/04)
(a) Large for date baby
(b) Galactosemia
(c) Infant of diabetic mother
(d) Nesidioblastosis
232. The most appropriate management for maintaining patency of ductus arteriosus in a neonate is:
(NEL/1480) (AIIMS/04)
(a) Prostaglandin E1
(b) Nitric oxide
(c) Oxygen
(d) Indomethacin
Nelson says during foetal life, patency of the ductus arteriosus appears to be maintained by the
combined relaxant effects of low oxygen tension and endogenously produced prostaglandin E2. In a full
term neonate, oxygen is the most important factor controlling ductal closure.

(1) Daptomycin
(2) Gemifloxacin
(3) Nitazoxanide
(4)
(5)
(6)
(7)

Sertaconazole
Alefacept
Aripiprazole
Atomoxetine

(8) Memantine
(9) Enfurvirtide
(10) Aprepitant
(11) Palonosetron
(12) Bortezomib
(13) Tositumomab
and Iodine I 131
(14) Geftinib
(15) Rosuvastatin
(16) Pegvisomant
(17) Adalimumab

227 A

228 A

229 B

NEW DRUGS APPROVED


- Skin and skin structure infections
- For resistant S. pneumonial bacteria
- For treatment of diarrhoea by cryptosporidiosis and giardiasis in 1 to
11 years of age.
- For interdigital tinea pedis
- Immunomodulators (for psoriasis)
- For schizophrenia
- Selective norepinephrine reuptake inhibitor for treatment of attention
deficit hyperactivity disorder.
- N-methyl-D-aspartate receptor antagonist for therapy of Alzheimers
disease.
- Fusion inhibitors for highly active antiretroviral therapy
- Substance P/neurokinin 1 receptor antagonist as antiemetic for
chemotherapy
- Selective serotonin (5HT3) receptor antagonist for emetogenic
chemotherapy
- Reversible prokosome inhibitor for multiple myeloma.
- Anti-CD 20 monoclonal antibody represents dual action therapy for
treating non-Hodgkins lymphoma.
- Selective epidermal growth factor receptor blocker/tyrosine kinase
inhibitor for treatment of non-small cell lung cancer.
- For dyslipidemia
- Growth hormone receptor antagonist for acromegaly
- Recombinant human Ig G1 monoclonal antibody specific for human
tumour necrosis factor
- Useful as DM ARD for rheumatoid arthritis.
230 B

231 A

232 A

Self-assessment / 136
(18)
(19)
(20)
(21)
(22)
(23)
(24)

Lumiracoxib
Teriparatide
Ibandronate sodium
Abarelix
Vardenafil
Tadalafil
Epinastine

Coxa-2 inhibitor for relief of osteoarthritis.


For treatment of postmenopausal osteoporosis.
Nitrogen containing biphosphonate for postmenopausal osteoporosis.
LHRH antagonist for prostate cancer.
For erectile dysfunction
For erectile dysfunction
Antihistaminic with mast cell stabilizing and antiinflammatory
property for allergic conjunctivitis.

Você também pode gostar